SlideShare a Scribd company logo
1 of 163
Download to read offline
NEW 2016 Adult
Reconstructive Surgery
of the Hip and Knee
Self-Assessment Examination
‫راجي‬‫بالل‬
AAOS 2016 Adult Reconstructive Surgery of the Hip and Knee
1
Figure 1a Figure 1b
Question 1 of 200
Figures 1a and 1b are the recent radiographs of an 82-year-old man with rheumatoid
arthritis who underwent total knee arthroplasty (TKA) 18 years ago. These radiographs reveal
osteolysis with loosening of the tibial component. Aspiration and laboratory study findings for
infection are negative. During the revision TKA, treatment of tibial bone loss should consist
of
1- filling the tibial defect with methylmethacrylate.
2- revision of the tibial component with porous metal augmentation.
3- reconstruction with iliac crest bone graft.
4- reconstruction with structural allograft.
PREFERRED RESPONSE: 2- revision of the tibial component with porous metal
augmentation.
DISCUSSION
AAOS 2016 Adult Reconstructive Surgery of the Hip and Knee
2
Video 1 for reference
For severe tibial defects (Anderson Orthopaedic Research Institute [AORI] types 2 and 3),
metaphyseal fixation is necessary to achieve construct fixation during revision TKA.
Metaphyseal fixation may be achieved with cement, structural allograft, or conical metallic
implants. The major concerns regarding structural allograft are graft resorption and mechanical
failure and technical issues related to fashioning the graft and obtaining a good host-allograft
interface. In a systematic review, porous metal cones were associated with a decreased
loosening rate in AORI 2 and 3 defects compared to structural allografts. Metallic trabecular
metal cones and metaphyseal porous coated sleeves provide a stable construct with which to
support the tibial component during revision TKA. Clinical results with these devices include
good metaphyseal fixation for severe tibial bone defects.
Question 2 of 200
Patient-specific instrumentation (PSI) reliably demonstrates which benefit over
conventional intramedullary guidance systems?
1- Lower cost
2- Improved functional outcomes
3- Better coronal alignment
4- Fewer trays
PREFERRED RESPONSE: 4- Fewer trays
AAOS 2016 Adult Reconstructive Surgery of the Hip and Knee
3
DISCUSSION
Cost is usually increased with PSI because the theoretical decreased number of surgical
trays and shortened surgical time do not offset cost of presurgical imaging and extra cost
associated with the necessary jigs. Studies do not demonstrate a reliable improvement in
functional outcomes or coronal alignment when PSI is compared to standard instrumentation.
Evidence reveals that PSI necessitates fewer trays than standard instrumentation.
Question 3 of 200
When performing a posterior cruciate-substituting total knee revision, trial components are
inserted. The knee comes to full extension but is tight in flexion. The surgeon should consider
1- flexing the femoral component.
2- downsizing the femoral component.
3- downsizing the tibial component thickness.
4- resecting more distal femur.
PREFERRED RESPONSE: 2- downsizing the femoral component.
DISCUSSION
In this scenario, the extension gap is normal and the flexion gap is tight. Increasing the
flexion gap without changing the extension gap can be performed by downsizing the femoral
component or adding posterior slope to the tibia resection. Flexing the femoral component
tightens the flexion gap. Decreasing the tibial component thickness loosens the flexion and
extension gaps. Resecting more distal femur only loosens the extension gap.
AAOS 2016 Adult Reconstructive Surgery of the Hip and Knee
4
Figure 4
CLINICAL SITUATION FOR QUESTIONS 4 THROUGH 7
Figure 4 is the radiograph of a 73-year-old woman who returns for her annual follow-up
14 years after undergoing total hip arthroplasty. She denies pain and has no discomfort upon
examination.
Question 4 of 200
Which method of terminal polyethylene sterilization would most likely cause these
radiographic findings?
1- Gamma irradiation in nitrogen
2- Gamma irradiation in air
3- Gamma irradiation in argon
4- Gas plasma
PREFERRED RESPONSE: 2- Gamma irradiation in air
Question 5 of 200
AAOS 2016 Adult Reconstructive Surgery of the Hip and Knee
5
The patient is asked to return in 6 months for a repeat radiograph. Six months later she
remains asymptomatic, but a radiograph reveals progressive osteolysis. Treatment should
consist of
1- femoral head/liner exchange with retroacetabular bone grafting.
2- femoral and acetabular revision.
3- acetabular revision.
4- follow-up in 1 year.
PREFERRED RESPONSE: 1- femoral head/liner exchange with retroacetabular bone
grafting.
Question 6 of 200
The patient begins to experience pain, and a decision is made to proceed with surgical
intervention. When performing a posterior approach to the hip, which structure protects the
anterior retractor from causing damage to the femoral neurovascular structures?
1- Obturator externus
2- Piriformis
3- Psoas
4- Rectus femoris
PREFERRED RESPONSE: 3- Psoas
Question 7 of 200
The patient develops an inability to dorsiflex her foot 2 days after surgical intervention
while she is sitting in a chair after physical therapy. Initial treatment should consist of
1- lying completely supine in bed.
2- remaining seated and placing the postsurgical leg on a stool.
3- transferring back to bed with the head of the bed no lower than 60 degrees.
4- transferring back to bed with the head of the bed level and the surgical knee flexed.
AAOS 2016 Adult Reconstructive Surgery of the Hip and Knee
6
PREFERRED RESPONSE: 4- transferring back to bed with the head of the bed level and
the surgical knee flexed.
DISCUSSION
Gamma irradiation produces free radicals. Although these free radicals can form cross-
links with other polyethylene chains, the free radicals can also form a bond with oxygen,
resulting in early oxidation. Gamma irradiation in air produces the highest risk for oxidized
polyethylene, resulting in the highest risk for wear, delamination, and subsequent osteolysis.
This patient demonstrates severe periarticular osteolysis. When she is asymptomatic, this
suggests the acetabular and femoral components remain well fixed to the bone. Consequently,
she can be treated by removing the wear generator (polyethylene exchange), along with bone
grafting of the osteolytic defect. Considering the extensive amount of osteolysis, observation
for 1 year would not be appropriate.
The psoas is the anatomic structure that runs anterior to the acetabulum. The femoral
neurovascular structures are at risk if the retractor is placed anterior and inferior to the psoas
tendon.
The patient develops a foot drop 2 days after surgery. As a result, it can be assumed that
the nerve was not injured directly during the surgical procedure. Although MR imaging or a
CT scan may be indicated to identify an evolving hematoma, the immediate concern is to
minimize pressure on the sciatic nerve. Tension on the nerve can be decreased by flexing the
surgical knee and positioning the bed flat.
Question 8 of 200
An 80-year-old African American woman who lives in a large city is scheduled for total
hip arthroplasty to address primary osteoarthritis. Part of the presurgical protocol includes
nasal swab screening to assess for methicillin-resistant Staphylococcus aureus (MRSA)
colonization. Which demographic factor places this patient at highest risk for a positive result?
1- Gender
2- Age
3- Race
4- Environment
PREFERRED RESPONSE: 3- Race
DISCUSSION
AAOS 2016 Adult Reconstructive Surgery of the Hip and Knee
7
Demographic factors are associated with increased risk for MRSA colonization, so it is
important to identify vulnerable patients. Female gender and advanced age decrease risk for
colonization, while African American race increases this risk. Urban environments do not
influence MRSA colonization.
Figure 9
Question 9 of 200
Figure 9 is the clinical photograph of a 68-year-old woman 10 days after undergoing
primary total knee replacement. She is experiencing hemarthrosis, discoloration, and bruising
of the soft tissue about the knee; her history includes persistent serous drainage. This clinical
appearance likely is associated with
1- failure to use a tourniquet.
2- failure to use a drain.
3- use of low-molecular-weight heparin (LMWH).
4- use of regional anesthesia.
PREFERRED RESPONSE: 3- use of low-molecular-weight heparin (LMWH).
AAOS 2016 Adult Reconstructive Surgery of the Hip and Knee
8
DISCUSSION
Certain anticoagulants are associated with an increased risk for wound complications. Two
studies showed an increase in postsurgical bleeding and wound drainage following use of
LMWH. Other investigators have associated use of anticoagulants such as LMWH with an
increased incidence of persistent wound drainage and subsequent infection. Current evidence
does not support a significant difference in complication rates with and without the use of
wound drains or a tourniquet. Regional anesthesia has been associated with less blood loss
than general anesthesia and is not associated with a difference in wound complication
incidence.
Figure 10
Question 10 of 200
Figure 10 is the radiograph of a 44-year-old man with a long-standing history of severe
hip pain and a limp. Which clinical scenario most likely could occur when performing total
hip arthroplasty on this patient?
1- Placing the hip center too inferior
2- Overmedializing the acetabular component
3- Overlengthening the extremity
4- Intrasurgical acetabular fracture
AAOS 2016 Adult Reconstructive Surgery of the Hip and Knee
9
PREFERRED RESPONSE: 3- Overlengthening the extremity
DISCUSSION
The radiograph reveals hip dysplasia. Patients with hip dysplasia and severe limb
shortening are at high risk for sciatic nerve palsy from overlengthening. Overmedializing the
acetabular component is not the preferred response because overlateralization is more of a
concern if the cup is placed in the pseudoacetabulum instead of in the true acetabulum. Placing
the hip center too inferior is not the preferred response because the concern in this scenario is
placing the hip center too superior if the cup is placed in the pseudoacetabulum or if a large-
diameter cup is used. Acetabular fractures are possible because of osteoporotic bone at the true
hip center, but is less likely than overlengthening of the extremity.
Figure 11a Figure 11b
CLINICAL SITUATION FOR QUESTIONS 11 THROUGH 13
Figures 11a and 11b are the radiographs of a 35-year-old woman with end-stage
debilitating osteoarthritis of the right hip. She is contemplating total hip arthroplasty (THA).
She has a history of right hip dysplasia and underwent hip osteotomy as an adolescent. Over
the years, she has failed nonsurgical treatment including weight loss, activity modifications,
and intra-articular injections. Her infection workup reveals laboratory findings within defined
limits.
Question 11 of 200
Which bearing surface is contraindicated for this patient?
AAOS 2016 Adult Reconstructive Surgery of the Hip and Knee
10
1- Ceramic-on-ceramic
2- Ceramic-on-highly cross-linked polyethylene (HXPE)
3- Metal-on-HXPE
4- Metal-on-metal
PREFERRED RESPONSE: 4- Metal-on-metal
Question 12 of 200
The patient undergoes successful primary THA with a metal-on-metal bearing. At her 1-
year follow-up appointment, she reports no pain and is highly satisfied with the procedure.
However, 3 years after the index procedure, she reports atraumatic right hip pain that worsens
with activities. Radiographs reveal implants in good position with no sign of loosening or lysis.
An initial laboratory evaluation reveals a normal sedimentation rate and C-reactive protein
(CRP) level. The most appropriate next diagnostic step is
1- metal artifact reduction sequence (MARS) MR imaging only.
2- serum cobalt only.
3- serum cobalt and chromium levels.
4- serum cobalt and chromium levels and MARS MR imaging.
PREFERRED RESPONSE: 4- serum cobalt and chromium levels and MARS MR
imaging.
Question 13 of 200
A further workup reveals elevations in serum cobalt and chromium levels and fluid
collections surrounding the hip on MARS MR imaging. Revision THA is recommended. The
most common complication following revision of a failed metal-on-metal hip arthroplasty is
1- infection.
2- instability.
3- loosening.
4- periprosthetic fracture.
AAOS 2016 Adult Reconstructive Surgery of the Hip and Knee
11
PREFERRED RESPONSE: 2- instability.
DISCUSSION
THA has proven durable and reliable for pain relief and improving function for patients
with end-stage arthritis. Appropriate bearing selection is critical to minimize wear and hip
complications. A metal-on-metal articulation is associated with excellent wear rates in vitro.
With its capacity to offer a low wear rate with large femoral heads, it is an attractive bearing
choice for THA. However, local soft-tissue reactions, pseudotumors, and potential systemic
reactions including renal failure, cardiomyopathy, carcinogenesis, and potential teratogenesis
with potential transfer of metal ions across the placental barrier make metal-on-metal bearings
less desirable and relatively contraindicated for younger women of child-bearing age.
The workup of a painful metal-on-metal hip arthroplasty necessitates a systematic
approach. Several algorithms have been proposed. Routine laboratory studies including
sedimentation rate, CRP, and serum cobalt and chromium ion levels should be obtained for all
patients with pain. Advanced imaging including MARS MRI should be performed to evaluate
for the presence of fluid collections, pseudotumors, and abductor mechanism destruction.
Infection can coexist with metal-on-metal reactions, so, when indicated (if the CRP level is
elevated), a hip arthrocentesis should be obtained. However, in this setting, a manual cell count
and differential should be obtained because an automated cell counter may provide falsely
elevated cell counts.
The results of revision surgery for a failed metal-on-metal hip prosthesis can be variable.
The amount of local tissue destruction and the integrity of the hip abductor mechanism can
greatly influence outcomes. Instability is the most common complication following revision
of failed metal-on-metal hip replacements.
Question 14 of 200
A surgeon is preparing a medial gastrocnemius rotational flap to cover a medial proximal
tibia defect at the time of revision knee replacement surgery. To optimize coverage, the
surgeon must optimally mobilize which artery?
1- Profundus femoris
2- Middle genicular
3- Medial sural
4- Inferior medial genicular
PREFERRED RESPONSE: 3- Medial sural
AAOS 2016 Adult Reconstructive Surgery of the Hip and Knee
12
DISCUSSION
The medial sural arteries vascularize the gastrocnemius, plantaris, and soleus muscles
proximally. They arise from the popliteal artery. If not adequately mobilized, a gastroc soleus
flap can be devascularized.
RESPONSES FOR QUESTIONS 15 THROUGH 17
1- Semimembranosis release
2- Medial gastrocnemius release
3- Medial tibial plateau downsizing osteotomy
4- Iliotibial band pie crusting
5- Popliteus tendon release
6- Cruciate release of the capsule posterior lateral corner
Select the most appropriate release listed above to address each scenario described below.
Question 15 of 200
A 57-year-old man is taken to the operating room to address a severe varus right knee
deformity of approximately 17 degrees. After releasing the deep medial collateral ligament
(MCL) as part of the approach, the diseased side of the knee remains tight. Following
osteophyte removal, which soft-tissue release is now most appropriate to obtain a balanced
knee?
1- Semimembranosis release
2- Medial gastrocnemius release
3- Medial tibial plateau downsizing osteotomy
4- Iliotibial band pie crusting
5- Popliteus tendon release
6- Cruciate release of the capsule posterior lateral corner
PREFERRED RESPONSE: 1- Semimembranosis release
Question 16 of 200
Despite that release, the knee remains very tight medially. Cuts are validated for the
patient's axial alignment. For a patient with a large tibia, what is the best release to address a
large residual deformity?
AAOS 2016 Adult Reconstructive Surgery of the Hip and Knee
13
1- Semimembranosis release
2- Medial gastrocnemius release
3- Medial tibial plateau downsizing osteotomy
4- Iliotibial band pie crusting
5- Popliteus tendon release
6- Cruciate release of the capsule posterior lateral corner
PREFERRED RESPONSE: 3- Medial tibial plateau downsizing osteotomy
Question 17 of 200
The patient is planning on having his contralateral knee replaced as well. He has a mild
valgus deformity in his left knee with an overall windswept deformity. Which release is most
appropriate in this case if the knee remains tight in extension?
1- Semimembranosis release
2- Medial gastrocnemius release
3- Medial tibial plateau downsizing osteotomy
4- Iliotibial band pie crusting
5- Popliteus tendon release
6- Cruciate release of the capsule posterior lateral corner
PREFERRED RESPONSE: 4- Iliotibial band pie crusting
DISCUSSION
Balancing a total knee is important for longevity of the device and functional benefit. The
surgeon should be systematic in the release of a varus knee. The deep MCL is typically
released as part of the approach and osteophytes are then removed. The semimembranosus
tendon can then be released from the posterior medial aspect of the tibia. A downsizing
osteotomy can be considered for a large deformity if a patient has adequate tibial sizing. If a
patient has the smallest implant available prior to the osteotomy, an osteotomy will lead to
overhang of the implant and medial impingement on the MCL.
A valgus knee can be treated with pie crusting of the iliotibial band in mild extension
deformity. Surgeons should pause prior to taking down the popliteus and lateral collateral
AAOS 2016 Adult Reconstructive Surgery of the Hip and Knee
14
ligament because this can induce posterior rotatory subluxation of a primary knee, especially
in the case of a posterior collateral ligament-sacrificing total knee arthroplasty design.
Figure 18
Question 18 of 200
Figure 18 is the radiograph of a 52-year-old woman who has leg length inequality and
chronic activity-related buttock discomfort. This has been a life-long problem, but it is getting
worse and increasingly causing back pain. What is the best current technique for total hip
arthroplasty?
1- High hip center
2- Anatomic hip center with trochanteric osteotomy and progressive femoral shortening
3- Anatomic hip center with subtrochanteric shortening osteotomy
4- Iliofemoral lengthening followed by an anatomic hip center
PREFERRED RESPONSE: 3- Anatomic hip center with subtrochanteric shortening
osteotomy
DISCUSSION
AAOS 2016 Adult Reconstructive Surgery of the Hip and Knee
15
A high hip center is not recommended for Crowe IV hips because of the lack of acetabular
bone and altered hip biomechanics. An anatomic center is a better option but necessitates a
technique to address the tight soft-tissue envelope. A trochanteric osteotomy with progressive
femoral shortening has been described but can be prone to trochanter nonunion. Iliofemoral
lengthening prior to surgery has been described but may not be tolerated by all patients. A
shortening subtrochanteric osteotomy avoids trochanter nonunion and allows adjustment of
femoral anteversion. Fixation of the osteotomy can include a stem with distal rotational
control, plate fixation, a step vs. oblique cut, or strut grafts.
Question 19 of 200
The optimal method with which to diagnose component malrotation in total knee
arthroplasty (TKA) is
1- clinical assessment of foot position and patellar tracking.
2- radiographic skyline view of the patella.
3- CT scan with metal artifact suppression.
4- MR imaging with metal artifact reduction sequences.
PREFERRED RESPONSE: 3- CT scan with metal artifact suppression.
DISCUSSION
The epicondylar axis and tibial tubercle can be used as references on CT scans to
quantitatively measure rotational alignment of the femoral and tibial components. This
technique has been used to determine whether rotational malalignment is present and whether
revision of 1 or both components may be indicated. Although clinical assessment is useful,
malrotation can occur as a result of deformities unrelated to the arthroplasty. Similarly, an
isolated radiographic skyline view of the patella may indicate a problem with patellar
maltracking, but cannot quantitatively assess malrotation of the components. MR imaging
proves useful for evaluating painful TKA, but it is dependent on the center’s quality and has
not yet been shown to quantify component malrotation.
AAOS 2016 Adult Reconstructive Surgery of the Hip and Knee
16
Figure 20a Figure 20b
CLINICAL SITUATION FOR QUESTIONS 20 THROUGH 22
Figures 20a and 20b are the radiographs of an elderly woman who underwent total knee
arthroplasty (TKA) several years ago. She now states that something is not right; her knee
frequently swells and is diffusely painful, especially at the end of the day. She does not trust
her knee, especially while going up and down stairs or getting up from a chair.
Question 20 of 200
The most important diagnostic step is to
1- examine flexion and extension gap stability.
2- examine extensor power.
3- perform a neurological examination.
4- feel for the presence of foot pulses.
PREFERRED RESPONSE: 1- examine flexion and extension gap stability.
Question 21 of 200
AAOS 2016 Adult Reconstructive Surgery of the Hip and Knee
17
The initial diagnostic workup should include
1- CT scan of the knee.
2- erythrocyte sedimentation rate and C-reactive protein level.
3- electromyography and nerve conduction studies.
4- a technetium bone scan.
PREFERRED RESPONSE: 2- erythrocyte sedimentation rate and C-reactive protein level.
Question 22 of 200
Surgical treatment for this patient should include
1- excision arthroplasty with placement of an articulating antibiotic cement spacer.
2- excision arthroplasty with placement of a static antibiotic cement spacer.
3- revision to a more constrained prosthesis, with reestablishment of the flexion and
extension gap balance.
4- a thicker polyethylene insert.
PREFERRED RESPONSE: 3- revision to a more constrained prosthesis, with
reestablishment of the flexion and extension gap balance.
DISCUSSION
AAOS 2016 Adult Reconstructive Surgery of the Hip and Knee
18
Video 22 for reference
This patient has an unstable cruciate-retaining TKA. The tibial cut appears to be
substantial, necessitating a very thick polyethylene liner. In addition, the femoral component
may be slightly more proximally located. Tests for flexion-extension gap balancing would
indicate flexion instability. However, the possibility of infection remains, so screening blood
tests are appropriate. The surgical treatment for this unstable knee is revision TKA to a more
constrained implant, ensuring flexion-extension gap balancing. No evidence indicates that the
knee is infected. A thicker polyethylene insert will not adequately balance the knee.
Figure 23a Figure 23b
AAOS 2016 Adult Reconstructive Surgery of the Hip and Knee
19
Figure 23c Figure 23d
Figure 23e Figure 23f
AAOS 2016 Adult Reconstructive Surgery of the Hip and Knee
20
Figure 23g Figure 23h
Question 23 of 200
Figures 23a through 23h are the radiographs and MR images of a 32-year-old man with
worsening left knee pain. A 3-foot hip-to-ankle radiograph shows a 13-degree varus knee
deformity. The patient sustained a major left knee injury 5 years ago and a confirmed complete
anterior cruciate ligament (ACL) tear. He managed this injury nonsurgically with a functional
brace but experienced worsening pain. He was seen by an orthopaedic surgeon 18 months ago
and a medial meniscus tear was diagnosed; the tear was treated with an arthroscopic partial
medial meniscectomy. Since then, his knee has been giving way more often and he no longer
feels safe working on a pitched roof. The patient received 6 months of formal physical therapy
and was fitted for a new functional ACL brace, but he still has pain and instability symptoms.
He believes he has exhausted his nonsurgical options and would like to undergo surgery. What
is the most appropriate treatment at this time?
1- ACL reconstruction and subsequent proximal tibial osteotomy
2- ACL reconstruction alone
3- Distal femoral osteotomy with simultaneous ACL reconstruction
4- Proximal tibial osteotomy with subsequent ACL reconstruction
PREFERRED RESPONSE: 4- Proximal tibial osteotomy with subsequent ACL
reconstruction
AAOS 2016 Adult Reconstructive Surgery of the Hip and Knee
21
DISCUSSION
Proximal tibial osteotomy is the most appropriate intervention to correct varus
malalignment and to decrease stress on the ACL. In some cases, proximal tibial osteotomy
alone may address both pain and instability, but, if instability persists, particularly in the setting
in which instability can be dangerous, subsequent ACL reconstruction can further stabilize the
knee with less stress on the graft after correction of malalignment. Varus alignment places
increased stress on the native or reconstructed ACL. ACL reconstruction should only be
performed at the same time as or following proximal tibial osteotomy to correct alignment in
the setting of varus malalignment. It is not appropriate to perform ACL reconstruction prior to
proximal tibial osteotomy in this setting. Distal femoral osteotomy is not indicated to correct
varus malalignment. Varus alignment places increased stress on the native or reconstructed
ACL, and ACL reconstruction alone is not indicated for this patient.
Question 24 of 200
A surgeon performs a minimally invasive total knee arthroplasty through a quadriceps-
sparing approach using medial-to-lateral cutting jigs. When beginning therapy that afternoon,
the patient can passively but not actively extend her knee, although she has minimal knee pain.
All regional blocks have been discontinued. What is the most likely reason for this finding?
1- Quadriceps inhibition
2- Avulsion of the quadriceps tendon
3- Laceration of the patella tendon
4- Femoral nerve palsy
PREFERRED RESPONSE: 3- Laceration of the patella tendon
DISCUSSION
This patient lacks active knee extension. It is not attributable to the regional block because
that block is no longer acting. The most likely cause is laceration of the patella tendon, which
has been described during both large-incision surgery and minimally invasive surgery.
However, this is reported with increased frequency during minimally invasive surgery.
Quadriceps inhibition, avulsion of the quadriceps tendon, and femoral nerve palsy can cause
lack of active extension, but these problems are less likely because the patient has minimal
pain.
AAOS 2016 Adult Reconstructive Surgery of the Hip and Knee
22
Figure 25a Figure 25b
Question 25 of 200
Figures 25a and 25b are the radiographs of a 63-year-old man who had right total hip
arthroplasty (THA) 4 months ago. Progressive stiffness began 2 months after surgery, and he
now reports pain only after prolonged physical activity. His examination reveals normal gait
and painless range of motion with flexion of 70 degrees, extension of 0 degrees, internal
rotation of 20 degrees, external rotation of 20 degrees, abduction of 10 degrees, and adduction
of 10 degrees. His erythrocyte sedimentation rate (ESR) and C-reactive protein (CRP) levels
are within defined limits. Physical therapy has produced no benefit. What is the most
appropriate next step?
1- 25 mg of indomethacin 3 times daily for 6 weeks
2- 1 dose of irradiation at 800 Gy
3- Surgical excision of heterotypic ossification (HO)
4- Reevaluation in 6 months
PREFERRED RESPONSE: 4- Reevaluation in 6 months
AAOS 2016 Adult Reconstructive Surgery of the Hip and Knee
23
DISCUSSION
This patient presents with HO 4 months after undergoing THA. Symptomatic HO may
complicate nearly 7% of primary THA cases. Improvement in pain is expected within 6
months, and most patients will not need surgical treatment. Surgical excision may be warranted
for symptomatic patients after full maturation of the HO, usually 6 to 18 months after the
surgery. Patients can be followed with repeated serum alkaline phosphatase levels, which are
elevated initially and should return to normal upon maturation of HO. Alternatively, a bone
scan can show decreased activity once the HO has matured. Twenty-five milligrams of
indomethacin 3 times daily for 6 weeks or 1 dose of irradiation at 700 to 800 Gy is effective
in the prevention of HO, not for the treatment of established HO.
RESPONSES FOR QUESTIONS 26 THROUGH 29
1- Minimum inhibitory concentration (MIC)
2- Minimum bactericidal concentration (MBC)
3- Antiobiotic susceptibility
4- Antibiograms
5- Antimicrobial resistance
6- Spectrum of coverage
Match the description below with the appropriate response above.
Question 26 of 200
The minimum concentration of an antimicrobial agent to prevent growth of a
microorganism.
1- Minimum inhibitory concentration (MIC)
2- Minimum bactericidal concentration (MBC)
3- Antiobiotic susceptibility
4- Antibiograms
5- Antimicrobial resistance
6- Spectrum of coverage
PREFERRED RESPONSE: 1- Minimum inhibitory concentration (MIC)
AAOS 2016 Adult Reconstructive Surgery of the Hip and Knee
24
Question 27 of 200
Refers to the prevalence of microorganisms at a particular hospital or institution.
1- Minimum inhibitory concentration (MIC)
2- Minimum bactericidal concentration (MBC)
3- Antiobiotic susceptibility
4- Antibiograms
5- Antimicrobial resistance
6- Spectrum of coverage
PREFERRED RESPONSE: 4- Antibiograms
Question 28 of 200
Refers to the effectiveness of an antibiotic against various classes of microorganisms.
1- Minimum inhibitory concentration (MIC)
2- Minimum bactericidal concentration (MBC)
3- Antiobiotic susceptibility
4- Antibiograms
5- Antimicrobial resistance
6- Spectrum of coverage
PREFERRED RESPONSE: 6- Spectrum of coverage
Question 29 of 200
Refers to the effectiveness of various antibiotics against a particular microorganism.
1- Minimum inhibitory concentration (MIC)
2- Minimum bactericidal concentration (MBC)
3- Antiobiotic susceptibility
4- Antibiograms
5- Antimicrobial resistance
6- Spectrum of coverage
AAOS 2016 Adult Reconstructive Surgery of the Hip and Knee
25
PREFERRED RESPONSE: 3- Antiobiotic susceptibility
DISCUSSION
Knowledge of the basic nomenclature of antibiotic use is important to effectively treat
patients and communicate with colleagues. The effectiveness of an antimicrobial against an
infecting organism is measured by the MIC, which refers to the concentration needed to
prevent growth of a microorganism on culture medium, and MBC, which is the smallest
concentration of the antibiotic necessary to kill the microorganism in culture. Typically, an
antibiotic is considered bactericidal if the MBC is no more than 4 times the MIC. The spectrum
of antimicrobial coverage refers to an agent’s effectiveness against a range of bacteria. An
antibiogram refers to the tabulation of prevalence of different bacteria in a specific setting or
specific patient population. Antibiotic susceptibility and resistance refers to the bacteria’s
ability to be affected or unaffected by a given antibiotic.
Figure 30a
Question 30 of 200
Figure 30a is the anteroposterior radiograph of a 20-year-old woman with mild right groin
pain and intermittent “catching” in the hip region. What is the most appropriate next step?
AAOS 2016 Adult Reconstructive Surgery of the Hip and Knee
26
1- Arthroscopic evaluation and treatment of the hypertrophic labrum and a possible labral
tear
2- A hip injection to confirm an intra-articular source of the pain
3- Nonsurgical treatment and subsequent total hip arthroplasty (THA) when the patient is
sufficiently symptomatic
4- Periacetabular osteotomy
PREFERRED RESPONSE: 4- Periacetabular osteotomy
DISCUSSION
Because this patient is young, substantial bilateral acetabular dysplasia is present, and the
joint space is well preserved, periacetabular osteotomy is the treatment of choice (Figure 30b).
Arthroscopic evaluation and treatment is insufficient to address the mechanical deformity.
Although a hip injection can be diagnostically helpful, it would not alter the treatment plan in
this scenario. The patient’s young age would make observation and subsequent THA less
desirable. Femoral osteotomies also were performed to address rotational deformity.
Figure 31
AAOS 2016 Adult Reconstructive Surgery of the Hip and Knee
27
Question 31 of 200
Figure 31 is the abdominal radiograph of a 70-year-old woman who experiences nausea
and abdominal tightness 48 hours following left total knee arthroplasty performed under
general anesthesia. She received 24 hours of cefazolin antibiotic prophylaxis and a patient-
controlled analgesia narcotic pump for pain management. She has been receiving warfarin for
thromboembolic prophylaxis. Her severe abdominal distension and markedly decreased bowel
sounds are most likely secondary to
1- general anesthesia.
2- administration of antibiotics.
3- administration of warfarin.
4- administration of narcotics.
PREFERRED RESPONSE: 4- administration of narcotics.
DISCUSSION
The radiograph reveals severe intestinal dilatation, which has occurred as the result of
acute colonic pseudo-obstruction and is associated with excessive narcotic administration
following total joint arthroplasty. Anesthetic type, antibiotic administration, and warfarin have
not been associated with this obstruction. Electrolyte imbalances such as hypokalemia have
been associated with postsurgical acute colonic pseudo-obstruction.
Figure 32a Figure 32b Figure 32c
CLINICAL SITUATION FOR QUESTIONS 32 THROUGH 35
Figures 32a through 32c are the radiographs of a 30-year old man who is experiencing
right hip pain. He has no current medical problems, but, with a body mass index of 41, he is
morbidly obese. He was previously treated for leukemia with chemotherapy that included
AAOS 2016 Adult Reconstructive Surgery of the Hip and Knee
28
high-dose steroids. He undergoes total hip arthroplasty (THA) with a ceramic-on-ceramic
bearing.
Question 32 of 200
When counseling this patient regarding the long-term outcomes of surgery using a
ceramic-on-ceramic bearing, he should be informed that
1- there will be more overall bearing wear than metal on polyethylene.
2- there will be less long-term revision risk.
3- complication rates will be the same.
4- pain and function scores will be better.
PREFERRED RESPONSE: 2- there will be less long-term revision risk.
Question 33 of 200
Three years after undergoing THA with a ceramic-on-ceramic bearing, this patient returns
because of right hip pain and a grinding sensation. At revision, it is discovered that the ceramic
liner has fractured. The most likely cause for this complication is
1- horizontal cup position.
2- morbid obesity.
3- trunnion damage.
4- activity level.
PREFERRED RESPONSE: 2- morbid obesity.
Question 34 of 200
At revision, the ceramic femoral head is removed. Upon visual inspection, the most likely
finding on the head is
1- stripe wear.
2- no gross damage.
3- metal transfer.
4- fracture.
AAOS 2016 Adult Reconstructive Surgery of the Hip and Knee
29
PREFERRED RESPONSE: 1- stripe wear.
Question 35 of 200
At revision, the stem is retained and a new head with a polyethylene bearing is selected.
The best option for the head is
1- ceramic with a metal sleeve.
2- ceramic alone.
3- metal with a metal sleeve.
4- metal alone.
PREFERRED RESPONSE: 1- ceramic with a metal sleeve.
DISCUSSION
Ceramic-on-ceramic is a controversial bearing surface typically reserved for younger
patients such as this one. Some studies have suggested that the bearing is more expensive and
does not really prolong the service life of the implant, although a recent meta-analysis of high-
quality trials showed that there is a decreased revision rate with ceramic-on-ceramic, so its use
may be justified. Complications of intraoperative bearing fracture and squeaking are more
common than with conventional bearings, but pain and function scores are equivalent. Stripe
wear associated with a vertical cup and morbid obesity are related to an increased risk for liner
fracture. Concerns about head fractures with a new ceramic head and a damaged trunnion have
led investigators to conclude that using a harder bearing than the initial bearing surface with a
built-in titanium sleeve is probably the best solution when a stem is retained during revision
surgery.
AAOS 2016 Adult Reconstructive Surgery of the Hip and Knee
30
Figure 36a Figure 36b
Question 36 of 200
Figures 36a and 36b are the radiographs of a 79-year-old woman who has been
experiencing increasing tibial pain 10 years after undergoing revision total knee arthroplasty.
There is no evidence of infection. What is the most appropriate treatment?
1- Retain the components and implant a tibial strut allograft
2- Revise the tibial component with a metaphyseal cone and metaphyseal uncemented stem
3- Revise the tibial component with a metaphyseal cone and a press-fit diaphyseal-
engaging stem
4- Revise the tibial component with a long cemented diaphyseal-engaging stem
PREFERRED RESPONSE: 3- Revise the tibial component with a metaphyseal cone and
a press-fit diaphyseal-engaging stem
DISCUSSION
Stems are available for cemented and press-fit implantation. To be effective, press-fit
stems should engage the diaphysis (Figures 36c and 36d). They also assist in obtaining correct
limb alignment. Short metaphyseal-engaging stems are associated with failure rates between
AAOS 2016 Adult Reconstructive Surgery of the Hip and Knee
31
16% and 29%. Cemented stems may be shorter than press-fit stems because they do not have
to engage the diaphysis. Short, fully cemented stems offer the advantage of metaphyseal
fixation. Hybrid stem fixation makes use of the metaphysis for cement fixation with
metaphyseal cones or sleeves and diaphyseal-engaging press-fit stems.
RESPONSES FOR QUESTIONS 37 THROUGH 40
1- Lateral femoral cutaneous nerve
2- Lateral femoral circumflex artery
3- Superior gluteal nerve
4- Superior gluteal artery
5- Sciatic nerve
6- Femoral artery
7- Femoral vein
8- Femoral nerve
9- Saphenous branch of the femoral nerve
10- Profunda femoris artery
11- Inferior gluteal nerve
Match each description below with the anatomic structure listed above.
Question 37 of 200
When performing a modified direct lateral approach (modified Hardinge) to the hip, this
is the neurovascular structure at higher risk during anterior retraction of the gluteus
medius/vastus lateralis sleeve.
1- Lateral femoral cutaneous nerve
2- Lateral femoral circumflex artery
3- Superior gluteal nerve
4- Superior gluteal artery
5- Sciatic nerve
6- Femoral artery
7- Femoral vein
8- Femoral nerve
9- Saphenous branch of the femoral nerve
10- Profunda femoris artery
AAOS 2016 Adult Reconstructive Surgery of the Hip and Knee
32
11- Inferior gluteal nerve
PREFERRED RESPONSE: 3- Superior gluteal nerve
Question 38 of 200
The direct anterior approach to the hip places this neurovascular structure at highest risk
for injury in its superficial extent.
1- Lateral femoral cutaneous nerve
2- Lateral femoral circumflex artery
3- Superior gluteal nerve
4- Superior gluteal artery
5- Sciatic nerve
6- Femoral artery
7- Femoral vein
8- Femoral nerve
9- Saphenous branch of the femoral nerve
10- Profunda femoris artery
11- Inferior gluteal nerve
PREFERRED RESPONSE: 1- Lateral femoral cutaneous nerve
Question 39 of 200
This neurovascular structure limits distal extension of the direct anterior approach.
1- Lateral femoral cutaneous nerve
2- Lateral femoral circumflex artery
3- Superior gluteal nerve
4- Superior gluteal artery
5- Sciatic nerve
6- Femoral artery
7- Femoral vein
AAOS 2016 Adult Reconstructive Surgery of the Hip and Knee
33
8- Femoral nerve
9- Saphenous branch of the femoral nerve
10- Profunda femoris artery
11- Inferior gluteal nerve
PREFERRED RESPONSE: 8- Femoral nerve
Question 40 of 200
CT and cadaveric studies have shown that anteroinferior placement of the anterior
acetabular retractor results in less than 1 cm of distance between the retractor and which
structure?
1- Lateral femoral cutaneous nerve
2- Lateral femoral circumflex artery
3- Superior gluteal nerve
4- Superior gluteal artery
5- Sciatic nerve
6- Femoral artery
7- Femoral vein
8- Femoral nerve
9- Saphenous branch of the femoral nerve
10- Profunda femoris artery
11- Inferior gluteal nerve
PREFERRED RESPONSE: 8- Femoral nerve
DISCUSSION
Proximal dissection through the gluteus medius places the superior gluteal nerve at risk.
The main branch of the superior gluteal nerve has been measured within the muscle to a
distance of between 4.5 and 4.9 cm above the level of the superior acetabular rim, and safe
dissection is assured if the surgeon does not exceed 4 cm above the acetabulum or 5 cm above
the tip of the greater trochanter.
AAOS 2016 Adult Reconstructive Surgery of the Hip and Knee
34
The lateral femoral cutaneous nerve courses medial to the anterior superior iliac spine and
near the interval between the sartorious and tensor fascia muscle. Risk for injury can be
decreased by remaining in the fascial sheath of the tensor and minimizing excessive retraction
medially. The lateral femoral circumflex artery is ligated during this approach and is not at
risk.
Distal extension of the direct anterior approach beyond the intertrochanteric line poses risk
for injury to the lateral and medial division of the femoral nerve, which innervates the
anterolateral parts of the quadriceps muscle group. In addition, branches of the lateral
circumflex artery are routinely ligated in a standard approach.
A recent CT scan and cadaveric evaluation of acetabular retractor placement and its
proximity to adjacent neurovascular structures demonstrates that the anterior retractor is
furthest from the adjacent neurovascular bundle if placed more superiorly near the anterior
superior iliac spine. The more inferiorly it is placed, the higher the risk to the adjacent
structures, the most lateral of which is the femoral nerve.
Question 41 of 200
A 58-year-old man has a painful right hip 3 years after undergoing a large head metal-on-
metal total hip arthroplasty (THA) in which the components are well positioned. MR imaging
confirms a cystic mass around the hip and metal ion levels show a marked increase in cobalt
compared to chromium levels. The erythrocyte sedimentation rate (ESR) and C-reactive
protein (CRP) level are within defined limits. What is the most likely cause for his discomfort?
1- Chronic periprosthetic infection
2- Trochanteric bursitis
3- Pseudotumor related to corrosion at the head/neck taper junction
4- Tendonitis from iliopsoas tendon impingement
PREFERRED RESPONSE: 3- Pseudotumor related to corrosion at the head/neck taper
junction
DISCUSSION
This patient presents with a pseudotumor likely attributable to local tissue reaction
resulting from either articular metal wear debris and/or corrosion and fretting of the trunnion.
The trunnion is a more likely source of the problem for a number of reasons: good position of
metal articulation, increased trunnion corrosion and fretting associated with large-head THA,
and markedly increased cobalt levels compared to chromium levels. Infection is very unlikely
AAOS 2016 Adult Reconstructive Surgery of the Hip and Knee
35
in the setting of normal ESR and CRP findings. MR imaging findings are consistent with
pseudotumor and not iliopsoas tendonitis or trochanteric bursitis.
Figure 42a Figure 42b Figure 42c
Figure 42d Figure 42e
CLINICAL SITUATION FOR QUESTIONS 42 THROUGH 45
Figures 42a through 42e are the radiographs, MR image, and MR arthrogram of a 25-year-
old collegiate soccer player who has new-onset left groin pain. He played competitive soccer
from a young age and has either competed or practiced 5 to 6 times per week since the age of
10. He denies any specific hip injury that necessitated treatment, but his trainer contends that
he had a groin pull. He has groin pain with passive flexion and internal rotation of his left hip,
and his hip has less internal rotation than his asymptomatic right hip. He is otherwise healthy.
AAOS 2016 Adult Reconstructive Surgery of the Hip and Knee
36
Question 42 of 200
Approximately which percentage of asymptomatic athletes have CAM deformities of the
hip?
1- 5%
2- 10%
3- 25%
4- At least 50%
PREFERRED RESPONSE: 4- At least 50%
Question 43 of 200
What is the primary cause of a CAM deformity?
1- A genetic problem
2- Repetitive activities involving an open proximal femoral physis
3- Early closure of the proximal femoral physis
4- Hip dysplasia
PREFERRED RESPONSE: 2- Repetitive activities involving an open proximal femoral
physis
Question 44 of 200
When counseling patients who have a CAM deformity, the orthopaedic surgeon should
note that
1- osteoarthritis of the hip is likely to occur later in life.
2- correction prevents later development of osteoarthritis.
3- most acetabular tears are symptomatic, and surgical treatment will be necessary.
4- this is an inherited deformity.
AAOS 2016 Adult Reconstructive Surgery of the Hip and Knee
37
PREFERRED RESPONSE: 1- osteoarthritis of the hip is likely to occur later in life.
Question 45 of 200
Which factor is a contraindication to surgical treatment of a symptomatic CAM deformity?
1- Degenerative tear of the anterosuperior acetabular labrum
2- Superior hip joint space of 2 mm or less on radiographs
3- Ipsilateral knee instability
4- Lumbar spondylolisthesis
PREFERRED RESPONSE: 2- Superior hip joint space of 2 mm or less on radiographs
DISCUSSION
Multiple studies have confirmed that CAM or pincer anatomy is commonly present in
asymptomatic hips. According to a large systematic review, CAM deformities are present in
approximately one-third of asymptomatic hips in young adults, and the proportion was higher
than 50% in the subgroup of athletes.
Ganz and associates proposed that femoral acetabular impingement is the root cause of
osteoarthritis in the majority of nontraumatic, nondysplastic hips, and functional improvement
with surgical correction of the deformity has been demonstrated. Despite the link between
CAM deformity and hip osteoarthritis, a corresponding link between correction of the
deformity and prevention of osteoarthritis has never been proven.
Results of CAM deformity correction, typically including repair of the degenerative labral
tear, are much poorer when there is significant joint space loss. A typical joint space cutoff of
2 mm or less is used to recommend against hip preservation surgery.
Question 46 of 200
Patellofemoral arthroplasty is contraindicated in the presence of
1- moderate patellar tilt.
2- trochlear dysplasia.
3- inflammatory arthritis.
4- severe crepitus.
AAOS 2016 Adult Reconstructive Surgery of the Hip and Knee
38
PREFERRED RESPONSE: 3- inflammatory arthritis.
DISCUSSION
Any inflammatory disease, including crystalline arthropathy, is a contraindication for
patellofemoral arthroplasty. Patellar tilt and dysplasia frequently occur in the setting of
patellofemoral arthritis and do not constitute a contraindication to this procedure. Severe
crepitus is common and frequently addressed with patellofemoral arthroplasty.
Figure 47
CLINICAL SITUATION FOR QUESTIONS 47 THROUGH 50
A 70-year-old woman has a 3-year history of gradually increasing diffuse and global right
knee pain. Her main issues are difficulty with stairs, stiffness with prolonged sitting, and
swelling. She has taken nonsteroidal anti-inflammatory medications and has received intra-
articular steroid injections, all with decreasing efficacy. Her right knee examination reveals
range of motion of 15 to 80 degrees with a fixed deformity to varus and valgus stress. Her
symptoms are no longer manageable nonsurgically. Radiographs reveal a 30-degree
mechanical axis deformity.
Question 47 of 200
The deformity shown in Figure 47 is predominantly associated with
AAOS 2016 Adult Reconstructive Surgery of the Hip and Knee
39
1- a hypoplastic lateral femoral condyle.
2- a contracted medial collateral ligament.
3- an excessive proximal tibial slope.
4- trochlear dysplasia.
PREFERRED RESPONSE: 1- a hypoplastic lateral femoral condyle.
Question 48 of 200
When using the measured resection technique during total knee arthroplasty (TKA), the
best way to avoid femoral malrotation is to reference the
1- anteroposterior axis.
2- tibial intramedullary axis.
3- posterior condylar axis.
4- femoral intramedullary axis.
PREFERRED RESPONSE: 1- anteroposterior axis.
Question 49 of 200
When balancing gaps in the coronal plane, which structure preferentially impacts the
flexion space more than the extension space?
1- Iliotibial band
2- Popliteus tendon
3- Lateral collateral ligament
4- Lateral head of the gastrocnemius
PREFERRED RESPONSE: 2- Popliteus tendon
Question 50 of 200
For this patient, which TKA design is most appropriate?
AAOS 2016 Adult Reconstructive Surgery of the Hip and Knee
40
1- Bicruciate-retaining TKA
2- Unicompartmental arthroplasty
3- Hinged TKA
4- Posterior substitution TKA
PREFERRED RESPONSE: 4- Posterior substitution TKA
DISCUSSION
TKA in the setting of valgus deformities poses different challenges than those encountered
when varus deformities are present. Most valgus alignment is attributable to a deformity of the
distal femur rather than the proximal tibia, as seen in varus knees. One of the major anatomical
differences is a hypoplastic lateral femoral condyle which, when used as a rotational reference
point, can lead to internal rotation of the femoral component if not recognized. This
malrotation will in turn lead to patellofemoral maltracking or instability, which is a common
complication associated with primary TKA. The deformity is too severe to consider a
bicruciate-retaining TKA or unicompartmental arthroplasty and does not necessitate a hinged
TKA.
Question 51 of 200
When performing a cruciate-retaining total knee arthroplasty, trial components are
inserted. The knee comes to full extension but is tight in flexion. The surgeon should consider
1- flexing the femoral component.
2- releasing the posterior cruciate ligament.
3- downsizing the tibial insert thickness.
4- resecting more distal femur.
PREFERRED RESPONSE: 2- releasing the posterior cruciate ligament.
DISCUSSION
In this scenario, the flexion gap needs to be increased. Increase in flexion gap can be
accomplished by downsizing the femoral component and increasing posterior tibial slope. In
posterior cruciate-retaining TKA procedures, recession or release of the posterior cruciate
ligament can loosen the flexion gap, allowing for an increase in flexion. Flexing the femoral
AAOS 2016 Adult Reconstructive Surgery of the Hip and Knee
41
component tightens the flexion gap, and downsizing the tibial insert thickness decreases
flexion and extension gaps, while resection of the distal femur only increases the extension
gap.
Figure 52
CLINICAL SITUATION FOR QUESTIONS 52 THROUGH 55
Figure 52 is the standing anteroposterior radiograph of a 55-year-old man who has a 5-
year history of daily left knee pain with weight-bearing activities. He denies night pain or
symptoms of instability. Upon examination his range of motion is 0 to 140 degrees. He has a
mild, fully correctable varus deformity and a negative Lachman test result. He has failed
nonsurgical treatment.
Question 52 of 200
Based on a knee examination and radiographic findings, what is the most likely finding at
the time of surgery?
1- Anteromedial osteoarthritis
2- An intact posterior cruciate ligament (PCL) and incompetent anterior cruciate ligament
(ACL)
3- Incompetence of both the ACL and PCL
4- Posteromedial osteoarthritis
AAOS 2016 Adult Reconstructive Surgery of the Hip and Knee
42
PREFERRED RESPONSE: 1- Anteromedial osteoarthritis
Question 53 of 200
Unicompartmental knee arthroplasty (UKA) is discussed with the patient. The most
appropriate next radiographic examination should be
1- MR imaging of the left knee to evaluate the lateral compartment.
2- a CT arthrogram to evaluate the status of the medial and lateral meniscus.
3- a stress radiograph to evaluate correction of the varus deformity.
4- a sunrise view to determine the status of the patellofemoral joint.
PREFERRED RESPONSE: 3- a stress radiograph to evaluate correction of the varus
deformity.
Question 54 of 200
What is the UKA survivorship for a 55-year-old patient compared to survivorship for total
knee arthroplasty?
1- Equal at 10 years
2- Lower at 10 years
3- Higher at 10 years
4- Not known when using a mobile-bearing UKA
PREFERRED RESPONSE: 2- Lower at 10 years
Question 55 of 200
The patient undergoes a mobile-bearing UKA. When compared to a fixed-bearing metal-
backed unicompartmental arthroplasty, this procedure is associated with a
1- higher risk for failure attributable to wear.
2- higher risk for failure attributable to bearing spinout.
3- lower risk for loosening.
AAOS 2016 Adult Reconstructive Surgery of the Hip and Knee
43
4- lower risk for arthritic progression of the lateral compartment.
PREFERRED RESPONSE: 2- higher risk for failure attributable to bearing spinout.
DISCUSSION
A patient with medial compartment arthritis and a correctable varus deformity with no
clinical or examination findings of knee instability most likely has an intact ACL. The pattern
of medial compartment osteoarthritis most commonly associated with an intact ACL is that of
anteromedial osteoarthritis. An incompetent ACL is commonly associated with a fixed varus
deformity and radiographic signs of posteromedial wear. An incompetent ACL is a relative
contraindication to a mobile-bearing UKA. When evaluating patients for a mobile-bearing
UKA, a stress radiograph will allow the orthopaedic surgeon to determine the correction of
the varus deformity and assess the lateral compartment. Inability to fully correct the deformity
or narrowing of the lateral compartment with valgus stress should influence the surgeon
against UKA. Joint registries across the world have shown decreased survivorship associated
with TKA and UKA in men compared to other age groups, but survivorship is lower for UKA
than TKA. No studies to date have shown differences in survivorship between fixed- or
mobile-bearing UKAs. The complication that is unique to mobile-bearing UKA is bearing
spinout, and this occurs in fewer than 1% of mobile-bearing UKA procedures. In vivo and in
vitro polyethylene wear in mobile-bearing UKA is low. Arthritis progression may be faster for
mobile-bearing UKAs than fixed-bearing UKAs.
AAOS 2016 Adult Reconstructive Surgery of the Hip and Knee
44
Figure 56a Figure 56b Figure 56c
Question 56 of 200
Figures 56a through 56c are the radiographs of a 65-year-old man with a 6-week history
of severe left hip pain. He had a left total hip arthroplasty 19 years ago and a femoral revision
10 years ago. His erythrocyte sedimentation rate (ESR) and C-reactive protein (CRP) level are
within defined limits. What is the most appropriate next step?
1- A trial of limited weight-bearing activity and physical therapy
2- A triple-phase bone scan to evaluate for loosening
3- Acetabular component revision
4- Femoral component revision
PREFERRED RESPONSE: 4- Femoral component revision
DISCUSSION
AAOS 2016 Adult Reconstructive Surgery of the Hip and Knee
45
The radiographs reveal a fracture of the extensively porous coated stem. This entity,
although rare, is associated with higher risk for occurrence when thin stems are implanted in
patients with thick cortices and there is a lack of proximal stable support for the prosthesis.
Nonsurgical care likely would not help this patient. A triple-phase bone scan would not add
any information that would change the treatment plan.
Question 57 of 200
At the time of revision knee arthroplasty, a surgeon performs a rectus snip to gain exposure
to the knee. When compared to a standard parapatellar approach, what is the expected
outcome?
1- Improvement in range of motion
2- Reduction in range of motion
3- Increase in extensor mechanism lag
4- No differences in motion and strength
PREFERRED RESPONSE: 4- No differences in motion and strength
DISCUSSION
Rectus snip during total knee arthroplasty has no effect on motion or strength at long-term
follow-up. It has not been associated with extensor mechanism lag.
AAOS 2016 Adult Reconstructive Surgery of the Hip and Knee
46
Figure 58 Figure 59a Figure 59b
AAOS 2016 Adult Reconstructive Surgery of the Hip and Knee
47
Figure 60a Figure 60b Figure 60c
Figure 61 Figure 62a Figure 62b
AAOS 2016 Adult Reconstructive Surgery of the Hip and Knee
48
RESPONSES FOR QUESTIONS 58 THROUGH 62
1- Nerve palsy
2- Skin necrosis
3- Flexion instability
4- Patellar instability
5- Anterior knee pain
6- Malalignment
Total knee arthroplasty (TKA) is performed to address each condition shown in Figures
58 through 62b. Which complication is most commonly associated with each image?
Question 58 of 200
Figure 58
1- Nerve palsy
2- Skin necrosis
3- Flexion instability
4- Patellar instability
5- Anterior knee pain
6- Malalignment
PREFERRED RESPONSE: 1- Nerve palsy
Question 59 of 200
Figure 59a and Figure 59b
1- Nerve palsy
2- Skin necrosis
3- Flexion instability
4- Patellar instability
5- Anterior knee pain
6- Malalignment
AAOS 2016 Adult Reconstructive Surgery of the Hip and Knee
49
PREFERRED RESPONSE: 4- Patellar instability
Question 60 of 200
Figure 60a through Figure 60c
1- Nerve palsy
2- Skin necrosis
3- Flexion instability
4- Patellar instability
5- Anterior knee pain
6- Malalignment
PREFERRED RESPONSE: 2- Skin necrosis
Question 61 of 200
Figure 61
1- Nerve palsy
2- Skin necrosis
3- Flexion instability
4- Patellar instability
5- Anterior knee pain
6- Malalignment
PREFERRED RESPONSE: 5- Anterior knee pain
Question 62 of 200
Figure 62a and Figure 62b
1- Nerve palsy
2- Skin necrosis
3- Flexion instability
4- Patellar instability
AAOS 2016 Adult Reconstructive Surgery of the Hip and Knee
50
5- Anterior knee pain
6- Malalignment
PREFERRED RESPONSE: 6- Malalignment
DISCUSSION
Figure 58 reveals a posttraumatic valgus deformity. Correction of valgus with lateral soft-
tissue release places tension on the peroneal nerve, resulting in an increased risk for nerve
palsy. Figures 59a (lateral view) and 59b (Merchant view) illustrate juvenile rheumatoid
arthritis with tibiofibular fusion and lateral patellar dislocation. Chronic patellar dislocation is
associated with contracture of the lateral retinacular soft tissues and increased risk for patellar
subluxation or dislocation after TKA. Extensor mechanism realignment, possibly including
tibial tubercle osteotomy and/or proximal soft-tissue realignment, may be required during
TKA to centralize the extensor mechanism. Figures 60a (anteroposterior [AP] view) and 60b
(lateral view) reveal a fused knee in full extension.
TKA after fusion is associated with multiple complications including skin necrosis,
infection, and instability. The skin is contracted because of limited knee motion and has
multiple scars (Figure 60c). Mobilization of the skin during and after knee arthroplasty can
place excess tension on the soft tissues, resulting in skin necrosis and infection. Treatment
consisting of prompt debridement and soft-tissue coverage, usually with medial gastrocnemius
muscle transposition, is required. Figure 61 shows a knee with prior tibial tubercle fixation
and marked patella infera. Shortening of the patellar ligament is associated with restricted knee
motion. This may necessitate more extensile exposure using tibial tubercle osteotomy or rectus
snip during TKA to obtain adequate surgical exposure. The inferior position of the patella can
cause impingement between the patellar component and tibial insert, resulting in anterior knee
pain. Restoring a more normal position of the patella may necessitate distal positioning of the
femoral component as well as tibial tubercle osteotomy with proximal recession of the
osteotomized tibial tubercle. Figures 62a (AP view of the distal femur) and 62b (AP view of
the proximal femur) show a posttraumatic deformity with a large retained intramedullary rod.
There is a varus distal femoral deformity that is not severe enough to necessitate extra-articular
corrective osteotomy. However, intramedullary hardware precludes use of conventional
intramedullary instrumentation, so computer navigation or patient-specific cutting guides will
be necessary to orient the bone cuts and avoid implant malalignment.
Question 63 of 200
Injury to the popliteal artery during total knee arthroplasty (TKA) is most likely when
placing a sharp retractor
1- directly posterior to the posterior cruciate ligament (PCL).
AAOS 2016 Adult Reconstructive Surgery of the Hip and Knee
51
2- posteromedial to the PCL.
3- posterolateral to the PCL.
4- in the posteromedial corner of the knee.
PREFERRED RESPONSE: 3- posterolateral to the PCL.
DISCUSSION
Vascular complications during TKA are rare but do occur. Traditionally, it was taught that
the popliteal artery was situated posterior to the PCL; however, more recent anatomic
dissections have demonstrated that this artery is usually posterolateral to the PCL.
Question 64 of 200
A 30-year-old patient is indicated for distal femoral osteotomy. This procedure results in
survivorship with
1- a functional result for at least 20 years.
2- a functional result that deteriorates within the first 10 years.
3- an eventual conversion to a constrained knee arthroplasty.
4- an eventual need for arthrodesis.
PREFERRED RESPONSE: 2- a functional result that deteriorates within the first 10 years.
DISCUSSION
Distal femoral varus osteotomy (DFVO) is intended for patients younger than age 50, more
active patients with isolated lateral compartment arthritis and valgus malalignment. Although
the knee functional score improves at 1-year follow-up, the function scores significantly
deteriorate at 10-year follow-up. At 15-year follow-up, the knee function further declines,
resulting in an overall failure rate of 48.5%. DFVO provides longer lasting benefit in patients
with better presurgical knee function.
Total knee arthroplasty following DFVO provides improved function and successful
outcomes. Standard posterior stabilized components provide satisfactory stability after
appropriate ligament balancing without the need for stemmed or highly constrained implants
for most patients.
AAOS 2016 Adult Reconstructive Surgery of the Hip and Knee
52
Figure 65a Figure 65b
CLINICAL SITUATION FOR QUESTIONS 65 THROUGH 67
Figures 65a and 65b are the radiographs of an 80-year-old woman with long-standing knee
pain who has failed nonsurgical treatment that includes a structured physical therapy program,
nonsteroidal anti-inflammatory medications, and intra-articular steroid injections. Her
radiographs reveal significant degenerative changes. She has experienced some cognitive
decline during the last several years and poorly tolerates pain medications, but she remains
very active.
Question 65 of 200
Cognitive function following total knee arthroplasty (TKA) among elderly patients who
receive either neuraxial or general anesthesia is
1- less affected with neuraxial anesthesia.
2- less affected with general anesthesia.
3- unchanged with either anesthesia type.
4- generally worsened with both neuraxial and general anesthesia.
AAOS 2016 Adult Reconstructive Surgery of the Hip and Knee
53
PREFERRED RESPONSE: 1- less affected with neuraxial anesthesia.
Question 66 of 200
To reduce the patient’s use of narcotics after surgery, which anesthetic regimen would be
most appropriate?
1- Spinal anesthesia with a peripheral nerve block
2- Spinal anesthesia without a peripheral nerve block
3- General anesthesia without a peripheral nerve block
4- General anesthesia with a long-acting mu opioid agonist
PREFERRED RESPONSE: 1- Spinal anesthesia with a peripheral nerve block
Question 67 of 200
Which treatment is associated with decreased complications related to femoral nerve
blocks for TKA?
1- Knee immobilizer
2- Compression stockings
3- Tranexamic acid
4- Celecoxib
PREFERRED RESPONSE: 1- Knee immobilizer
DISCUSSION
TKA among elderly patients can be problematic, considering their potential for complex
comorbidities including diminished cognitive function. As patients age, their tolerance for
certain medications diminishes. Regional anesthesia is an important adjunct to a multimodal
pain program, which can reduce narcotic pain medication use and improve cognitive function
through less reliance on systemic medications. With regional pain management such as
femoral nerve blocks comes potential for an increase in complications such as falls. Femoral
nerve blocks improve pain but also have a large impact on quadriceps and motor function,
which places patients at higher risk for falls.
AAOS 2016 Adult Reconstructive Surgery of the Hip and Knee
54
Question 68 of 200
A 47-year-old obese man with a body mass index of 42 comes in with left knee pain 1 year
after undergoing an uncomplicated left medial unicompartmental knee arthroplasty (UKA).
Radiographs show a loose tibial component in varus. What is the most appropriate next step
to treat this failed construct?
1- Aspiration of joint fluid to obtain a cell count
2- Revision of the UKA using primary total knee arthroplasty (TKA) components
3- Revision of the UKA using a revision TKA with augments
4- Obtain erythrocyte sedimentation rate (ESR) and C-reactive protein (CRP) levels
PREFERRED RESPONSE: 4- Obtain erythrocyte sedimentation rate (ESR) and C-
reactive protein (CRP) levels
DISCUSSION
This patient likely is experiencing failure of his UKA secondary to poor patient selection.
This young, heavy man likely loosened his component secondary to the ongoing varus
alignment of the knee and his elevated weight. Despite this likely scenario, the next step is to
determine if an infection is the cause of his pain. Prior to obtaining an aspiration, the surgeon
can order an ESR and CRP to determine if aspiration is warranted. If laboratory studies are
unremarkable, the surgeon likely can forgo the aspiration and proceed to a revision TKA with
possible augments on standby.
AAOS 2016 Adult Reconstructive Surgery of the Hip and Knee
55
Figure 69a Figure 69b
Figure 69c Figure 70a
AAOS 2016 Adult Reconstructive Surgery of the Hip and Knee
56
Figure 70b Figure 70c
Figure 70d
CLINICAL SITUATION FOR QUESTIONS 69 THROUGH 72
Figures 69a through 69c are the radiograph and MR images of a 37-year-old woman who
has had a 2-month history of left hip pain.
Question 69 of 200
Which single symptom and examination finding combination is most likely in this
scenario?
1- Sitting pain with flexion abduction and external rotation of the hip
AAOS 2016 Adult Reconstructive Surgery of the Hip and Knee
57
2- Groin pain and pain with internal rotation and adduction while supine with the hip and
knee flexed 90 degrees
3- Clicking and abductor lurch
4- Buttock pain and pain with hip extension, adduction, and external rotation while prone
PREFERRED RESPONSE: 2- Groin pain and pain with internal rotation and adduction
while supine with the hip and knee flexed 90 degrees
Question 70 of 200
Measurements have been taken on the radiographs shown in Figures 70a through 70d.
Which measurement is abnormal?
1- Head center position
2- Tonnis angle
3- Lateral center edge angle
4- Acetabular inclination
PREFERRED RESPONSE: 3- Lateral center edge angle
Question 71 of 200
Based on the patient’s continued pain, her imitations, the previously noted radiographic
findings, and failure of physical therapy and nonsteroidal anti-inflammatory treatment, what
is the best surgical option?
1- Arthroscopic labral repair
2- Arthroscopy with pincer resection
3- Periacetabular osteotomy (PAO)
4- Hip arthroscopy and PAO
PREFERRED RESPONSE: 1- Arthroscopic labral repair
Question 72 of 200
AAOS 2016 Adult Reconstructive Surgery of the Hip and Knee
58
Which presurgical condition is most commonly associated with a poor outcome after a hip
joint salvage procedure?
1- Age older than 40
2- Body mass index higher than 30
3- Tonnis grade of 2 or higher
4- Outerbridge grade of III or IV
PREFERRED RESPONSE: 3- Tonnis grade of 2 or higher
DISCUSSION
MR imaging reveals an anterior labral tear, and the radiograph shows minimal arthritis
with possible dysplasia. The most common pain location among patients with a labral tear is
the groin, and the most common physical finding is a positive impingement test result. Pain
with sitting, clicking, and buttock pain are frequently described by patients with a labral tear,
but these symptoms are less common than groin pain. A positive posterior impingement test
finding is more common in patients with a posterior labral tear.
The lateral center edge angle is pictured in Figure 70a. This angle is considered abnormal
when it is less than 25 degrees, which may indicate inadequate head coverage. The Tonnis
angle and acetabular inclination are different terms for the same angle as seen in Figure 70b.
The normal value is between 0 and 10 degrees. The head center position is the distance from
the medial aspect of the femoral head to the ilioischial line, as measured in Figure 70c. The
head is considered lateralized if the measurement exceeds 10 mm. Figure 70d marks the outline
of the anterior and posterior wall, confirming the absence of a cross-over sign. A cross-over
sign is present when the 2 lines cross below the lateral aspect of the acetabulum, and it indicates
acetabular retroversion.
An arthroscopy to treat the labral tear is probably the best option for this patient. A PAO
with prior hip arthroscopy might be considered based on the mildly increased anterior center
edge angle; the Tonnis angle finding is normal, the femoral head is not lateralized, and the
patient is approaching age 40. A PAO without addressing the anterior labral tear could lead to
failure and is not the best choice. The patient does not have an acetabular pincer lesion and
would not benefit from an acetabular edge resection.
Although age beyond 40 years and body mass index higher than 30 can adversely affect
clinical outcomes after joint preservation procedures (PAO, hip arthroscopy, and femoral
acetabular impingement surgery), the presence of hip arthritis on presurgical radiographs is
the most commonly mentioned cause of failed hip joint preservation surgery. Tonnis grade is
a radiographic measure of hip arthritis. A higher Outerbridge score is associated with more
frequent poor outcomes after hip arthroscopy; however, the Outerbridge cartilage score is
AAOS 2016 Adult Reconstructive Surgery of the Hip and Knee
59
determined by direct visualization at the time of surgery. An Outerbridge score cannot be
determined presurgically.
Question 73 of 200
Clinical studies on the use of topical and intravenous (IV) forms of tranexamic acid (TXA)
administration demonstrate which results?
1- IV administration of TXA is substantially more efficacious in minimizing blood loss
than topical administration.
2- IV administration of TXA places high-risk patients such as those with coronary stents
at an unacceptable risk for a cardiac event during the perioperative period.
3- IV administration of TXA decreases intrasurgical blood loss but has not been shown to
decrease postsurgical transfusion rates.
4- Both IV and topical administration of TXA decrease intrasurgical blood loss and
postsurgical transfusion rates.
PREFERRED RESPONSE: 4- Both IV and topical administration of TXA decrease
intrasurgical blood loss and postsurgical transfusion rates.
DISCUSSION
Numerous studies have demonstrated efficacy of both IV and topical administration of
TXA for decreasing blood loss and transfusion rates. Several studies have shown no significant
difference between TXA IV and topical administration in decreasing blood loss or lowering
transfusion rates. Inconclusive evidence shows that IV administration of TXA places
individuals at higher risk for a thromboembolic event. Both IV and topical TXA are equally
effective in decreasing blood loss and minimizing transfusion rates.
AAOS 2016 Adult Reconstructive Surgery of the Hip and Knee
60
Figure 74a Figure 74b Figure 74c
Question 74 of 200
Figures 74a through 74c are the postsurgical radiographs of a 74-year-old man who has
stiffness in his right knee 8 weeks after undergoing elective right total knee arthroplasty
(TKA). The surgery was performed for primary varus osteoarthritis and was uncomplicated.
His range of motion is 5 to 80 degrees. What is the most appropriate next treatment step?
1- Manipulation under anesthesia (MUA)
2- Arthroscopic lysis of adhesions
3- Open arthrolysis of adhesions
4- Revision TKA
PREFERRED RESPONSE: 1- Manipulation under anesthesia (MUA)
DISCUSSION
Stiffness following TKA is a disabling complication. One option involves MUA, a
valuable technique with which to increase range of motion after TKA for patients with stiff
knees. A systematic review of the literature was performed to identify studies that reported the
clinical outcomes and measured range of motion for patients undergoing MUA. Fourteen
studies (913 patients) reported range of motion results following MUA at up to 10-year follow-
up. The review demonstrated that MUA for a stiff primary TKA is an efficacious procedure to
restore range of motion, and it carries a low complication rate. Early gains in motion were
AAOS 2016 Adult Reconstructive Surgery of the Hip and Knee
61
reported to be maintained in the long term. A second review systematically evaluated the
outcomes of 4 treatments for arthrofibrosis that develops subsequent to TKA (MUA,
arthroscopic debridement, open surgical release, and revision TKA). This study showed that
there were no significant differences in the Knee Society Score of the 4 treatment modalities.
Although open surgical release resulted in the greatest increase in range of motion, there were
methodological study limitations because the majority of the papers were case series, which
decreased the quality of the evidence. Response 1 is correct because the TKA appears
appropriately sized and is well aligned. Responses 2 and 3 are incorrect, considering the early
time frame (8 weeks) from initial surgery. Although the patella is unresurfaced in the
radiographs, there is no indication for revision TKA (even secondary patellar resurfacing) at
this early juncture.
Question 75 of 200
During total hip arthroplasty, which characteristic of irradiated (10 Mrad) and
subsequently melted highly cross-linked polyethylene should provide a more wear-resistant
construct than traditional gamma-irradiated (2.5-4 Mrad)-in-air polyethylene mated with the
same head?
1- Resistance to adhesive wear
2- Resistance to abrasive wear
3- Resistance to fatigue wear
4- Resistance to creep
PREFERRED RESPONSE: 1- Resistance to adhesive wear
DISCUSSION
Highly cross-linked polyethylene makes material resistant to adhesive wear. Abrasive
wear from third bodies does not decrease wear. The fatigue strength of this material is inferior
to traditional polyethylene, and its resistance to creep is the same, if not lower, than that of
traditional polyethylene.
AAOS 2016 Adult Reconstructive Surgery of the Hip and Knee
62
Figure 76a Figure 76b
Figure 77a Figure 77b
AAOS 2016 Adult Reconstructive Surgery of the Hip and Knee
63
Figure 78a Figure 78b
Figure 79a Figure 79b
AAOS 2016 Adult Reconstructive Surgery of the Hip and Knee
64
Figure 80a Figure 80b
RESPONSES FOR QUESTIONS 76 THROUGH 80
1- Cemented full revision
2- Cemented femoral revision
3- Cemented acetabular revision
4- Cementless full revision
5- Cementless femoral revision without an extended trochanteric osteotomy
6- Cementless femoral revision with an extended trochanteric osteotomy
7- Cementless acetabular revision
8- Isolated liner/head exchange
9- Isolated liner/head exchange with cementing of the liner
Select the most appropriate treatment for each clinical scenario.
Question 76 of 200
A 72-year-old man with a history of prior revision for a loose femoral component (Figures
76a and 76b)
1- Cemented full revision
2- Cemented femoral revision
AAOS 2016 Adult Reconstructive Surgery of the Hip and Knee
65
3- Cemented acetabular revision
4- Cementless full revision
5- Cementless femoral revision without an extended trochanteric osteotomy
6- Cementless femoral revision with an extended trochanteric osteotomy
7- Cementless acetabular revision
8- Isolated liner/head exchange
9- Isolated liner/head exchange with cementing of the liner
PREFERRED RESPONSE: 6- Cementless femoral revision with an extended trochanteric
osteotomy
Question 77 of 200
A 65-year-old woman with intraoperative findings of well-fixed components and a
deficient liner locking mechanism (Figures 77a and 77b)
1- Cemented full revision
2- Cemented femoral revision
3- Cemented acetabular revision
4- Cementless full revision
5- Cementless femoral revision without an extended trochanteric osteotomy
6- Cementless femoral revision with an extended trochanteric osteotomy
7- Cementless acetabular revision
8- Isolated liner/head exchange
9- Isolated liner/head exchange with cementing of the liner
PREFERRED RESPONSE: 7- Cementless acetabular revision
Question 78 of 200
A 68-year-old woman with a history of thigh pain on startup since her primary total hip
arthroplasty (THA) 1 year ago (Figures 78a and 78b)
1- Cemented full revision
AAOS 2016 Adult Reconstructive Surgery of the Hip and Knee
66
2- Cemented femoral revision
3- Cemented acetabular revision
4- Cementless full revision
5- Cementless femoral revision without an extended trochanteric osteotomy
6- Cementless femoral revision with an extended trochanteric osteotomy
7- Cementless acetabular revision
8- Isolated liner/head exchange
9- Isolated liner/head exchange with cementing of the liner
PREFERRED RESPONSE: 5- Cementless femoral revision without an extended
trochanteric osteotomy
Question 79 of 200
A 54-year-old woman who had a THA 18 years ago and has mild pain with activities
(Figures 79a and 79b)
1- Cemented full revision
2- Cemented femoral revision
3- Cemented acetabular revision
4- Cementless full revision
5- Cementless femoral revision without an extended trochanteric osteotomy
6- Cementless femoral revision with an extended trochanteric osteotomy
7- Cementless acetabular revision
8- Isolated liner/head exchange
9- Isolated liner/head exchange with cementing of the liner
PREFERRED RESPONSE: 8- Isolated liner/head exchange
Question 80 of 200
An 82-year-old woman who had a THA 14 years ago and has difficulty bearing weight
(Figures 80a and 80b)
AAOS 2016 Adult Reconstructive Surgery of the Hip and Knee
67
1- Cemented full revision
2- Cemented femoral revision
3- Cemented acetabular revision
4- Cementless full revision
5- Cementless femoral revision without an extended trochanteric osteotomy
6- Cementless femoral revision with an extended trochanteric osteotomy
7- Cementless acetabular revision
8- Isolated liner/head exchange
9- Isolated liner/head exchange with cementing of the liner
PREFERRED RESPONSE: 6- Cementless femoral revision with an extended trochanteric
osteotomy
DISCUSSION
Figures 76a and 76b show likely undersizing of the stem with subsidence. Revision, with
possibly a modular fluted stem, is indicated. Extended trochanteric osteotomy is indicated to
avoid trochanteric fracture and to facilitate fully coated stem removal, especially if trephines
are needed. It is unlikely cemented fixation after multiple revisions would provide reliable
long-term survivorship.
Figures 77a and 77b show an older-style cementless acetabular component with severe
liner wear and extensive osteolysis. The well-fixed acetabular component with poor
component position and a poor locking mechanism is a good indication for cementless
acetabular revision.
Figures 78a and 78b reveal likely subsidence and lack of ongrowth of the femoral
component. Cementless femoral revision offers the best chance for long-term survivorship.
An osteotomy likely is not necessary for removal or implantation.
Figures 79a and 79b reveal severe polyethylene wear with well-fixed acetabular and
femoral components. This is a good indication for a head and liner exchange.
Figures 80a and 80b show a loose cemented stem. Cementless fixation offers good long-
term outcomes in this case. An osteotomy is likely necessary in this case because of the varus
remodeling of the proximal femur, which would increase the chance of a greater trochanteric
fracture with extraction of the existing stem and implantation of the revision stem.
Question 81 of 200
AAOS 2016 Adult Reconstructive Surgery of the Hip and Knee
68
Which modality has the broadest application for reduction of postsurgical transfusion?
1- Regional anesthesia
2- Tranexamic (TXA) acid administration
3- Reduced transfusion trigger
4- Hypotensive anesthesia
PREFERRED RESPONSE: 2- Tranexamic (TXA) acid administration
DISCUSSION
TXA is easy to administer, inexpensive, and safe for virtually all patients. Multiple studies
have demonstrated transfusion rates lower than 3% for total knee arthroplasty and lower than
10% for total hip arthroplasty. Regional and hypotensive anesthesia effectively reduce
transfusion; however, they cannot be used in as wide a range of patients as TXA. A reduced
transfusion trigger must be considered along with patient symptoms when determining the
need for transfusion.
Figure 82
Question 82 of 200
AAOS 2016 Adult Reconstructive Surgery of the Hip and Knee
69
Which wear mechanism is most likely responsible for the wear damage on the modular
tibial insert retrieval shown in Figure 82?
1- Adhesive
2- Abrasive
3- Fatigue
4- Creep
PREFERRED RESPONSE: 3- Fatigue
DISCUSSION
The figure shows the top side of a retrieved tibial liner. Pitting and delamination, which
are associated with fatigue wear, are noted. Creep is deformation without wear. Adhesive and
abrasive wear is associated with removal of material on the back side of modular tibial
components.
CLINICAL SITUATION FOR QUESTIONS 83 THROUGH 87
A bilateral cemented total knee arthroplasty (TKA) was performed on an otherwise healthy
63-year-old woman. The surgery and immediate postsurgical course were uneventful. Two
days after surgery, while in physical therapy at the hospital, the patient’s oxygen saturation is
noted at 92%.
Question 83 of 200
What is the most reasonable next step?
1- Multidetector CT scan
2- Ventilation-perfusion scan
3- Reassess on supplemental oxygen
4- Anteroposterior and lateral chest radiographs
PREFERRED RESPONSE: 3- Reassess on supplemental oxygen
AAOS 2016 Adult Reconstructive Surgery of the Hip and Knee
70
Question 84 of 200
What is the incidence of pulmonary embolism after bilateral TKA under a single
anesthetic?
1- Lower than 0.5%
2- Between 0.5% and 1.5%
3- Between 5% and 10%
4- Higher than 10%
PREFERRED RESPONSE: 2- Between 0.5% and 1.5%
Question 85 of 200
The use of pharmacologic agents and/or mechanical compressive devices for the
prevention of venous thromboembolism in patients undergoing elective hip or knee
arthroplasty who are not at elevated risk beyond risk associated with the surgery for venous
thromboembolism or bleeding was assigned which grade of recommendation by the 2011
AAOS Clinical Practice Guideline, Preventing Venous Thromboembolic Disease in Patients
Undergoing Elective Hip and Knee Arthroplasty?
1-- Strong
2- Moderate
3- Limited
4- Inconclusive
PREFERRED RESPONSE: 2- Moderate
Question 86 of 200
The patient knows that TXA was used as part of the blood conservation strategy for her
surgery. She asks you if TXA increases risk for pulmonary embolism. What is the most
appropriate response?
1- TXA increases risk for deep venous thrombosis but not pulmonary embolism after TKA.
2- TXA does not increase risk for pulmonary embolism after TKA.
3- TXA increases risk for pulmonary embolism, but the advantages outweigh the risks.
4- The risk for pulmonary embolism after TKA involving TXA has never been studied.
AAOS 2016 Adult Reconstructive Surgery of the Hip and Knee
71
PREFERRED RESPONSE: 2- TXA does not increase risk for pulmonary embolism after
TKA.
Question 87 of 200
Which gene mutation or polymorphism has been shown to most increase the risk for
venous thromboembolic disease after elective total joint arthroplasty?
1- Factor V Leiden
2- Prothrombin G20210A
3- MTHFR/C677T/TT
4- Lupus anticoagulant
PREFERRED RESPONSE: 3- MTHFR/C677T/TT
DISCUSSION
Simultaneous bilateral TKA accounts for approximately 6% of the TKAs performed in the
United States and is more frequently performed for women. The incidence of pulmonary
embolism in this group was between 0.57 and 1.14, according to a 1999 to 2008 registry-based
study in the United States. There was not a significant change in incidence during that period.
Hypoxemia alone is not an indication for advanced testing for pulmonary embolism. Winters
and associates demonstrated that to avoid unnecessary testing, the use of a hypoxia algorithm
is a reasonable first step. The use of pharmacologic agents and/or mechanical compressive
devices for the prevention of venous thromboembolism in patients undergoing elective hip or
knee arthroplasty and who are not at elevated risk beyond that of the surgery itself for venous
thromboembolism or bleeding was given a Moderate grade of recommendation in the 2011
AAOS Clinical Practice Guideline, Preventing Venous Thromboembolic Disease in Patients
Undergoing Elective Hip and Knee Arthroplasty. Various genetic factors are associated with
increased risk for venous thromboembolic disease after TKA. A recent meta-analysis
evaluated the genetic and polymorphism profiles associated with venous thromboembolism
after arthroplasty. The mutation MTHFR/C677T/TT carried the highest risk (OR 2.36; 95%
CI, 1.03-5.42, P = 0.04) for the gene mutations and polymorphisms studied. With the increased
use of TXA as a blood-conservation strategy for total joint arthroplasty, there is a theoretical
concern about an increased risk for venous thromboembolic disease. A recent study by Duncan
and associates included 13,262 elective total joint arthroplasty procedures and demonstrated
that TXA does not increase the risk of venous thromboembolism.
AAOS 2016 Adult Reconstructive Surgery of the Hip and Knee
72
Figure 88
Question 88 of 200
Figure 88 is the radiograph of a 68-year-old man who fell 3 weeks after undergoing a
successful left primary total hip arthroplasty. He is experiencing a substantial increase in pain
and an inability to bear weight. What is an appropriate treatment plan?
1- Open reduction and internal fixation (ORIF) of the fracture
2- Remove the current stem, femur ORIF, and insertion of a longer revision stem
3- Femur ORIF with cables and strut graft, leaving the current stem in situ
4- Femur ORIF combined reimplantation of the primary component
AAOS 2016 Adult Reconstructive Surgery of the Hip and Knee
73
PREFERRED RESPONSE: 2- Remove the current stem, femur ORIF, and insertion of a
longer revision stem
DISCUSSION
Based on the fact that the fracture is occurring around the stem (type B) and the stem is
clearly loose (type B2), the appropriate treatment is removal of the in situ stem (which is
loose), ORIF of the femur (cerclage wires, cables, or a plate would be acceptable), and
insertion of a longer revision stem (a tapered fluted modular titanium or fully porous coated
cylindrical stem) to bypass the fracture. All other responses are incorrect because they provide
inappropriate treatment options for a Vancouver B2 fracture.
Question 89 of 200
A patient has a painful metal-on-metal (MOM) left total hip arthroplasty (THA). Which
test(s) best correlate with prognosis if this patient is having a reaction to metal debris?
1- Erythrocyte sedimentation rate, C-reactive protein, and white blood cell count
2- Serum cobalt and chromium ion levels
3- Metal artifact reduction sequence (MARS) MRI
4- CT scan of pelvis
PREFERRED RESPONSE: 3- Metal artifact reduction sequence (MARS) MRI
DISCUSSION
Painful MOM THA and taper corrosion can cause substantial damage to a patient's hip if
left untreated. In this case, the workup for a painful MOM THA starts the same as a workup
for a painful metal-on-polyethylene bearing couple. Infection must be ruled out in every case
with a set of inflammatory markers. If these markers are remotely elevated, this is an indication
for joint aspiration. In patients with metal debris, the pathology report often indicates too many
cells to count or cellular debris. Metal ion levels do not seem to correlate with prognosis. There
are well-functioning patients with high ion levels and poor-functioning patients with low ion
levels. Advanced imaging with MARS MRI to evaluate for peritrochanteric fluid collection, a
soft-tissue mass, or synovial/capsular hypertrophy will reveal signs of a metal reaction that
indicate the need for a revision discussion. A CT scan can show more advanced bony
destruction as an indicator of poor prognosis. These films can be used to determine the need
for a structural graft or augments for reconstruction of bone loss attributable to metal debris.
AAOS 2016 Adult Reconstructive Surgery of the Hip and Knee
74
Question 90 of 200
A 58-year-old man with insulin-dependent diabetes mellitus underwent primary total knee
arthroplasty (TKA) and developed a full-thickness skin slough 3 cm x 4 cm with postsurgical
exposure of the patella tendon. There is no change in the appearance of the wound after 2
weeks of wet-to-dry dressing changes. What is the best next treatment step for the soft-tissue
defect?
1- Continue dressing changes
2- Split-thickness skin graft
3- Full-thickness skin graft
4- Local rotational flap
PREFERRED RESPONSE: 4- Local rotational flap
DISCUSSION
Following TKA, if wound healing does not occur and deep soft tissues such as the patella
tendon are exposed, local rotational flaps are the procedure of choice and should be performed
relatively early following the recognition of a soft-tissue wound-healing problem. Fortunately,
in the setting of TKA, gastrocnemius flaps are an excellent source for wound coverage of the
proximal tibia.
RESPONSES FOR QUESTIONS 91 THROUGH 94
1- Isolated head and liner revision
2- 2-stage exchange
3- Acute hemodialysis
4- Femoral component revision
5- Acetabular component revision
6- Revision total hip (all components)
Match the most appropriate treatment listed above with each clinical scenario described
below.
Question 91 of 200
A 48-year-old has had 6 weeks of groin pain and squeaking with motion of the hip joint 2
years after undergoing total hip arthroplasty (THA) with a ceramic-on-ceramic bearing surface
AAOS 2016 Adult Reconstructive Surgery of the Hip and Knee
75
using a modular acetabular component. Imaging of the hip shows an asymmetric position of
the femoral head within the acetabular component. Erythrocyte sedimentation rate (ESR) and
C-reactive protein (CRP) levels are within normal limits. Implants are in appropriate position
and not substantially damaged.
1- Isolated head and liner revision
2- 2-stage exchange
3- Acute hemodialysis
4- Femoral component revision
5- Acetabular component revision
6- Revision total hip (all components)
PREFERRED RESPONSE: 1- Isolated head and liner revision
Question 92 of 200
A 55-year-old man has groin pain and a limp with Trendelenburg gait 5 years after
undergoing THA with a metal-on-metal bearing surface and a modular acetabular component.
Serum cobalt and chromium ion levels are 55 ppb and 42 ppb, respectively. Acetabular
inclination measures 60 degrees. MR images reveal fluid collection and soft-tissue changes.
1- Isolated head and liner revision
2- 2-stage exchange
3- Acute hemodialysis
4- Femoral component revision
5- Acetabular component revision
6- Revision total hip (all components)
PREFERRED RESPONSE: 5- Acetabular component revision
Question 93 of 200
A 60-year-old has had 6 months of groin pain and a limp with Trendelenburg gait 5 years
after undergoing THA with a metal-on-polyethylene bearing surface. Serum cobalt and
chromium ion levels are 7.8 ppb and 1.1 ppb, respectively. Cross-sectional imaging shows a
3-cm pseudotumor.
AAOS 2016 Adult Reconstructive Surgery of the Hip and Knee
76
1- Isolated head and liner revision
2- 2-stage exchange
3- Acute hemodialysis
4- Femoral component revision
5- Acetabular component revision
6- Revision total hip (all components)
PREFERRED RESPONSE: 1- Isolated head and liner revision
Question 94 of 200
A 70-year-old man has a posterior dislocation 20 years after undergoing cementless THA
with a metal-on-polyethylene bearing. Acetabular inclination is 55 degrees with neutral
version. This is his third dislocation, and he has been treated with closed reduction 3 times
during the past month. His ESR is 42 mm/h (reference range [rr], 0-20 mm/h) and CRP level
is 16.2 mg/L (rr, 0.08-3.1 mg/L). Joint aspiration reveals a cell count of 865 cells (55%
neutrophils).
1- Isolated head and liner revision
2- 2-stage exchange
3- Acute hemodialysis
4- Femoral component revision
5- Acetabular component revision
6- Revision total hip (all components)
PREFERRED RESPONSE: 5- Acetabular component revision
DISCUSSION
Treatment options for various pathologic conditions after THA can be challenging, and
there are often multiple potential options. Question 91 involves a patient who has fractured a
modular ceramic acetabular liner, and an isolated head and liner exchange should be sufficient.
Question 92 involves a patient with markedly elevated serum metal ions and abductor
dysfunction, suggesting poor bearing function and a probable adverse local tissue reaction
(ALTR). Additionally, the acetabular component position is suboptimal, and complete
AAOS 2016 Adult Reconstructive Surgery of the Hip and Knee
77
acetabular revision would be appropriate. Question 93 involves a patient with elevated serum
metal ions, with cobalt disproportionately higher than chromium in a roughly 7:1 ratio,
suggesting corrosive changes at the prosthetic femoral neck (trunnionosis). Cross-sectional
imaging continues to be a key part of the evaluation and treatment of patients with metal
reactions. This patient’s pain and Trendelenburg gait are suggestive of ALTR. An isolated
head and liner exchange should be performed, typically using a ceramic head and titanium
sleeve adapter. Question 94 involves late, recurrent instability, probably related to bearing
surface wear and acetabular component position. Continued nonsurgical management is
unlikely to succeed at this point, so it would be appropriate to proceed with acetabular
component revision.
Question 95 of 200
When compared to patients with a body mass index (BMI) lower than 35, patients with a
BMI above 40 who undergo primary total hip arthroplasty (THA) and total knee arthroplasty
(TKA) are likely to have
1- smaller incisions.
2- increased wound complications.
3- fewer 30- and 90-day readmissions.
4- lower rates of patient satisfaction.
PREFERRED RESPONSE: 2- increased wound complications.
DISCUSSION
The obesity epidemic is increasing, and the number of patients with a BMI higher than 35
undergoing THA and TKA is increasing. Controversy exists regarding the optimal BMI cutoff
and the ability to perform joint replacements safely in patients who are morbidly obese. Several
clinical series as well as national database analyses have shown that morbidly obese patients
undergoing THA/TKA are at increased risk for wound complications and 30- and 90-day
readmissions. These patients’ incisions are typically larger because of the size of the soft-tissue
envelope; although their clinical scores following successful THA/TKA often are lower than
scores for controls, the overall change in clinical function and satisfaction is equivalent among
nonobese and obese patients.
AAOS 2016 Adult Reconstructive Surgery of the Hip and Knee
78
Figure 96a Figure 96b Figure 96c
CLINICAL SITUATION FOR QUESTIONS 96 THROUGH 99
Figures 96a through 96c are the anteroposterior and lateral radiographs of a 64-year-old
man with long-standing right knee osteoarthritis and pain unresponsive to nonsurgical
treatment. This patient is scheduled for navigated cruciate-retaining right total knee
arthroplasty. His range of motion is 20 to 120 degrees before surgery. Following bone
resections and release of the posterolateral capsule and iliotibial band the knee is stable and
extends fully, but during knee flexion there is lift-off of the anterior portion of the trial insert.
Alignment is neutral to the mechanical axis. A distal femoral medial resection involved 9 mm
of bone with a 9-mm-thick implant. An appropriate tibial resection was perpendicular to the
long axis, and the posterior tibial slope was 7 degrees.
Question 96 of 200
What is the best next step?
1- Increased distal femoral resection
2- Recession of the posterior cruciate ligament (PCL)
3- Use of a polyethylene insert with better sagittal conformity
4- Use of a thinner polyethylene insert
AAOS 2016 Adult Reconstructive Surgery of the Hip and Knee
79
PREFERRED RESPONSE: 2- Recession of the posterior cruciate ligament (PCL)
Question 97 of 200
Following treatment, the knee extends fully with good stability in extension, but there is
increased anterior translation in flexion. The most appropriate next step is to
1- increase polyethylene thickness.
2- increase distal femur resection and joint line elevation.
3- exchange to a polyethylene insert with more sagittal conformity.
4- perform posterior femoral resection and downsize the femoral implant.
PREFERRED RESPONSE: 3- exchange to a polyethylene insert with more sagittal
conformity.
Question 98 of 200
What is the most likely early postsurgical complication for this patient?
1- Knee dislocation
2- Peroneal nerve palsy
3- Patella fracture
4- Supracondylar fracture
PREFERRED RESPONSE: 2- Peroneal nerve palsy
Question 99 of 200
After surgery, this patient continues to experience pain and swelling of the knee with
recurrent effusions. He returns to the office with continued pain 2 years after surgery. He
describes instability, particularly when descending stairs. Upon examination, there is range of
motion from 0 to 120 degrees with no extensor lag. The knee is stable to varus and valgus
stress in extension, but there is flexion instability in both the anterior-posterior direction and
in the varus-valgus direction. Bracing leads to a slight decrease in symptoms but is not well
tolerated. Isokinetic testing demonstrates decreased knee extension velocity at mid push.
Aao2016 recon
Aao2016 recon
Aao2016 recon
Aao2016 recon
Aao2016 recon
Aao2016 recon
Aao2016 recon
Aao2016 recon
Aao2016 recon
Aao2016 recon
Aao2016 recon
Aao2016 recon
Aao2016 recon
Aao2016 recon
Aao2016 recon
Aao2016 recon
Aao2016 recon
Aao2016 recon
Aao2016 recon
Aao2016 recon
Aao2016 recon
Aao2016 recon
Aao2016 recon
Aao2016 recon
Aao2016 recon
Aao2016 recon
Aao2016 recon
Aao2016 recon
Aao2016 recon
Aao2016 recon
Aao2016 recon
Aao2016 recon
Aao2016 recon
Aao2016 recon
Aao2016 recon
Aao2016 recon
Aao2016 recon
Aao2016 recon
Aao2016 recon
Aao2016 recon
Aao2016 recon
Aao2016 recon
Aao2016 recon
Aao2016 recon
Aao2016 recon
Aao2016 recon
Aao2016 recon
Aao2016 recon
Aao2016 recon
Aao2016 recon
Aao2016 recon
Aao2016 recon
Aao2016 recon
Aao2016 recon
Aao2016 recon
Aao2016 recon
Aao2016 recon
Aao2016 recon
Aao2016 recon
Aao2016 recon
Aao2016 recon
Aao2016 recon
Aao2016 recon
Aao2016 recon
Aao2016 recon
Aao2016 recon
Aao2016 recon
Aao2016 recon
Aao2016 recon
Aao2016 recon
Aao2016 recon
Aao2016 recon
Aao2016 recon
Aao2016 recon
Aao2016 recon
Aao2016 recon
Aao2016 recon
Aao2016 recon
Aao2016 recon
Aao2016 recon
Aao2016 recon
Aao2016 recon
Aao2016 recon

More Related Content

What's hot

Percutaneous fixation of bilateral anterior column acetabular fractures: A ca...
Percutaneous fixation of bilateral anterior column acetabular fractures: A ca...Percutaneous fixation of bilateral anterior column acetabular fractures: A ca...
Percutaneous fixation of bilateral anterior column acetabular fractures: A ca...Apollo Hospitals
 
AAOS Guidelines Infection Work-Up
AAOS Guidelines Infection Work-UpAAOS Guidelines Infection Work-Up
AAOS Guidelines Infection Work-Upwashingtonortho
 
Anterior cruciate ligament reconstruction- allograft versus autograft
Anterior cruciate ligament reconstruction- allograft versus autograftAnterior cruciate ligament reconstruction- allograft versus autograft
Anterior cruciate ligament reconstruction- allograft versus autograftTunO pulciņš
 
The Direct Anterior Hip Replacement
The Direct Anterior Hip ReplacementThe Direct Anterior Hip Replacement
The Direct Anterior Hip Replacementwashingtonortho
 
Results of Mini-Open Latarjet Procedure in Failed in Arthroscopic Bankart Rep...
Results of Mini-Open Latarjet Procedure in Failed in Arthroscopic Bankart Rep...Results of Mini-Open Latarjet Procedure in Failed in Arthroscopic Bankart Rep...
Results of Mini-Open Latarjet Procedure in Failed in Arthroscopic Bankart Rep...TheRightDoctors
 
ACL Reconstruction (1)
ACL Reconstruction (1)ACL Reconstruction (1)
ACL Reconstruction (1)Ramon Valdez
 
Incorporating Biologic Technology Advances into my Practice
Incorporating Biologic Technology Advances into my PracticeIncorporating Biologic Technology Advances into my Practice
Incorporating Biologic Technology Advances into my PracticeAlan M. Hirahara, M.D., FRCSC
 
Hammer Toe Correction Comparative Study
Hammer Toe Correction Comparative StudyHammer Toe Correction Comparative Study
Hammer Toe Correction Comparative StudyWenjay Sung
 
Limb salvage vs amputation final
Limb salvage vs amputation finalLimb salvage vs amputation final
Limb salvage vs amputation finalSagar Savsani
 
Retrograde Intramedullary Nail with Femoral Head Allograft for Large Deficit ...
Retrograde Intramedullary Nail with Femoral Head Allograft for Large Deficit ...Retrograde Intramedullary Nail with Femoral Head Allograft for Large Deficit ...
Retrograde Intramedullary Nail with Femoral Head Allograft for Large Deficit ...skisnfeet
 
Arthroscopic Anterior Cruciate Ligament Reconstruction Using Four-Strand Hams...
Arthroscopic Anterior Cruciate Ligament Reconstruction Using Four-Strand Hams...Arthroscopic Anterior Cruciate Ligament Reconstruction Using Four-Strand Hams...
Arthroscopic Anterior Cruciate Ligament Reconstruction Using Four-Strand Hams...Apollo Hospitals
 
Total knee replacement (tkr) ppt
Total knee replacement (tkr) pptTotal knee replacement (tkr) ppt
Total knee replacement (tkr) pptdrshamswazir
 
Arthroscopic Latarjet: A New Fixation Technique-Dr. Sanjay Garude
Arthroscopic Latarjet: A New Fixation Technique-Dr. Sanjay GarudeArthroscopic Latarjet: A New Fixation Technique-Dr. Sanjay Garude
Arthroscopic Latarjet: A New Fixation Technique-Dr. Sanjay GarudeTheRightDoctors
 
Rotator cuff patches literature review 2012 - fraser taylor
Rotator cuff patches   literature review 2012 - fraser taylorRotator cuff patches   literature review 2012 - fraser taylor
Rotator cuff patches literature review 2012 - fraser taylorLennard Funk
 
Why do total knees fail
Why do total knees failWhy do total knees fail
Why do total knees failjatinder12345
 

What's hot (20)

Percutaneous fixation of bilateral anterior column acetabular fractures: A ca...
Percutaneous fixation of bilateral anterior column acetabular fractures: A ca...Percutaneous fixation of bilateral anterior column acetabular fractures: A ca...
Percutaneous fixation of bilateral anterior column acetabular fractures: A ca...
 
AAOS Guidelines Infection Work-Up
AAOS Guidelines Infection Work-UpAAOS Guidelines Infection Work-Up
AAOS Guidelines Infection Work-Up
 
Cementless TKA
Cementless TKACementless TKA
Cementless TKA
 
Anterior cruciate ligament reconstruction- allograft versus autograft
Anterior cruciate ligament reconstruction- allograft versus autograftAnterior cruciate ligament reconstruction- allograft versus autograft
Anterior cruciate ligament reconstruction- allograft versus autograft
 
Zimmer studies powerpoint
Zimmer studies powerpointZimmer studies powerpoint
Zimmer studies powerpoint
 
The Direct Anterior Hip Replacement
The Direct Anterior Hip ReplacementThe Direct Anterior Hip Replacement
The Direct Anterior Hip Replacement
 
Results of Mini-Open Latarjet Procedure in Failed in Arthroscopic Bankart Rep...
Results of Mini-Open Latarjet Procedure in Failed in Arthroscopic Bankart Rep...Results of Mini-Open Latarjet Procedure in Failed in Arthroscopic Bankart Rep...
Results of Mini-Open Latarjet Procedure in Failed in Arthroscopic Bankart Rep...
 
ACL Reconstruction (1)
ACL Reconstruction (1)ACL Reconstruction (1)
ACL Reconstruction (1)
 
Incorporating Biologic Technology Advances into my Practice
Incorporating Biologic Technology Advances into my PracticeIncorporating Biologic Technology Advances into my Practice
Incorporating Biologic Technology Advances into my Practice
 
Limb salvage surgery
Limb salvage surgery Limb salvage surgery
Limb salvage surgery
 
Hammer Toe Correction Comparative Study
Hammer Toe Correction Comparative StudyHammer Toe Correction Comparative Study
Hammer Toe Correction Comparative Study
 
Limb salvage vs amputation final
Limb salvage vs amputation finalLimb salvage vs amputation final
Limb salvage vs amputation final
 
Retrograde Intramedullary Nail with Femoral Head Allograft for Large Deficit ...
Retrograde Intramedullary Nail with Femoral Head Allograft for Large Deficit ...Retrograde Intramedullary Nail with Femoral Head Allograft for Large Deficit ...
Retrograde Intramedullary Nail with Femoral Head Allograft for Large Deficit ...
 
Avinash bioscrew
Avinash bioscrewAvinash bioscrew
Avinash bioscrew
 
Arthroscopic Anterior Cruciate Ligament Reconstruction Using Four-Strand Hams...
Arthroscopic Anterior Cruciate Ligament Reconstruction Using Four-Strand Hams...Arthroscopic Anterior Cruciate Ligament Reconstruction Using Four-Strand Hams...
Arthroscopic Anterior Cruciate Ligament Reconstruction Using Four-Strand Hams...
 
Total knee replacement (tkr) ppt
Total knee replacement (tkr) pptTotal knee replacement (tkr) ppt
Total knee replacement (tkr) ppt
 
Arthroscopic Latarjet: A New Fixation Technique-Dr. Sanjay Garude
Arthroscopic Latarjet: A New Fixation Technique-Dr. Sanjay GarudeArthroscopic Latarjet: A New Fixation Technique-Dr. Sanjay Garude
Arthroscopic Latarjet: A New Fixation Technique-Dr. Sanjay Garude
 
Rotator cuff patches literature review 2012 - fraser taylor
Rotator cuff patches   literature review 2012 - fraser taylorRotator cuff patches   literature review 2012 - fraser taylor
Rotator cuff patches literature review 2012 - fraser taylor
 
Aci
AciAci
Aci
 
Why do total knees fail
Why do total knees failWhy do total knees fail
Why do total knees fail
 

Similar to Aao2016 recon

Open Operative Treatment for Anterior Shoulder Instability | Orthopedic Surge...
Open Operative Treatment for Anterior Shoulder Instability | Orthopedic Surge...Open Operative Treatment for Anterior Shoulder Instability | Orthopedic Surge...
Open Operative Treatment for Anterior Shoulder Instability | Orthopedic Surge...Peter Millett MD
 
Rotator cuff-repair
Rotator cuff-repairRotator cuff-repair
Rotator cuff-repairSoulderPain
 
British Columbia Medical Journal - November 2010: Knee Arthroplasty
British Columbia Medical Journal - November 2010: Knee ArthroplastyBritish Columbia Medical Journal - November 2010: Knee Arthroplasty
British Columbia Medical Journal - November 2010: Knee ArthroplastyBritish Columbia Medical Journal
 
Rotator cuff-repair
Rotator cuff-repairRotator cuff-repair
Rotator cuff-repairSoulderPain
 
Patella in total knee arthroplasty to resurface or not is the question
Patella in total knee arthroplasty  to resurface or not is the  questionPatella in total knee arthroplasty  to resurface or not is the  question
Patella in total knee arthroplasty to resurface or not is the questionBipulBorthakur
 
Rotator cuff-repair
Rotator cuff-repairRotator cuff-repair
Rotator cuff-repairSoulderPain
 
Management of Avascular Necrosis of femoral head at Pre-Collapse stage - Dr.C...
Management of Avascular Necrosis of femoral head at Pre-Collapse stage - Dr.C...Management of Avascular Necrosis of femoral head at Pre-Collapse stage - Dr.C...
Management of Avascular Necrosis of femoral head at Pre-Collapse stage - Dr.C...DrChintan Patel
 
Calcaneal Lengthening Using Bone Substitute Graft for Neurological Flatfoot T...
Calcaneal Lengthening Using Bone Substitute Graft for Neurological Flatfoot T...Calcaneal Lengthening Using Bone Substitute Graft for Neurological Flatfoot T...
Calcaneal Lengthening Using Bone Substitute Graft for Neurological Flatfoot T...CrimsonPublishersOPROJ
 
Short Term Analysis of Clinical, Functional Radiological Outcome of Total Kne...
Short Term Analysis of Clinical, Functional Radiological Outcome of Total Kne...Short Term Analysis of Clinical, Functional Radiological Outcome of Total Kne...
Short Term Analysis of Clinical, Functional Radiological Outcome of Total Kne...iosrjce
 
Bankart Lesion: Comparison between Open and Arthroscopic Techniques – Crimson...
Bankart Lesion: Comparison between Open and Arthroscopic Techniques – Crimson...Bankart Lesion: Comparison between Open and Arthroscopic Techniques – Crimson...
Bankart Lesion: Comparison between Open and Arthroscopic Techniques – Crimson...CrimsonPublishersOPROJ
 
AAOS.2001.adult reconstruction.pdf
AAOS.2001.adult reconstruction.pdfAAOS.2001.adult reconstruction.pdf
AAOS.2001.adult reconstruction.pdfalhassansaad1
 
Fracture both forearm team v
Fracture both forearm team vFracture both forearm team v
Fracture both forearm team vReza Fahlevi
 
Biocartilage to Treat Osteochondral Defects of the Talus: Case Report and Tec...
Biocartilage to Treat Osteochondral Defects of the Talus: Case Report and Tec...Biocartilage to Treat Osteochondral Defects of the Talus: Case Report and Tec...
Biocartilage to Treat Osteochondral Defects of the Talus: Case Report and Tec...Jennifer Gerres, DPM
 
Comparison Results between Patients with Developmental Hip Dysplasia Treated ...
Comparison Results between Patients with Developmental Hip Dysplasia Treated ...Comparison Results between Patients with Developmental Hip Dysplasia Treated ...
Comparison Results between Patients with Developmental Hip Dysplasia Treated ...CrimsonPublishersOPROJ
 
Pentagon Intraarticular Osteotomy: A Novel Surgical Approach to Complex Defor...
Pentagon Intraarticular Osteotomy: A Novel Surgical Approach to Complex Defor...Pentagon Intraarticular Osteotomy: A Novel Surgical Approach to Complex Defor...
Pentagon Intraarticular Osteotomy: A Novel Surgical Approach to Complex Defor...skisnfeet
 
Tunnel Enlargement in Single Bundle ACL Reconstruction Using Bio-Interference...
Tunnel Enlargement in Single Bundle ACL Reconstruction Using Bio-Interference...Tunnel Enlargement in Single Bundle ACL Reconstruction Using Bio-Interference...
Tunnel Enlargement in Single Bundle ACL Reconstruction Using Bio-Interference...TheRightDoctors
 
Management of OA knee by osteotomies around the knee.docx
Management of OA knee by osteotomies around the knee.docxManagement of OA knee by osteotomies around the knee.docx
Management of OA knee by osteotomies around the knee.docxSanthosh Raj
 

Similar to Aao2016 recon (20)

Exploring Advances In THA
Exploring  Advances In  THAExploring  Advances In  THA
Exploring Advances In THA
 
Open Operative Treatment for Anterior Shoulder Instability | Orthopedic Surge...
Open Operative Treatment for Anterior Shoulder Instability | Orthopedic Surge...Open Operative Treatment for Anterior Shoulder Instability | Orthopedic Surge...
Open Operative Treatment for Anterior Shoulder Instability | Orthopedic Surge...
 
Rotator cuff-repair
Rotator cuff-repairRotator cuff-repair
Rotator cuff-repair
 
British Columbia Medical Journal - November 2010: Knee Arthroplasty
British Columbia Medical Journal - November 2010: Knee ArthroplastyBritish Columbia Medical Journal - November 2010: Knee Arthroplasty
British Columbia Medical Journal - November 2010: Knee Arthroplasty
 
Rotator cuff-repair
Rotator cuff-repairRotator cuff-repair
Rotator cuff-repair
 
Patella in total knee arthroplasty to resurface or not is the question
Patella in total knee arthroplasty  to resurface or not is the  questionPatella in total knee arthroplasty  to resurface or not is the  question
Patella in total knee arthroplasty to resurface or not is the question
 
Rotator cuff-repair
Rotator cuff-repairRotator cuff-repair
Rotator cuff-repair
 
Management of Avascular Necrosis of femoral head at Pre-Collapse stage - Dr.C...
Management of Avascular Necrosis of femoral head at Pre-Collapse stage - Dr.C...Management of Avascular Necrosis of femoral head at Pre-Collapse stage - Dr.C...
Management of Avascular Necrosis of femoral head at Pre-Collapse stage - Dr.C...
 
Calcaneal Lengthening Using Bone Substitute Graft for Neurological Flatfoot T...
Calcaneal Lengthening Using Bone Substitute Graft for Neurological Flatfoot T...Calcaneal Lengthening Using Bone Substitute Graft for Neurological Flatfoot T...
Calcaneal Lengthening Using Bone Substitute Graft for Neurological Flatfoot T...
 
Short Term Analysis of Clinical, Functional Radiological Outcome of Total Kne...
Short Term Analysis of Clinical, Functional Radiological Outcome of Total Kne...Short Term Analysis of Clinical, Functional Radiological Outcome of Total Kne...
Short Term Analysis of Clinical, Functional Radiological Outcome of Total Kne...
 
Bankart Lesion: Comparison between Open and Arthroscopic Techniques – Crimson...
Bankart Lesion: Comparison between Open and Arthroscopic Techniques – Crimson...Bankart Lesion: Comparison between Open and Arthroscopic Techniques – Crimson...
Bankart Lesion: Comparison between Open and Arthroscopic Techniques – Crimson...
 
AAOS.2001.adult reconstruction.pdf
AAOS.2001.adult reconstruction.pdfAAOS.2001.adult reconstruction.pdf
AAOS.2001.adult reconstruction.pdf
 
Fracture both forearm team v
Fracture both forearm team vFracture both forearm team v
Fracture both forearm team v
 
Biocartilage to Treat Osteochondral Defects of the Talus: Case Report and Tec...
Biocartilage to Treat Osteochondral Defects of the Talus: Case Report and Tec...Biocartilage to Treat Osteochondral Defects of the Talus: Case Report and Tec...
Biocartilage to Treat Osteochondral Defects of the Talus: Case Report and Tec...
 
Comparison Results between Patients with Developmental Hip Dysplasia Treated ...
Comparison Results between Patients with Developmental Hip Dysplasia Treated ...Comparison Results between Patients with Developmental Hip Dysplasia Treated ...
Comparison Results between Patients with Developmental Hip Dysplasia Treated ...
 
Pentagon Intraarticular Osteotomy: A Novel Surgical Approach to Complex Defor...
Pentagon Intraarticular Osteotomy: A Novel Surgical Approach to Complex Defor...Pentagon Intraarticular Osteotomy: A Novel Surgical Approach to Complex Defor...
Pentagon Intraarticular Osteotomy: A Novel Surgical Approach to Complex Defor...
 
arthrodesis
 arthrodesis arthrodesis
arthrodesis
 
Tunnel Enlargement in Single Bundle ACL Reconstruction Using Bio-Interference...
Tunnel Enlargement in Single Bundle ACL Reconstruction Using Bio-Interference...Tunnel Enlargement in Single Bundle ACL Reconstruction Using Bio-Interference...
Tunnel Enlargement in Single Bundle ACL Reconstruction Using Bio-Interference...
 
International Journal of Orthopedics: Research & Therapy
International Journal of Orthopedics: Research & TherapyInternational Journal of Orthopedics: Research & Therapy
International Journal of Orthopedics: Research & Therapy
 
Management of OA knee by osteotomies around the knee.docx
Management of OA knee by osteotomies around the knee.docxManagement of OA knee by osteotomies around the knee.docx
Management of OA knee by osteotomies around the knee.docx
 

Recently uploaded

Call Girls Varanasi Just Call 9907093804 Top Class Call Girl Service Available
Call Girls Varanasi Just Call 9907093804 Top Class Call Girl Service AvailableCall Girls Varanasi Just Call 9907093804 Top Class Call Girl Service Available
Call Girls Varanasi Just Call 9907093804 Top Class Call Girl Service AvailableDipal Arora
 
Call Girls Cuttack Just Call 9907093804 Top Class Call Girl Service Available
Call Girls Cuttack Just Call 9907093804 Top Class Call Girl Service AvailableCall Girls Cuttack Just Call 9907093804 Top Class Call Girl Service Available
Call Girls Cuttack Just Call 9907093804 Top Class Call Girl Service AvailableDipal Arora
 
Best Rate (Hyderabad) Call Girls Jahanuma ⟟ 8250192130 ⟟ High Class Call Girl...
Best Rate (Hyderabad) Call Girls Jahanuma ⟟ 8250192130 ⟟ High Class Call Girl...Best Rate (Hyderabad) Call Girls Jahanuma ⟟ 8250192130 ⟟ High Class Call Girl...
Best Rate (Hyderabad) Call Girls Jahanuma ⟟ 8250192130 ⟟ High Class Call Girl...astropune
 
Call Girls Aurangabad Just Call 9907093804 Top Class Call Girl Service Available
Call Girls Aurangabad Just Call 9907093804 Top Class Call Girl Service AvailableCall Girls Aurangabad Just Call 9907093804 Top Class Call Girl Service Available
Call Girls Aurangabad Just Call 9907093804 Top Class Call Girl Service AvailableDipal Arora
 
Russian Escorts Girls Nehru Place ZINATHI 🔝9711199012 ☪ 24/7 Call Girls Delhi
Russian Escorts Girls  Nehru Place ZINATHI 🔝9711199012 ☪ 24/7 Call Girls DelhiRussian Escorts Girls  Nehru Place ZINATHI 🔝9711199012 ☪ 24/7 Call Girls Delhi
Russian Escorts Girls Nehru Place ZINATHI 🔝9711199012 ☪ 24/7 Call Girls DelhiAlinaDevecerski
 
♛VVIP Hyderabad Call Girls Chintalkunta🖕7001035870🖕Riya Kappor Top Call Girl ...
♛VVIP Hyderabad Call Girls Chintalkunta🖕7001035870🖕Riya Kappor Top Call Girl ...♛VVIP Hyderabad Call Girls Chintalkunta🖕7001035870🖕Riya Kappor Top Call Girl ...
♛VVIP Hyderabad Call Girls Chintalkunta🖕7001035870🖕Riya Kappor Top Call Girl ...astropune
 
Call Girls Colaba Mumbai ❤️ 9920874524 👈 Cash on Delivery
Call Girls Colaba Mumbai ❤️ 9920874524 👈 Cash on DeliveryCall Girls Colaba Mumbai ❤️ 9920874524 👈 Cash on Delivery
Call Girls Colaba Mumbai ❤️ 9920874524 👈 Cash on Deliverynehamumbai
 
(👑VVIP ISHAAN ) Russian Call Girls Service Navi Mumbai🖕9920874524🖕Independent...
(👑VVIP ISHAAN ) Russian Call Girls Service Navi Mumbai🖕9920874524🖕Independent...(👑VVIP ISHAAN ) Russian Call Girls Service Navi Mumbai🖕9920874524🖕Independent...
(👑VVIP ISHAAN ) Russian Call Girls Service Navi Mumbai🖕9920874524🖕Independent...Taniya Sharma
 
Call Girl Number in Vashi Mumbai📲 9833363713 💞 Full Night Enjoy
Call Girl Number in Vashi Mumbai📲 9833363713 💞 Full Night EnjoyCall Girl Number in Vashi Mumbai📲 9833363713 💞 Full Night Enjoy
Call Girl Number in Vashi Mumbai📲 9833363713 💞 Full Night Enjoybabeytanya
 
VIP Call Girls Indore Kirti 💚😋 9256729539 🚀 Indore Escorts
VIP Call Girls Indore Kirti 💚😋  9256729539 🚀 Indore EscortsVIP Call Girls Indore Kirti 💚😋  9256729539 🚀 Indore Escorts
VIP Call Girls Indore Kirti 💚😋 9256729539 🚀 Indore Escortsaditipandeya
 
High Profile Call Girls Coimbatore Saanvi☎️ 8250192130 Independent Escort Se...
High Profile Call Girls Coimbatore Saanvi☎️  8250192130 Independent Escort Se...High Profile Call Girls Coimbatore Saanvi☎️  8250192130 Independent Escort Se...
High Profile Call Girls Coimbatore Saanvi☎️ 8250192130 Independent Escort Se...narwatsonia7
 
Vip Call Girls Anna Salai Chennai 👉 8250192130 ❣️💯 Top Class Girls Available
Vip Call Girls Anna Salai Chennai 👉 8250192130 ❣️💯 Top Class Girls AvailableVip Call Girls Anna Salai Chennai 👉 8250192130 ❣️💯 Top Class Girls Available
Vip Call Girls Anna Salai Chennai 👉 8250192130 ❣️💯 Top Class Girls AvailableNehru place Escorts
 
Call Girls Ooty Just Call 9907093804 Top Class Call Girl Service Available
Call Girls Ooty Just Call 9907093804 Top Class Call Girl Service AvailableCall Girls Ooty Just Call 9907093804 Top Class Call Girl Service Available
Call Girls Ooty Just Call 9907093804 Top Class Call Girl Service AvailableDipal Arora
 
VIP Call Girls Tirunelveli Aaradhya 8250192130 Independent Escort Service Tir...
VIP Call Girls Tirunelveli Aaradhya 8250192130 Independent Escort Service Tir...VIP Call Girls Tirunelveli Aaradhya 8250192130 Independent Escort Service Tir...
VIP Call Girls Tirunelveli Aaradhya 8250192130 Independent Escort Service Tir...narwatsonia7
 
Call Girls Horamavu WhatsApp Number 7001035870 Meeting With Bangalore Escorts
Call Girls Horamavu WhatsApp Number 7001035870 Meeting With Bangalore EscortsCall Girls Horamavu WhatsApp Number 7001035870 Meeting With Bangalore Escorts
Call Girls Horamavu WhatsApp Number 7001035870 Meeting With Bangalore Escortsvidya singh
 
Top Rated Bangalore Call Girls Mg Road ⟟ 8250192130 ⟟ Call Me For Genuine Sex...
Top Rated Bangalore Call Girls Mg Road ⟟ 8250192130 ⟟ Call Me For Genuine Sex...Top Rated Bangalore Call Girls Mg Road ⟟ 8250192130 ⟟ Call Me For Genuine Sex...
Top Rated Bangalore Call Girls Mg Road ⟟ 8250192130 ⟟ Call Me For Genuine Sex...narwatsonia7
 
Bangalore Call Girl Whatsapp Number 100% Complete Your Sexual Needs
Bangalore Call Girl Whatsapp Number 100% Complete Your Sexual NeedsBangalore Call Girl Whatsapp Number 100% Complete Your Sexual Needs
Bangalore Call Girl Whatsapp Number 100% Complete Your Sexual NeedsGfnyt
 
All Time Service Available Call Girls Marine Drive 📳 9820252231 For 18+ VIP C...
All Time Service Available Call Girls Marine Drive 📳 9820252231 For 18+ VIP C...All Time Service Available Call Girls Marine Drive 📳 9820252231 For 18+ VIP C...
All Time Service Available Call Girls Marine Drive 📳 9820252231 For 18+ VIP C...Arohi Goyal
 
Call Girls Kochi Just Call 9907093804 Top Class Call Girl Service Available
Call Girls Kochi Just Call 9907093804 Top Class Call Girl Service AvailableCall Girls Kochi Just Call 9907093804 Top Class Call Girl Service Available
Call Girls Kochi Just Call 9907093804 Top Class Call Girl Service AvailableDipal Arora
 
Call Girls Darjeeling Just Call 9907093804 Top Class Call Girl Service Available
Call Girls Darjeeling Just Call 9907093804 Top Class Call Girl Service AvailableCall Girls Darjeeling Just Call 9907093804 Top Class Call Girl Service Available
Call Girls Darjeeling Just Call 9907093804 Top Class Call Girl Service AvailableDipal Arora
 

Recently uploaded (20)

Call Girls Varanasi Just Call 9907093804 Top Class Call Girl Service Available
Call Girls Varanasi Just Call 9907093804 Top Class Call Girl Service AvailableCall Girls Varanasi Just Call 9907093804 Top Class Call Girl Service Available
Call Girls Varanasi Just Call 9907093804 Top Class Call Girl Service Available
 
Call Girls Cuttack Just Call 9907093804 Top Class Call Girl Service Available
Call Girls Cuttack Just Call 9907093804 Top Class Call Girl Service AvailableCall Girls Cuttack Just Call 9907093804 Top Class Call Girl Service Available
Call Girls Cuttack Just Call 9907093804 Top Class Call Girl Service Available
 
Best Rate (Hyderabad) Call Girls Jahanuma ⟟ 8250192130 ⟟ High Class Call Girl...
Best Rate (Hyderabad) Call Girls Jahanuma ⟟ 8250192130 ⟟ High Class Call Girl...Best Rate (Hyderabad) Call Girls Jahanuma ⟟ 8250192130 ⟟ High Class Call Girl...
Best Rate (Hyderabad) Call Girls Jahanuma ⟟ 8250192130 ⟟ High Class Call Girl...
 
Call Girls Aurangabad Just Call 9907093804 Top Class Call Girl Service Available
Call Girls Aurangabad Just Call 9907093804 Top Class Call Girl Service AvailableCall Girls Aurangabad Just Call 9907093804 Top Class Call Girl Service Available
Call Girls Aurangabad Just Call 9907093804 Top Class Call Girl Service Available
 
Russian Escorts Girls Nehru Place ZINATHI 🔝9711199012 ☪ 24/7 Call Girls Delhi
Russian Escorts Girls  Nehru Place ZINATHI 🔝9711199012 ☪ 24/7 Call Girls DelhiRussian Escorts Girls  Nehru Place ZINATHI 🔝9711199012 ☪ 24/7 Call Girls Delhi
Russian Escorts Girls Nehru Place ZINATHI 🔝9711199012 ☪ 24/7 Call Girls Delhi
 
♛VVIP Hyderabad Call Girls Chintalkunta🖕7001035870🖕Riya Kappor Top Call Girl ...
♛VVIP Hyderabad Call Girls Chintalkunta🖕7001035870🖕Riya Kappor Top Call Girl ...♛VVIP Hyderabad Call Girls Chintalkunta🖕7001035870🖕Riya Kappor Top Call Girl ...
♛VVIP Hyderabad Call Girls Chintalkunta🖕7001035870🖕Riya Kappor Top Call Girl ...
 
Call Girls Colaba Mumbai ❤️ 9920874524 👈 Cash on Delivery
Call Girls Colaba Mumbai ❤️ 9920874524 👈 Cash on DeliveryCall Girls Colaba Mumbai ❤️ 9920874524 👈 Cash on Delivery
Call Girls Colaba Mumbai ❤️ 9920874524 👈 Cash on Delivery
 
(👑VVIP ISHAAN ) Russian Call Girls Service Navi Mumbai🖕9920874524🖕Independent...
(👑VVIP ISHAAN ) Russian Call Girls Service Navi Mumbai🖕9920874524🖕Independent...(👑VVIP ISHAAN ) Russian Call Girls Service Navi Mumbai🖕9920874524🖕Independent...
(👑VVIP ISHAAN ) Russian Call Girls Service Navi Mumbai🖕9920874524🖕Independent...
 
Call Girl Number in Vashi Mumbai📲 9833363713 💞 Full Night Enjoy
Call Girl Number in Vashi Mumbai📲 9833363713 💞 Full Night EnjoyCall Girl Number in Vashi Mumbai📲 9833363713 💞 Full Night Enjoy
Call Girl Number in Vashi Mumbai📲 9833363713 💞 Full Night Enjoy
 
VIP Call Girls Indore Kirti 💚😋 9256729539 🚀 Indore Escorts
VIP Call Girls Indore Kirti 💚😋  9256729539 🚀 Indore EscortsVIP Call Girls Indore Kirti 💚😋  9256729539 🚀 Indore Escorts
VIP Call Girls Indore Kirti 💚😋 9256729539 🚀 Indore Escorts
 
High Profile Call Girls Coimbatore Saanvi☎️ 8250192130 Independent Escort Se...
High Profile Call Girls Coimbatore Saanvi☎️  8250192130 Independent Escort Se...High Profile Call Girls Coimbatore Saanvi☎️  8250192130 Independent Escort Se...
High Profile Call Girls Coimbatore Saanvi☎️ 8250192130 Independent Escort Se...
 
Vip Call Girls Anna Salai Chennai 👉 8250192130 ❣️💯 Top Class Girls Available
Vip Call Girls Anna Salai Chennai 👉 8250192130 ❣️💯 Top Class Girls AvailableVip Call Girls Anna Salai Chennai 👉 8250192130 ❣️💯 Top Class Girls Available
Vip Call Girls Anna Salai Chennai 👉 8250192130 ❣️💯 Top Class Girls Available
 
Call Girls Ooty Just Call 9907093804 Top Class Call Girl Service Available
Call Girls Ooty Just Call 9907093804 Top Class Call Girl Service AvailableCall Girls Ooty Just Call 9907093804 Top Class Call Girl Service Available
Call Girls Ooty Just Call 9907093804 Top Class Call Girl Service Available
 
VIP Call Girls Tirunelveli Aaradhya 8250192130 Independent Escort Service Tir...
VIP Call Girls Tirunelveli Aaradhya 8250192130 Independent Escort Service Tir...VIP Call Girls Tirunelveli Aaradhya 8250192130 Independent Escort Service Tir...
VIP Call Girls Tirunelveli Aaradhya 8250192130 Independent Escort Service Tir...
 
Call Girls Horamavu WhatsApp Number 7001035870 Meeting With Bangalore Escorts
Call Girls Horamavu WhatsApp Number 7001035870 Meeting With Bangalore EscortsCall Girls Horamavu WhatsApp Number 7001035870 Meeting With Bangalore Escorts
Call Girls Horamavu WhatsApp Number 7001035870 Meeting With Bangalore Escorts
 
Top Rated Bangalore Call Girls Mg Road ⟟ 8250192130 ⟟ Call Me For Genuine Sex...
Top Rated Bangalore Call Girls Mg Road ⟟ 8250192130 ⟟ Call Me For Genuine Sex...Top Rated Bangalore Call Girls Mg Road ⟟ 8250192130 ⟟ Call Me For Genuine Sex...
Top Rated Bangalore Call Girls Mg Road ⟟ 8250192130 ⟟ Call Me For Genuine Sex...
 
Bangalore Call Girl Whatsapp Number 100% Complete Your Sexual Needs
Bangalore Call Girl Whatsapp Number 100% Complete Your Sexual NeedsBangalore Call Girl Whatsapp Number 100% Complete Your Sexual Needs
Bangalore Call Girl Whatsapp Number 100% Complete Your Sexual Needs
 
All Time Service Available Call Girls Marine Drive 📳 9820252231 For 18+ VIP C...
All Time Service Available Call Girls Marine Drive 📳 9820252231 For 18+ VIP C...All Time Service Available Call Girls Marine Drive 📳 9820252231 For 18+ VIP C...
All Time Service Available Call Girls Marine Drive 📳 9820252231 For 18+ VIP C...
 
Call Girls Kochi Just Call 9907093804 Top Class Call Girl Service Available
Call Girls Kochi Just Call 9907093804 Top Class Call Girl Service AvailableCall Girls Kochi Just Call 9907093804 Top Class Call Girl Service Available
Call Girls Kochi Just Call 9907093804 Top Class Call Girl Service Available
 
Call Girls Darjeeling Just Call 9907093804 Top Class Call Girl Service Available
Call Girls Darjeeling Just Call 9907093804 Top Class Call Girl Service AvailableCall Girls Darjeeling Just Call 9907093804 Top Class Call Girl Service Available
Call Girls Darjeeling Just Call 9907093804 Top Class Call Girl Service Available
 

Aao2016 recon

  • 1. NEW 2016 Adult Reconstructive Surgery of the Hip and Knee Self-Assessment Examination ‫راجي‬‫بالل‬
  • 2. AAOS 2016 Adult Reconstructive Surgery of the Hip and Knee 1 Figure 1a Figure 1b Question 1 of 200 Figures 1a and 1b are the recent radiographs of an 82-year-old man with rheumatoid arthritis who underwent total knee arthroplasty (TKA) 18 years ago. These radiographs reveal osteolysis with loosening of the tibial component. Aspiration and laboratory study findings for infection are negative. During the revision TKA, treatment of tibial bone loss should consist of 1- filling the tibial defect with methylmethacrylate. 2- revision of the tibial component with porous metal augmentation. 3- reconstruction with iliac crest bone graft. 4- reconstruction with structural allograft. PREFERRED RESPONSE: 2- revision of the tibial component with porous metal augmentation. DISCUSSION
  • 3. AAOS 2016 Adult Reconstructive Surgery of the Hip and Knee 2 Video 1 for reference For severe tibial defects (Anderson Orthopaedic Research Institute [AORI] types 2 and 3), metaphyseal fixation is necessary to achieve construct fixation during revision TKA. Metaphyseal fixation may be achieved with cement, structural allograft, or conical metallic implants. The major concerns regarding structural allograft are graft resorption and mechanical failure and technical issues related to fashioning the graft and obtaining a good host-allograft interface. In a systematic review, porous metal cones were associated with a decreased loosening rate in AORI 2 and 3 defects compared to structural allografts. Metallic trabecular metal cones and metaphyseal porous coated sleeves provide a stable construct with which to support the tibial component during revision TKA. Clinical results with these devices include good metaphyseal fixation for severe tibial bone defects. Question 2 of 200 Patient-specific instrumentation (PSI) reliably demonstrates which benefit over conventional intramedullary guidance systems? 1- Lower cost 2- Improved functional outcomes 3- Better coronal alignment 4- Fewer trays PREFERRED RESPONSE: 4- Fewer trays
  • 4. AAOS 2016 Adult Reconstructive Surgery of the Hip and Knee 3 DISCUSSION Cost is usually increased with PSI because the theoretical decreased number of surgical trays and shortened surgical time do not offset cost of presurgical imaging and extra cost associated with the necessary jigs. Studies do not demonstrate a reliable improvement in functional outcomes or coronal alignment when PSI is compared to standard instrumentation. Evidence reveals that PSI necessitates fewer trays than standard instrumentation. Question 3 of 200 When performing a posterior cruciate-substituting total knee revision, trial components are inserted. The knee comes to full extension but is tight in flexion. The surgeon should consider 1- flexing the femoral component. 2- downsizing the femoral component. 3- downsizing the tibial component thickness. 4- resecting more distal femur. PREFERRED RESPONSE: 2- downsizing the femoral component. DISCUSSION In this scenario, the extension gap is normal and the flexion gap is tight. Increasing the flexion gap without changing the extension gap can be performed by downsizing the femoral component or adding posterior slope to the tibia resection. Flexing the femoral component tightens the flexion gap. Decreasing the tibial component thickness loosens the flexion and extension gaps. Resecting more distal femur only loosens the extension gap.
  • 5. AAOS 2016 Adult Reconstructive Surgery of the Hip and Knee 4 Figure 4 CLINICAL SITUATION FOR QUESTIONS 4 THROUGH 7 Figure 4 is the radiograph of a 73-year-old woman who returns for her annual follow-up 14 years after undergoing total hip arthroplasty. She denies pain and has no discomfort upon examination. Question 4 of 200 Which method of terminal polyethylene sterilization would most likely cause these radiographic findings? 1- Gamma irradiation in nitrogen 2- Gamma irradiation in air 3- Gamma irradiation in argon 4- Gas plasma PREFERRED RESPONSE: 2- Gamma irradiation in air Question 5 of 200
  • 6. AAOS 2016 Adult Reconstructive Surgery of the Hip and Knee 5 The patient is asked to return in 6 months for a repeat radiograph. Six months later she remains asymptomatic, but a radiograph reveals progressive osteolysis. Treatment should consist of 1- femoral head/liner exchange with retroacetabular bone grafting. 2- femoral and acetabular revision. 3- acetabular revision. 4- follow-up in 1 year. PREFERRED RESPONSE: 1- femoral head/liner exchange with retroacetabular bone grafting. Question 6 of 200 The patient begins to experience pain, and a decision is made to proceed with surgical intervention. When performing a posterior approach to the hip, which structure protects the anterior retractor from causing damage to the femoral neurovascular structures? 1- Obturator externus 2- Piriformis 3- Psoas 4- Rectus femoris PREFERRED RESPONSE: 3- Psoas Question 7 of 200 The patient develops an inability to dorsiflex her foot 2 days after surgical intervention while she is sitting in a chair after physical therapy. Initial treatment should consist of 1- lying completely supine in bed. 2- remaining seated and placing the postsurgical leg on a stool. 3- transferring back to bed with the head of the bed no lower than 60 degrees. 4- transferring back to bed with the head of the bed level and the surgical knee flexed.
  • 7. AAOS 2016 Adult Reconstructive Surgery of the Hip and Knee 6 PREFERRED RESPONSE: 4- transferring back to bed with the head of the bed level and the surgical knee flexed. DISCUSSION Gamma irradiation produces free radicals. Although these free radicals can form cross- links with other polyethylene chains, the free radicals can also form a bond with oxygen, resulting in early oxidation. Gamma irradiation in air produces the highest risk for oxidized polyethylene, resulting in the highest risk for wear, delamination, and subsequent osteolysis. This patient demonstrates severe periarticular osteolysis. When she is asymptomatic, this suggests the acetabular and femoral components remain well fixed to the bone. Consequently, she can be treated by removing the wear generator (polyethylene exchange), along with bone grafting of the osteolytic defect. Considering the extensive amount of osteolysis, observation for 1 year would not be appropriate. The psoas is the anatomic structure that runs anterior to the acetabulum. The femoral neurovascular structures are at risk if the retractor is placed anterior and inferior to the psoas tendon. The patient develops a foot drop 2 days after surgery. As a result, it can be assumed that the nerve was not injured directly during the surgical procedure. Although MR imaging or a CT scan may be indicated to identify an evolving hematoma, the immediate concern is to minimize pressure on the sciatic nerve. Tension on the nerve can be decreased by flexing the surgical knee and positioning the bed flat. Question 8 of 200 An 80-year-old African American woman who lives in a large city is scheduled for total hip arthroplasty to address primary osteoarthritis. Part of the presurgical protocol includes nasal swab screening to assess for methicillin-resistant Staphylococcus aureus (MRSA) colonization. Which demographic factor places this patient at highest risk for a positive result? 1- Gender 2- Age 3- Race 4- Environment PREFERRED RESPONSE: 3- Race DISCUSSION
  • 8. AAOS 2016 Adult Reconstructive Surgery of the Hip and Knee 7 Demographic factors are associated with increased risk for MRSA colonization, so it is important to identify vulnerable patients. Female gender and advanced age decrease risk for colonization, while African American race increases this risk. Urban environments do not influence MRSA colonization. Figure 9 Question 9 of 200 Figure 9 is the clinical photograph of a 68-year-old woman 10 days after undergoing primary total knee replacement. She is experiencing hemarthrosis, discoloration, and bruising of the soft tissue about the knee; her history includes persistent serous drainage. This clinical appearance likely is associated with 1- failure to use a tourniquet. 2- failure to use a drain. 3- use of low-molecular-weight heparin (LMWH). 4- use of regional anesthesia. PREFERRED RESPONSE: 3- use of low-molecular-weight heparin (LMWH).
  • 9. AAOS 2016 Adult Reconstructive Surgery of the Hip and Knee 8 DISCUSSION Certain anticoagulants are associated with an increased risk for wound complications. Two studies showed an increase in postsurgical bleeding and wound drainage following use of LMWH. Other investigators have associated use of anticoagulants such as LMWH with an increased incidence of persistent wound drainage and subsequent infection. Current evidence does not support a significant difference in complication rates with and without the use of wound drains or a tourniquet. Regional anesthesia has been associated with less blood loss than general anesthesia and is not associated with a difference in wound complication incidence. Figure 10 Question 10 of 200 Figure 10 is the radiograph of a 44-year-old man with a long-standing history of severe hip pain and a limp. Which clinical scenario most likely could occur when performing total hip arthroplasty on this patient? 1- Placing the hip center too inferior 2- Overmedializing the acetabular component 3- Overlengthening the extremity 4- Intrasurgical acetabular fracture
  • 10. AAOS 2016 Adult Reconstructive Surgery of the Hip and Knee 9 PREFERRED RESPONSE: 3- Overlengthening the extremity DISCUSSION The radiograph reveals hip dysplasia. Patients with hip dysplasia and severe limb shortening are at high risk for sciatic nerve palsy from overlengthening. Overmedializing the acetabular component is not the preferred response because overlateralization is more of a concern if the cup is placed in the pseudoacetabulum instead of in the true acetabulum. Placing the hip center too inferior is not the preferred response because the concern in this scenario is placing the hip center too superior if the cup is placed in the pseudoacetabulum or if a large- diameter cup is used. Acetabular fractures are possible because of osteoporotic bone at the true hip center, but is less likely than overlengthening of the extremity. Figure 11a Figure 11b CLINICAL SITUATION FOR QUESTIONS 11 THROUGH 13 Figures 11a and 11b are the radiographs of a 35-year-old woman with end-stage debilitating osteoarthritis of the right hip. She is contemplating total hip arthroplasty (THA). She has a history of right hip dysplasia and underwent hip osteotomy as an adolescent. Over the years, she has failed nonsurgical treatment including weight loss, activity modifications, and intra-articular injections. Her infection workup reveals laboratory findings within defined limits. Question 11 of 200 Which bearing surface is contraindicated for this patient?
  • 11. AAOS 2016 Adult Reconstructive Surgery of the Hip and Knee 10 1- Ceramic-on-ceramic 2- Ceramic-on-highly cross-linked polyethylene (HXPE) 3- Metal-on-HXPE 4- Metal-on-metal PREFERRED RESPONSE: 4- Metal-on-metal Question 12 of 200 The patient undergoes successful primary THA with a metal-on-metal bearing. At her 1- year follow-up appointment, she reports no pain and is highly satisfied with the procedure. However, 3 years after the index procedure, she reports atraumatic right hip pain that worsens with activities. Radiographs reveal implants in good position with no sign of loosening or lysis. An initial laboratory evaluation reveals a normal sedimentation rate and C-reactive protein (CRP) level. The most appropriate next diagnostic step is 1- metal artifact reduction sequence (MARS) MR imaging only. 2- serum cobalt only. 3- serum cobalt and chromium levels. 4- serum cobalt and chromium levels and MARS MR imaging. PREFERRED RESPONSE: 4- serum cobalt and chromium levels and MARS MR imaging. Question 13 of 200 A further workup reveals elevations in serum cobalt and chromium levels and fluid collections surrounding the hip on MARS MR imaging. Revision THA is recommended. The most common complication following revision of a failed metal-on-metal hip arthroplasty is 1- infection. 2- instability. 3- loosening. 4- periprosthetic fracture.
  • 12. AAOS 2016 Adult Reconstructive Surgery of the Hip and Knee 11 PREFERRED RESPONSE: 2- instability. DISCUSSION THA has proven durable and reliable for pain relief and improving function for patients with end-stage arthritis. Appropriate bearing selection is critical to minimize wear and hip complications. A metal-on-metal articulation is associated with excellent wear rates in vitro. With its capacity to offer a low wear rate with large femoral heads, it is an attractive bearing choice for THA. However, local soft-tissue reactions, pseudotumors, and potential systemic reactions including renal failure, cardiomyopathy, carcinogenesis, and potential teratogenesis with potential transfer of metal ions across the placental barrier make metal-on-metal bearings less desirable and relatively contraindicated for younger women of child-bearing age. The workup of a painful metal-on-metal hip arthroplasty necessitates a systematic approach. Several algorithms have been proposed. Routine laboratory studies including sedimentation rate, CRP, and serum cobalt and chromium ion levels should be obtained for all patients with pain. Advanced imaging including MARS MRI should be performed to evaluate for the presence of fluid collections, pseudotumors, and abductor mechanism destruction. Infection can coexist with metal-on-metal reactions, so, when indicated (if the CRP level is elevated), a hip arthrocentesis should be obtained. However, in this setting, a manual cell count and differential should be obtained because an automated cell counter may provide falsely elevated cell counts. The results of revision surgery for a failed metal-on-metal hip prosthesis can be variable. The amount of local tissue destruction and the integrity of the hip abductor mechanism can greatly influence outcomes. Instability is the most common complication following revision of failed metal-on-metal hip replacements. Question 14 of 200 A surgeon is preparing a medial gastrocnemius rotational flap to cover a medial proximal tibia defect at the time of revision knee replacement surgery. To optimize coverage, the surgeon must optimally mobilize which artery? 1- Profundus femoris 2- Middle genicular 3- Medial sural 4- Inferior medial genicular PREFERRED RESPONSE: 3- Medial sural
  • 13. AAOS 2016 Adult Reconstructive Surgery of the Hip and Knee 12 DISCUSSION The medial sural arteries vascularize the gastrocnemius, plantaris, and soleus muscles proximally. They arise from the popliteal artery. If not adequately mobilized, a gastroc soleus flap can be devascularized. RESPONSES FOR QUESTIONS 15 THROUGH 17 1- Semimembranosis release 2- Medial gastrocnemius release 3- Medial tibial plateau downsizing osteotomy 4- Iliotibial band pie crusting 5- Popliteus tendon release 6- Cruciate release of the capsule posterior lateral corner Select the most appropriate release listed above to address each scenario described below. Question 15 of 200 A 57-year-old man is taken to the operating room to address a severe varus right knee deformity of approximately 17 degrees. After releasing the deep medial collateral ligament (MCL) as part of the approach, the diseased side of the knee remains tight. Following osteophyte removal, which soft-tissue release is now most appropriate to obtain a balanced knee? 1- Semimembranosis release 2- Medial gastrocnemius release 3- Medial tibial plateau downsizing osteotomy 4- Iliotibial band pie crusting 5- Popliteus tendon release 6- Cruciate release of the capsule posterior lateral corner PREFERRED RESPONSE: 1- Semimembranosis release Question 16 of 200 Despite that release, the knee remains very tight medially. Cuts are validated for the patient's axial alignment. For a patient with a large tibia, what is the best release to address a large residual deformity?
  • 14. AAOS 2016 Adult Reconstructive Surgery of the Hip and Knee 13 1- Semimembranosis release 2- Medial gastrocnemius release 3- Medial tibial plateau downsizing osteotomy 4- Iliotibial band pie crusting 5- Popliteus tendon release 6- Cruciate release of the capsule posterior lateral corner PREFERRED RESPONSE: 3- Medial tibial plateau downsizing osteotomy Question 17 of 200 The patient is planning on having his contralateral knee replaced as well. He has a mild valgus deformity in his left knee with an overall windswept deformity. Which release is most appropriate in this case if the knee remains tight in extension? 1- Semimembranosis release 2- Medial gastrocnemius release 3- Medial tibial plateau downsizing osteotomy 4- Iliotibial band pie crusting 5- Popliteus tendon release 6- Cruciate release of the capsule posterior lateral corner PREFERRED RESPONSE: 4- Iliotibial band pie crusting DISCUSSION Balancing a total knee is important for longevity of the device and functional benefit. The surgeon should be systematic in the release of a varus knee. The deep MCL is typically released as part of the approach and osteophytes are then removed. The semimembranosus tendon can then be released from the posterior medial aspect of the tibia. A downsizing osteotomy can be considered for a large deformity if a patient has adequate tibial sizing. If a patient has the smallest implant available prior to the osteotomy, an osteotomy will lead to overhang of the implant and medial impingement on the MCL. A valgus knee can be treated with pie crusting of the iliotibial band in mild extension deformity. Surgeons should pause prior to taking down the popliteus and lateral collateral
  • 15. AAOS 2016 Adult Reconstructive Surgery of the Hip and Knee 14 ligament because this can induce posterior rotatory subluxation of a primary knee, especially in the case of a posterior collateral ligament-sacrificing total knee arthroplasty design. Figure 18 Question 18 of 200 Figure 18 is the radiograph of a 52-year-old woman who has leg length inequality and chronic activity-related buttock discomfort. This has been a life-long problem, but it is getting worse and increasingly causing back pain. What is the best current technique for total hip arthroplasty? 1- High hip center 2- Anatomic hip center with trochanteric osteotomy and progressive femoral shortening 3- Anatomic hip center with subtrochanteric shortening osteotomy 4- Iliofemoral lengthening followed by an anatomic hip center PREFERRED RESPONSE: 3- Anatomic hip center with subtrochanteric shortening osteotomy DISCUSSION
  • 16. AAOS 2016 Adult Reconstructive Surgery of the Hip and Knee 15 A high hip center is not recommended for Crowe IV hips because of the lack of acetabular bone and altered hip biomechanics. An anatomic center is a better option but necessitates a technique to address the tight soft-tissue envelope. A trochanteric osteotomy with progressive femoral shortening has been described but can be prone to trochanter nonunion. Iliofemoral lengthening prior to surgery has been described but may not be tolerated by all patients. A shortening subtrochanteric osteotomy avoids trochanter nonunion and allows adjustment of femoral anteversion. Fixation of the osteotomy can include a stem with distal rotational control, plate fixation, a step vs. oblique cut, or strut grafts. Question 19 of 200 The optimal method with which to diagnose component malrotation in total knee arthroplasty (TKA) is 1- clinical assessment of foot position and patellar tracking. 2- radiographic skyline view of the patella. 3- CT scan with metal artifact suppression. 4- MR imaging with metal artifact reduction sequences. PREFERRED RESPONSE: 3- CT scan with metal artifact suppression. DISCUSSION The epicondylar axis and tibial tubercle can be used as references on CT scans to quantitatively measure rotational alignment of the femoral and tibial components. This technique has been used to determine whether rotational malalignment is present and whether revision of 1 or both components may be indicated. Although clinical assessment is useful, malrotation can occur as a result of deformities unrelated to the arthroplasty. Similarly, an isolated radiographic skyline view of the patella may indicate a problem with patellar maltracking, but cannot quantitatively assess malrotation of the components. MR imaging proves useful for evaluating painful TKA, but it is dependent on the center’s quality and has not yet been shown to quantify component malrotation.
  • 17. AAOS 2016 Adult Reconstructive Surgery of the Hip and Knee 16 Figure 20a Figure 20b CLINICAL SITUATION FOR QUESTIONS 20 THROUGH 22 Figures 20a and 20b are the radiographs of an elderly woman who underwent total knee arthroplasty (TKA) several years ago. She now states that something is not right; her knee frequently swells and is diffusely painful, especially at the end of the day. She does not trust her knee, especially while going up and down stairs or getting up from a chair. Question 20 of 200 The most important diagnostic step is to 1- examine flexion and extension gap stability. 2- examine extensor power. 3- perform a neurological examination. 4- feel for the presence of foot pulses. PREFERRED RESPONSE: 1- examine flexion and extension gap stability. Question 21 of 200
  • 18. AAOS 2016 Adult Reconstructive Surgery of the Hip and Knee 17 The initial diagnostic workup should include 1- CT scan of the knee. 2- erythrocyte sedimentation rate and C-reactive protein level. 3- electromyography and nerve conduction studies. 4- a technetium bone scan. PREFERRED RESPONSE: 2- erythrocyte sedimentation rate and C-reactive protein level. Question 22 of 200 Surgical treatment for this patient should include 1- excision arthroplasty with placement of an articulating antibiotic cement spacer. 2- excision arthroplasty with placement of a static antibiotic cement spacer. 3- revision to a more constrained prosthesis, with reestablishment of the flexion and extension gap balance. 4- a thicker polyethylene insert. PREFERRED RESPONSE: 3- revision to a more constrained prosthesis, with reestablishment of the flexion and extension gap balance. DISCUSSION
  • 19. AAOS 2016 Adult Reconstructive Surgery of the Hip and Knee 18 Video 22 for reference This patient has an unstable cruciate-retaining TKA. The tibial cut appears to be substantial, necessitating a very thick polyethylene liner. In addition, the femoral component may be slightly more proximally located. Tests for flexion-extension gap balancing would indicate flexion instability. However, the possibility of infection remains, so screening blood tests are appropriate. The surgical treatment for this unstable knee is revision TKA to a more constrained implant, ensuring flexion-extension gap balancing. No evidence indicates that the knee is infected. A thicker polyethylene insert will not adequately balance the knee. Figure 23a Figure 23b
  • 20. AAOS 2016 Adult Reconstructive Surgery of the Hip and Knee 19 Figure 23c Figure 23d Figure 23e Figure 23f
  • 21. AAOS 2016 Adult Reconstructive Surgery of the Hip and Knee 20 Figure 23g Figure 23h Question 23 of 200 Figures 23a through 23h are the radiographs and MR images of a 32-year-old man with worsening left knee pain. A 3-foot hip-to-ankle radiograph shows a 13-degree varus knee deformity. The patient sustained a major left knee injury 5 years ago and a confirmed complete anterior cruciate ligament (ACL) tear. He managed this injury nonsurgically with a functional brace but experienced worsening pain. He was seen by an orthopaedic surgeon 18 months ago and a medial meniscus tear was diagnosed; the tear was treated with an arthroscopic partial medial meniscectomy. Since then, his knee has been giving way more often and he no longer feels safe working on a pitched roof. The patient received 6 months of formal physical therapy and was fitted for a new functional ACL brace, but he still has pain and instability symptoms. He believes he has exhausted his nonsurgical options and would like to undergo surgery. What is the most appropriate treatment at this time? 1- ACL reconstruction and subsequent proximal tibial osteotomy 2- ACL reconstruction alone 3- Distal femoral osteotomy with simultaneous ACL reconstruction 4- Proximal tibial osteotomy with subsequent ACL reconstruction PREFERRED RESPONSE: 4- Proximal tibial osteotomy with subsequent ACL reconstruction
  • 22. AAOS 2016 Adult Reconstructive Surgery of the Hip and Knee 21 DISCUSSION Proximal tibial osteotomy is the most appropriate intervention to correct varus malalignment and to decrease stress on the ACL. In some cases, proximal tibial osteotomy alone may address both pain and instability, but, if instability persists, particularly in the setting in which instability can be dangerous, subsequent ACL reconstruction can further stabilize the knee with less stress on the graft after correction of malalignment. Varus alignment places increased stress on the native or reconstructed ACL. ACL reconstruction should only be performed at the same time as or following proximal tibial osteotomy to correct alignment in the setting of varus malalignment. It is not appropriate to perform ACL reconstruction prior to proximal tibial osteotomy in this setting. Distal femoral osteotomy is not indicated to correct varus malalignment. Varus alignment places increased stress on the native or reconstructed ACL, and ACL reconstruction alone is not indicated for this patient. Question 24 of 200 A surgeon performs a minimally invasive total knee arthroplasty through a quadriceps- sparing approach using medial-to-lateral cutting jigs. When beginning therapy that afternoon, the patient can passively but not actively extend her knee, although she has minimal knee pain. All regional blocks have been discontinued. What is the most likely reason for this finding? 1- Quadriceps inhibition 2- Avulsion of the quadriceps tendon 3- Laceration of the patella tendon 4- Femoral nerve palsy PREFERRED RESPONSE: 3- Laceration of the patella tendon DISCUSSION This patient lacks active knee extension. It is not attributable to the regional block because that block is no longer acting. The most likely cause is laceration of the patella tendon, which has been described during both large-incision surgery and minimally invasive surgery. However, this is reported with increased frequency during minimally invasive surgery. Quadriceps inhibition, avulsion of the quadriceps tendon, and femoral nerve palsy can cause lack of active extension, but these problems are less likely because the patient has minimal pain.
  • 23. AAOS 2016 Adult Reconstructive Surgery of the Hip and Knee 22 Figure 25a Figure 25b Question 25 of 200 Figures 25a and 25b are the radiographs of a 63-year-old man who had right total hip arthroplasty (THA) 4 months ago. Progressive stiffness began 2 months after surgery, and he now reports pain only after prolonged physical activity. His examination reveals normal gait and painless range of motion with flexion of 70 degrees, extension of 0 degrees, internal rotation of 20 degrees, external rotation of 20 degrees, abduction of 10 degrees, and adduction of 10 degrees. His erythrocyte sedimentation rate (ESR) and C-reactive protein (CRP) levels are within defined limits. Physical therapy has produced no benefit. What is the most appropriate next step? 1- 25 mg of indomethacin 3 times daily for 6 weeks 2- 1 dose of irradiation at 800 Gy 3- Surgical excision of heterotypic ossification (HO) 4- Reevaluation in 6 months PREFERRED RESPONSE: 4- Reevaluation in 6 months
  • 24. AAOS 2016 Adult Reconstructive Surgery of the Hip and Knee 23 DISCUSSION This patient presents with HO 4 months after undergoing THA. Symptomatic HO may complicate nearly 7% of primary THA cases. Improvement in pain is expected within 6 months, and most patients will not need surgical treatment. Surgical excision may be warranted for symptomatic patients after full maturation of the HO, usually 6 to 18 months after the surgery. Patients can be followed with repeated serum alkaline phosphatase levels, which are elevated initially and should return to normal upon maturation of HO. Alternatively, a bone scan can show decreased activity once the HO has matured. Twenty-five milligrams of indomethacin 3 times daily for 6 weeks or 1 dose of irradiation at 700 to 800 Gy is effective in the prevention of HO, not for the treatment of established HO. RESPONSES FOR QUESTIONS 26 THROUGH 29 1- Minimum inhibitory concentration (MIC) 2- Minimum bactericidal concentration (MBC) 3- Antiobiotic susceptibility 4- Antibiograms 5- Antimicrobial resistance 6- Spectrum of coverage Match the description below with the appropriate response above. Question 26 of 200 The minimum concentration of an antimicrobial agent to prevent growth of a microorganism. 1- Minimum inhibitory concentration (MIC) 2- Minimum bactericidal concentration (MBC) 3- Antiobiotic susceptibility 4- Antibiograms 5- Antimicrobial resistance 6- Spectrum of coverage PREFERRED RESPONSE: 1- Minimum inhibitory concentration (MIC)
  • 25. AAOS 2016 Adult Reconstructive Surgery of the Hip and Knee 24 Question 27 of 200 Refers to the prevalence of microorganisms at a particular hospital or institution. 1- Minimum inhibitory concentration (MIC) 2- Minimum bactericidal concentration (MBC) 3- Antiobiotic susceptibility 4- Antibiograms 5- Antimicrobial resistance 6- Spectrum of coverage PREFERRED RESPONSE: 4- Antibiograms Question 28 of 200 Refers to the effectiveness of an antibiotic against various classes of microorganisms. 1- Minimum inhibitory concentration (MIC) 2- Minimum bactericidal concentration (MBC) 3- Antiobiotic susceptibility 4- Antibiograms 5- Antimicrobial resistance 6- Spectrum of coverage PREFERRED RESPONSE: 6- Spectrum of coverage Question 29 of 200 Refers to the effectiveness of various antibiotics against a particular microorganism. 1- Minimum inhibitory concentration (MIC) 2- Minimum bactericidal concentration (MBC) 3- Antiobiotic susceptibility 4- Antibiograms 5- Antimicrobial resistance 6- Spectrum of coverage
  • 26. AAOS 2016 Adult Reconstructive Surgery of the Hip and Knee 25 PREFERRED RESPONSE: 3- Antiobiotic susceptibility DISCUSSION Knowledge of the basic nomenclature of antibiotic use is important to effectively treat patients and communicate with colleagues. The effectiveness of an antimicrobial against an infecting organism is measured by the MIC, which refers to the concentration needed to prevent growth of a microorganism on culture medium, and MBC, which is the smallest concentration of the antibiotic necessary to kill the microorganism in culture. Typically, an antibiotic is considered bactericidal if the MBC is no more than 4 times the MIC. The spectrum of antimicrobial coverage refers to an agent’s effectiveness against a range of bacteria. An antibiogram refers to the tabulation of prevalence of different bacteria in a specific setting or specific patient population. Antibiotic susceptibility and resistance refers to the bacteria’s ability to be affected or unaffected by a given antibiotic. Figure 30a Question 30 of 200 Figure 30a is the anteroposterior radiograph of a 20-year-old woman with mild right groin pain and intermittent “catching” in the hip region. What is the most appropriate next step?
  • 27. AAOS 2016 Adult Reconstructive Surgery of the Hip and Knee 26 1- Arthroscopic evaluation and treatment of the hypertrophic labrum and a possible labral tear 2- A hip injection to confirm an intra-articular source of the pain 3- Nonsurgical treatment and subsequent total hip arthroplasty (THA) when the patient is sufficiently symptomatic 4- Periacetabular osteotomy PREFERRED RESPONSE: 4- Periacetabular osteotomy DISCUSSION Because this patient is young, substantial bilateral acetabular dysplasia is present, and the joint space is well preserved, periacetabular osteotomy is the treatment of choice (Figure 30b). Arthroscopic evaluation and treatment is insufficient to address the mechanical deformity. Although a hip injection can be diagnostically helpful, it would not alter the treatment plan in this scenario. The patient’s young age would make observation and subsequent THA less desirable. Femoral osteotomies also were performed to address rotational deformity. Figure 31
  • 28. AAOS 2016 Adult Reconstructive Surgery of the Hip and Knee 27 Question 31 of 200 Figure 31 is the abdominal radiograph of a 70-year-old woman who experiences nausea and abdominal tightness 48 hours following left total knee arthroplasty performed under general anesthesia. She received 24 hours of cefazolin antibiotic prophylaxis and a patient- controlled analgesia narcotic pump for pain management. She has been receiving warfarin for thromboembolic prophylaxis. Her severe abdominal distension and markedly decreased bowel sounds are most likely secondary to 1- general anesthesia. 2- administration of antibiotics. 3- administration of warfarin. 4- administration of narcotics. PREFERRED RESPONSE: 4- administration of narcotics. DISCUSSION The radiograph reveals severe intestinal dilatation, which has occurred as the result of acute colonic pseudo-obstruction and is associated with excessive narcotic administration following total joint arthroplasty. Anesthetic type, antibiotic administration, and warfarin have not been associated with this obstruction. Electrolyte imbalances such as hypokalemia have been associated with postsurgical acute colonic pseudo-obstruction. Figure 32a Figure 32b Figure 32c CLINICAL SITUATION FOR QUESTIONS 32 THROUGH 35 Figures 32a through 32c are the radiographs of a 30-year old man who is experiencing right hip pain. He has no current medical problems, but, with a body mass index of 41, he is morbidly obese. He was previously treated for leukemia with chemotherapy that included
  • 29. AAOS 2016 Adult Reconstructive Surgery of the Hip and Knee 28 high-dose steroids. He undergoes total hip arthroplasty (THA) with a ceramic-on-ceramic bearing. Question 32 of 200 When counseling this patient regarding the long-term outcomes of surgery using a ceramic-on-ceramic bearing, he should be informed that 1- there will be more overall bearing wear than metal on polyethylene. 2- there will be less long-term revision risk. 3- complication rates will be the same. 4- pain and function scores will be better. PREFERRED RESPONSE: 2- there will be less long-term revision risk. Question 33 of 200 Three years after undergoing THA with a ceramic-on-ceramic bearing, this patient returns because of right hip pain and a grinding sensation. At revision, it is discovered that the ceramic liner has fractured. The most likely cause for this complication is 1- horizontal cup position. 2- morbid obesity. 3- trunnion damage. 4- activity level. PREFERRED RESPONSE: 2- morbid obesity. Question 34 of 200 At revision, the ceramic femoral head is removed. Upon visual inspection, the most likely finding on the head is 1- stripe wear. 2- no gross damage. 3- metal transfer. 4- fracture.
  • 30. AAOS 2016 Adult Reconstructive Surgery of the Hip and Knee 29 PREFERRED RESPONSE: 1- stripe wear. Question 35 of 200 At revision, the stem is retained and a new head with a polyethylene bearing is selected. The best option for the head is 1- ceramic with a metal sleeve. 2- ceramic alone. 3- metal with a metal sleeve. 4- metal alone. PREFERRED RESPONSE: 1- ceramic with a metal sleeve. DISCUSSION Ceramic-on-ceramic is a controversial bearing surface typically reserved for younger patients such as this one. Some studies have suggested that the bearing is more expensive and does not really prolong the service life of the implant, although a recent meta-analysis of high- quality trials showed that there is a decreased revision rate with ceramic-on-ceramic, so its use may be justified. Complications of intraoperative bearing fracture and squeaking are more common than with conventional bearings, but pain and function scores are equivalent. Stripe wear associated with a vertical cup and morbid obesity are related to an increased risk for liner fracture. Concerns about head fractures with a new ceramic head and a damaged trunnion have led investigators to conclude that using a harder bearing than the initial bearing surface with a built-in titanium sleeve is probably the best solution when a stem is retained during revision surgery.
  • 31. AAOS 2016 Adult Reconstructive Surgery of the Hip and Knee 30 Figure 36a Figure 36b Question 36 of 200 Figures 36a and 36b are the radiographs of a 79-year-old woman who has been experiencing increasing tibial pain 10 years after undergoing revision total knee arthroplasty. There is no evidence of infection. What is the most appropriate treatment? 1- Retain the components and implant a tibial strut allograft 2- Revise the tibial component with a metaphyseal cone and metaphyseal uncemented stem 3- Revise the tibial component with a metaphyseal cone and a press-fit diaphyseal- engaging stem 4- Revise the tibial component with a long cemented diaphyseal-engaging stem PREFERRED RESPONSE: 3- Revise the tibial component with a metaphyseal cone and a press-fit diaphyseal-engaging stem DISCUSSION Stems are available for cemented and press-fit implantation. To be effective, press-fit stems should engage the diaphysis (Figures 36c and 36d). They also assist in obtaining correct limb alignment. Short metaphyseal-engaging stems are associated with failure rates between
  • 32. AAOS 2016 Adult Reconstructive Surgery of the Hip and Knee 31 16% and 29%. Cemented stems may be shorter than press-fit stems because they do not have to engage the diaphysis. Short, fully cemented stems offer the advantage of metaphyseal fixation. Hybrid stem fixation makes use of the metaphysis for cement fixation with metaphyseal cones or sleeves and diaphyseal-engaging press-fit stems. RESPONSES FOR QUESTIONS 37 THROUGH 40 1- Lateral femoral cutaneous nerve 2- Lateral femoral circumflex artery 3- Superior gluteal nerve 4- Superior gluteal artery 5- Sciatic nerve 6- Femoral artery 7- Femoral vein 8- Femoral nerve 9- Saphenous branch of the femoral nerve 10- Profunda femoris artery 11- Inferior gluteal nerve Match each description below with the anatomic structure listed above. Question 37 of 200 When performing a modified direct lateral approach (modified Hardinge) to the hip, this is the neurovascular structure at higher risk during anterior retraction of the gluteus medius/vastus lateralis sleeve. 1- Lateral femoral cutaneous nerve 2- Lateral femoral circumflex artery 3- Superior gluteal nerve 4- Superior gluteal artery 5- Sciatic nerve 6- Femoral artery 7- Femoral vein 8- Femoral nerve 9- Saphenous branch of the femoral nerve 10- Profunda femoris artery
  • 33. AAOS 2016 Adult Reconstructive Surgery of the Hip and Knee 32 11- Inferior gluteal nerve PREFERRED RESPONSE: 3- Superior gluteal nerve Question 38 of 200 The direct anterior approach to the hip places this neurovascular structure at highest risk for injury in its superficial extent. 1- Lateral femoral cutaneous nerve 2- Lateral femoral circumflex artery 3- Superior gluteal nerve 4- Superior gluteal artery 5- Sciatic nerve 6- Femoral artery 7- Femoral vein 8- Femoral nerve 9- Saphenous branch of the femoral nerve 10- Profunda femoris artery 11- Inferior gluteal nerve PREFERRED RESPONSE: 1- Lateral femoral cutaneous nerve Question 39 of 200 This neurovascular structure limits distal extension of the direct anterior approach. 1- Lateral femoral cutaneous nerve 2- Lateral femoral circumflex artery 3- Superior gluteal nerve 4- Superior gluteal artery 5- Sciatic nerve 6- Femoral artery 7- Femoral vein
  • 34. AAOS 2016 Adult Reconstructive Surgery of the Hip and Knee 33 8- Femoral nerve 9- Saphenous branch of the femoral nerve 10- Profunda femoris artery 11- Inferior gluteal nerve PREFERRED RESPONSE: 8- Femoral nerve Question 40 of 200 CT and cadaveric studies have shown that anteroinferior placement of the anterior acetabular retractor results in less than 1 cm of distance between the retractor and which structure? 1- Lateral femoral cutaneous nerve 2- Lateral femoral circumflex artery 3- Superior gluteal nerve 4- Superior gluteal artery 5- Sciatic nerve 6- Femoral artery 7- Femoral vein 8- Femoral nerve 9- Saphenous branch of the femoral nerve 10- Profunda femoris artery 11- Inferior gluteal nerve PREFERRED RESPONSE: 8- Femoral nerve DISCUSSION Proximal dissection through the gluteus medius places the superior gluteal nerve at risk. The main branch of the superior gluteal nerve has been measured within the muscle to a distance of between 4.5 and 4.9 cm above the level of the superior acetabular rim, and safe dissection is assured if the surgeon does not exceed 4 cm above the acetabulum or 5 cm above the tip of the greater trochanter.
  • 35. AAOS 2016 Adult Reconstructive Surgery of the Hip and Knee 34 The lateral femoral cutaneous nerve courses medial to the anterior superior iliac spine and near the interval between the sartorious and tensor fascia muscle. Risk for injury can be decreased by remaining in the fascial sheath of the tensor and minimizing excessive retraction medially. The lateral femoral circumflex artery is ligated during this approach and is not at risk. Distal extension of the direct anterior approach beyond the intertrochanteric line poses risk for injury to the lateral and medial division of the femoral nerve, which innervates the anterolateral parts of the quadriceps muscle group. In addition, branches of the lateral circumflex artery are routinely ligated in a standard approach. A recent CT scan and cadaveric evaluation of acetabular retractor placement and its proximity to adjacent neurovascular structures demonstrates that the anterior retractor is furthest from the adjacent neurovascular bundle if placed more superiorly near the anterior superior iliac spine. The more inferiorly it is placed, the higher the risk to the adjacent structures, the most lateral of which is the femoral nerve. Question 41 of 200 A 58-year-old man has a painful right hip 3 years after undergoing a large head metal-on- metal total hip arthroplasty (THA) in which the components are well positioned. MR imaging confirms a cystic mass around the hip and metal ion levels show a marked increase in cobalt compared to chromium levels. The erythrocyte sedimentation rate (ESR) and C-reactive protein (CRP) level are within defined limits. What is the most likely cause for his discomfort? 1- Chronic periprosthetic infection 2- Trochanteric bursitis 3- Pseudotumor related to corrosion at the head/neck taper junction 4- Tendonitis from iliopsoas tendon impingement PREFERRED RESPONSE: 3- Pseudotumor related to corrosion at the head/neck taper junction DISCUSSION This patient presents with a pseudotumor likely attributable to local tissue reaction resulting from either articular metal wear debris and/or corrosion and fretting of the trunnion. The trunnion is a more likely source of the problem for a number of reasons: good position of metal articulation, increased trunnion corrosion and fretting associated with large-head THA, and markedly increased cobalt levels compared to chromium levels. Infection is very unlikely
  • 36. AAOS 2016 Adult Reconstructive Surgery of the Hip and Knee 35 in the setting of normal ESR and CRP findings. MR imaging findings are consistent with pseudotumor and not iliopsoas tendonitis or trochanteric bursitis. Figure 42a Figure 42b Figure 42c Figure 42d Figure 42e CLINICAL SITUATION FOR QUESTIONS 42 THROUGH 45 Figures 42a through 42e are the radiographs, MR image, and MR arthrogram of a 25-year- old collegiate soccer player who has new-onset left groin pain. He played competitive soccer from a young age and has either competed or practiced 5 to 6 times per week since the age of 10. He denies any specific hip injury that necessitated treatment, but his trainer contends that he had a groin pull. He has groin pain with passive flexion and internal rotation of his left hip, and his hip has less internal rotation than his asymptomatic right hip. He is otherwise healthy.
  • 37. AAOS 2016 Adult Reconstructive Surgery of the Hip and Knee 36 Question 42 of 200 Approximately which percentage of asymptomatic athletes have CAM deformities of the hip? 1- 5% 2- 10% 3- 25% 4- At least 50% PREFERRED RESPONSE: 4- At least 50% Question 43 of 200 What is the primary cause of a CAM deformity? 1- A genetic problem 2- Repetitive activities involving an open proximal femoral physis 3- Early closure of the proximal femoral physis 4- Hip dysplasia PREFERRED RESPONSE: 2- Repetitive activities involving an open proximal femoral physis Question 44 of 200 When counseling patients who have a CAM deformity, the orthopaedic surgeon should note that 1- osteoarthritis of the hip is likely to occur later in life. 2- correction prevents later development of osteoarthritis. 3- most acetabular tears are symptomatic, and surgical treatment will be necessary. 4- this is an inherited deformity.
  • 38. AAOS 2016 Adult Reconstructive Surgery of the Hip and Knee 37 PREFERRED RESPONSE: 1- osteoarthritis of the hip is likely to occur later in life. Question 45 of 200 Which factor is a contraindication to surgical treatment of a symptomatic CAM deformity? 1- Degenerative tear of the anterosuperior acetabular labrum 2- Superior hip joint space of 2 mm or less on radiographs 3- Ipsilateral knee instability 4- Lumbar spondylolisthesis PREFERRED RESPONSE: 2- Superior hip joint space of 2 mm or less on radiographs DISCUSSION Multiple studies have confirmed that CAM or pincer anatomy is commonly present in asymptomatic hips. According to a large systematic review, CAM deformities are present in approximately one-third of asymptomatic hips in young adults, and the proportion was higher than 50% in the subgroup of athletes. Ganz and associates proposed that femoral acetabular impingement is the root cause of osteoarthritis in the majority of nontraumatic, nondysplastic hips, and functional improvement with surgical correction of the deformity has been demonstrated. Despite the link between CAM deformity and hip osteoarthritis, a corresponding link between correction of the deformity and prevention of osteoarthritis has never been proven. Results of CAM deformity correction, typically including repair of the degenerative labral tear, are much poorer when there is significant joint space loss. A typical joint space cutoff of 2 mm or less is used to recommend against hip preservation surgery. Question 46 of 200 Patellofemoral arthroplasty is contraindicated in the presence of 1- moderate patellar tilt. 2- trochlear dysplasia. 3- inflammatory arthritis. 4- severe crepitus.
  • 39. AAOS 2016 Adult Reconstructive Surgery of the Hip and Knee 38 PREFERRED RESPONSE: 3- inflammatory arthritis. DISCUSSION Any inflammatory disease, including crystalline arthropathy, is a contraindication for patellofemoral arthroplasty. Patellar tilt and dysplasia frequently occur in the setting of patellofemoral arthritis and do not constitute a contraindication to this procedure. Severe crepitus is common and frequently addressed with patellofemoral arthroplasty. Figure 47 CLINICAL SITUATION FOR QUESTIONS 47 THROUGH 50 A 70-year-old woman has a 3-year history of gradually increasing diffuse and global right knee pain. Her main issues are difficulty with stairs, stiffness with prolonged sitting, and swelling. She has taken nonsteroidal anti-inflammatory medications and has received intra- articular steroid injections, all with decreasing efficacy. Her right knee examination reveals range of motion of 15 to 80 degrees with a fixed deformity to varus and valgus stress. Her symptoms are no longer manageable nonsurgically. Radiographs reveal a 30-degree mechanical axis deformity. Question 47 of 200 The deformity shown in Figure 47 is predominantly associated with
  • 40. AAOS 2016 Adult Reconstructive Surgery of the Hip and Knee 39 1- a hypoplastic lateral femoral condyle. 2- a contracted medial collateral ligament. 3- an excessive proximal tibial slope. 4- trochlear dysplasia. PREFERRED RESPONSE: 1- a hypoplastic lateral femoral condyle. Question 48 of 200 When using the measured resection technique during total knee arthroplasty (TKA), the best way to avoid femoral malrotation is to reference the 1- anteroposterior axis. 2- tibial intramedullary axis. 3- posterior condylar axis. 4- femoral intramedullary axis. PREFERRED RESPONSE: 1- anteroposterior axis. Question 49 of 200 When balancing gaps in the coronal plane, which structure preferentially impacts the flexion space more than the extension space? 1- Iliotibial band 2- Popliteus tendon 3- Lateral collateral ligament 4- Lateral head of the gastrocnemius PREFERRED RESPONSE: 2- Popliteus tendon Question 50 of 200 For this patient, which TKA design is most appropriate?
  • 41. AAOS 2016 Adult Reconstructive Surgery of the Hip and Knee 40 1- Bicruciate-retaining TKA 2- Unicompartmental arthroplasty 3- Hinged TKA 4- Posterior substitution TKA PREFERRED RESPONSE: 4- Posterior substitution TKA DISCUSSION TKA in the setting of valgus deformities poses different challenges than those encountered when varus deformities are present. Most valgus alignment is attributable to a deformity of the distal femur rather than the proximal tibia, as seen in varus knees. One of the major anatomical differences is a hypoplastic lateral femoral condyle which, when used as a rotational reference point, can lead to internal rotation of the femoral component if not recognized. This malrotation will in turn lead to patellofemoral maltracking or instability, which is a common complication associated with primary TKA. The deformity is too severe to consider a bicruciate-retaining TKA or unicompartmental arthroplasty and does not necessitate a hinged TKA. Question 51 of 200 When performing a cruciate-retaining total knee arthroplasty, trial components are inserted. The knee comes to full extension but is tight in flexion. The surgeon should consider 1- flexing the femoral component. 2- releasing the posterior cruciate ligament. 3- downsizing the tibial insert thickness. 4- resecting more distal femur. PREFERRED RESPONSE: 2- releasing the posterior cruciate ligament. DISCUSSION In this scenario, the flexion gap needs to be increased. Increase in flexion gap can be accomplished by downsizing the femoral component and increasing posterior tibial slope. In posterior cruciate-retaining TKA procedures, recession or release of the posterior cruciate ligament can loosen the flexion gap, allowing for an increase in flexion. Flexing the femoral
  • 42. AAOS 2016 Adult Reconstructive Surgery of the Hip and Knee 41 component tightens the flexion gap, and downsizing the tibial insert thickness decreases flexion and extension gaps, while resection of the distal femur only increases the extension gap. Figure 52 CLINICAL SITUATION FOR QUESTIONS 52 THROUGH 55 Figure 52 is the standing anteroposterior radiograph of a 55-year-old man who has a 5- year history of daily left knee pain with weight-bearing activities. He denies night pain or symptoms of instability. Upon examination his range of motion is 0 to 140 degrees. He has a mild, fully correctable varus deformity and a negative Lachman test result. He has failed nonsurgical treatment. Question 52 of 200 Based on a knee examination and radiographic findings, what is the most likely finding at the time of surgery? 1- Anteromedial osteoarthritis 2- An intact posterior cruciate ligament (PCL) and incompetent anterior cruciate ligament (ACL) 3- Incompetence of both the ACL and PCL 4- Posteromedial osteoarthritis
  • 43. AAOS 2016 Adult Reconstructive Surgery of the Hip and Knee 42 PREFERRED RESPONSE: 1- Anteromedial osteoarthritis Question 53 of 200 Unicompartmental knee arthroplasty (UKA) is discussed with the patient. The most appropriate next radiographic examination should be 1- MR imaging of the left knee to evaluate the lateral compartment. 2- a CT arthrogram to evaluate the status of the medial and lateral meniscus. 3- a stress radiograph to evaluate correction of the varus deformity. 4- a sunrise view to determine the status of the patellofemoral joint. PREFERRED RESPONSE: 3- a stress radiograph to evaluate correction of the varus deformity. Question 54 of 200 What is the UKA survivorship for a 55-year-old patient compared to survivorship for total knee arthroplasty? 1- Equal at 10 years 2- Lower at 10 years 3- Higher at 10 years 4- Not known when using a mobile-bearing UKA PREFERRED RESPONSE: 2- Lower at 10 years Question 55 of 200 The patient undergoes a mobile-bearing UKA. When compared to a fixed-bearing metal- backed unicompartmental arthroplasty, this procedure is associated with a 1- higher risk for failure attributable to wear. 2- higher risk for failure attributable to bearing spinout. 3- lower risk for loosening.
  • 44. AAOS 2016 Adult Reconstructive Surgery of the Hip and Knee 43 4- lower risk for arthritic progression of the lateral compartment. PREFERRED RESPONSE: 2- higher risk for failure attributable to bearing spinout. DISCUSSION A patient with medial compartment arthritis and a correctable varus deformity with no clinical or examination findings of knee instability most likely has an intact ACL. The pattern of medial compartment osteoarthritis most commonly associated with an intact ACL is that of anteromedial osteoarthritis. An incompetent ACL is commonly associated with a fixed varus deformity and radiographic signs of posteromedial wear. An incompetent ACL is a relative contraindication to a mobile-bearing UKA. When evaluating patients for a mobile-bearing UKA, a stress radiograph will allow the orthopaedic surgeon to determine the correction of the varus deformity and assess the lateral compartment. Inability to fully correct the deformity or narrowing of the lateral compartment with valgus stress should influence the surgeon against UKA. Joint registries across the world have shown decreased survivorship associated with TKA and UKA in men compared to other age groups, but survivorship is lower for UKA than TKA. No studies to date have shown differences in survivorship between fixed- or mobile-bearing UKAs. The complication that is unique to mobile-bearing UKA is bearing spinout, and this occurs in fewer than 1% of mobile-bearing UKA procedures. In vivo and in vitro polyethylene wear in mobile-bearing UKA is low. Arthritis progression may be faster for mobile-bearing UKAs than fixed-bearing UKAs.
  • 45. AAOS 2016 Adult Reconstructive Surgery of the Hip and Knee 44 Figure 56a Figure 56b Figure 56c Question 56 of 200 Figures 56a through 56c are the radiographs of a 65-year-old man with a 6-week history of severe left hip pain. He had a left total hip arthroplasty 19 years ago and a femoral revision 10 years ago. His erythrocyte sedimentation rate (ESR) and C-reactive protein (CRP) level are within defined limits. What is the most appropriate next step? 1- A trial of limited weight-bearing activity and physical therapy 2- A triple-phase bone scan to evaluate for loosening 3- Acetabular component revision 4- Femoral component revision PREFERRED RESPONSE: 4- Femoral component revision DISCUSSION
  • 46. AAOS 2016 Adult Reconstructive Surgery of the Hip and Knee 45 The radiographs reveal a fracture of the extensively porous coated stem. This entity, although rare, is associated with higher risk for occurrence when thin stems are implanted in patients with thick cortices and there is a lack of proximal stable support for the prosthesis. Nonsurgical care likely would not help this patient. A triple-phase bone scan would not add any information that would change the treatment plan. Question 57 of 200 At the time of revision knee arthroplasty, a surgeon performs a rectus snip to gain exposure to the knee. When compared to a standard parapatellar approach, what is the expected outcome? 1- Improvement in range of motion 2- Reduction in range of motion 3- Increase in extensor mechanism lag 4- No differences in motion and strength PREFERRED RESPONSE: 4- No differences in motion and strength DISCUSSION Rectus snip during total knee arthroplasty has no effect on motion or strength at long-term follow-up. It has not been associated with extensor mechanism lag.
  • 47. AAOS 2016 Adult Reconstructive Surgery of the Hip and Knee 46 Figure 58 Figure 59a Figure 59b
  • 48. AAOS 2016 Adult Reconstructive Surgery of the Hip and Knee 47 Figure 60a Figure 60b Figure 60c Figure 61 Figure 62a Figure 62b
  • 49. AAOS 2016 Adult Reconstructive Surgery of the Hip and Knee 48 RESPONSES FOR QUESTIONS 58 THROUGH 62 1- Nerve palsy 2- Skin necrosis 3- Flexion instability 4- Patellar instability 5- Anterior knee pain 6- Malalignment Total knee arthroplasty (TKA) is performed to address each condition shown in Figures 58 through 62b. Which complication is most commonly associated with each image? Question 58 of 200 Figure 58 1- Nerve palsy 2- Skin necrosis 3- Flexion instability 4- Patellar instability 5- Anterior knee pain 6- Malalignment PREFERRED RESPONSE: 1- Nerve palsy Question 59 of 200 Figure 59a and Figure 59b 1- Nerve palsy 2- Skin necrosis 3- Flexion instability 4- Patellar instability 5- Anterior knee pain 6- Malalignment
  • 50. AAOS 2016 Adult Reconstructive Surgery of the Hip and Knee 49 PREFERRED RESPONSE: 4- Patellar instability Question 60 of 200 Figure 60a through Figure 60c 1- Nerve palsy 2- Skin necrosis 3- Flexion instability 4- Patellar instability 5- Anterior knee pain 6- Malalignment PREFERRED RESPONSE: 2- Skin necrosis Question 61 of 200 Figure 61 1- Nerve palsy 2- Skin necrosis 3- Flexion instability 4- Patellar instability 5- Anterior knee pain 6- Malalignment PREFERRED RESPONSE: 5- Anterior knee pain Question 62 of 200 Figure 62a and Figure 62b 1- Nerve palsy 2- Skin necrosis 3- Flexion instability 4- Patellar instability
  • 51. AAOS 2016 Adult Reconstructive Surgery of the Hip and Knee 50 5- Anterior knee pain 6- Malalignment PREFERRED RESPONSE: 6- Malalignment DISCUSSION Figure 58 reveals a posttraumatic valgus deformity. Correction of valgus with lateral soft- tissue release places tension on the peroneal nerve, resulting in an increased risk for nerve palsy. Figures 59a (lateral view) and 59b (Merchant view) illustrate juvenile rheumatoid arthritis with tibiofibular fusion and lateral patellar dislocation. Chronic patellar dislocation is associated with contracture of the lateral retinacular soft tissues and increased risk for patellar subluxation or dislocation after TKA. Extensor mechanism realignment, possibly including tibial tubercle osteotomy and/or proximal soft-tissue realignment, may be required during TKA to centralize the extensor mechanism. Figures 60a (anteroposterior [AP] view) and 60b (lateral view) reveal a fused knee in full extension. TKA after fusion is associated with multiple complications including skin necrosis, infection, and instability. The skin is contracted because of limited knee motion and has multiple scars (Figure 60c). Mobilization of the skin during and after knee arthroplasty can place excess tension on the soft tissues, resulting in skin necrosis and infection. Treatment consisting of prompt debridement and soft-tissue coverage, usually with medial gastrocnemius muscle transposition, is required. Figure 61 shows a knee with prior tibial tubercle fixation and marked patella infera. Shortening of the patellar ligament is associated with restricted knee motion. This may necessitate more extensile exposure using tibial tubercle osteotomy or rectus snip during TKA to obtain adequate surgical exposure. The inferior position of the patella can cause impingement between the patellar component and tibial insert, resulting in anterior knee pain. Restoring a more normal position of the patella may necessitate distal positioning of the femoral component as well as tibial tubercle osteotomy with proximal recession of the osteotomized tibial tubercle. Figures 62a (AP view of the distal femur) and 62b (AP view of the proximal femur) show a posttraumatic deformity with a large retained intramedullary rod. There is a varus distal femoral deformity that is not severe enough to necessitate extra-articular corrective osteotomy. However, intramedullary hardware precludes use of conventional intramedullary instrumentation, so computer navigation or patient-specific cutting guides will be necessary to orient the bone cuts and avoid implant malalignment. Question 63 of 200 Injury to the popliteal artery during total knee arthroplasty (TKA) is most likely when placing a sharp retractor 1- directly posterior to the posterior cruciate ligament (PCL).
  • 52. AAOS 2016 Adult Reconstructive Surgery of the Hip and Knee 51 2- posteromedial to the PCL. 3- posterolateral to the PCL. 4- in the posteromedial corner of the knee. PREFERRED RESPONSE: 3- posterolateral to the PCL. DISCUSSION Vascular complications during TKA are rare but do occur. Traditionally, it was taught that the popliteal artery was situated posterior to the PCL; however, more recent anatomic dissections have demonstrated that this artery is usually posterolateral to the PCL. Question 64 of 200 A 30-year-old patient is indicated for distal femoral osteotomy. This procedure results in survivorship with 1- a functional result for at least 20 years. 2- a functional result that deteriorates within the first 10 years. 3- an eventual conversion to a constrained knee arthroplasty. 4- an eventual need for arthrodesis. PREFERRED RESPONSE: 2- a functional result that deteriorates within the first 10 years. DISCUSSION Distal femoral varus osteotomy (DFVO) is intended for patients younger than age 50, more active patients with isolated lateral compartment arthritis and valgus malalignment. Although the knee functional score improves at 1-year follow-up, the function scores significantly deteriorate at 10-year follow-up. At 15-year follow-up, the knee function further declines, resulting in an overall failure rate of 48.5%. DFVO provides longer lasting benefit in patients with better presurgical knee function. Total knee arthroplasty following DFVO provides improved function and successful outcomes. Standard posterior stabilized components provide satisfactory stability after appropriate ligament balancing without the need for stemmed or highly constrained implants for most patients.
  • 53. AAOS 2016 Adult Reconstructive Surgery of the Hip and Knee 52 Figure 65a Figure 65b CLINICAL SITUATION FOR QUESTIONS 65 THROUGH 67 Figures 65a and 65b are the radiographs of an 80-year-old woman with long-standing knee pain who has failed nonsurgical treatment that includes a structured physical therapy program, nonsteroidal anti-inflammatory medications, and intra-articular steroid injections. Her radiographs reveal significant degenerative changes. She has experienced some cognitive decline during the last several years and poorly tolerates pain medications, but she remains very active. Question 65 of 200 Cognitive function following total knee arthroplasty (TKA) among elderly patients who receive either neuraxial or general anesthesia is 1- less affected with neuraxial anesthesia. 2- less affected with general anesthesia. 3- unchanged with either anesthesia type. 4- generally worsened with both neuraxial and general anesthesia.
  • 54. AAOS 2016 Adult Reconstructive Surgery of the Hip and Knee 53 PREFERRED RESPONSE: 1- less affected with neuraxial anesthesia. Question 66 of 200 To reduce the patient’s use of narcotics after surgery, which anesthetic regimen would be most appropriate? 1- Spinal anesthesia with a peripheral nerve block 2- Spinal anesthesia without a peripheral nerve block 3- General anesthesia without a peripheral nerve block 4- General anesthesia with a long-acting mu opioid agonist PREFERRED RESPONSE: 1- Spinal anesthesia with a peripheral nerve block Question 67 of 200 Which treatment is associated with decreased complications related to femoral nerve blocks for TKA? 1- Knee immobilizer 2- Compression stockings 3- Tranexamic acid 4- Celecoxib PREFERRED RESPONSE: 1- Knee immobilizer DISCUSSION TKA among elderly patients can be problematic, considering their potential for complex comorbidities including diminished cognitive function. As patients age, their tolerance for certain medications diminishes. Regional anesthesia is an important adjunct to a multimodal pain program, which can reduce narcotic pain medication use and improve cognitive function through less reliance on systemic medications. With regional pain management such as femoral nerve blocks comes potential for an increase in complications such as falls. Femoral nerve blocks improve pain but also have a large impact on quadriceps and motor function, which places patients at higher risk for falls.
  • 55. AAOS 2016 Adult Reconstructive Surgery of the Hip and Knee 54 Question 68 of 200 A 47-year-old obese man with a body mass index of 42 comes in with left knee pain 1 year after undergoing an uncomplicated left medial unicompartmental knee arthroplasty (UKA). Radiographs show a loose tibial component in varus. What is the most appropriate next step to treat this failed construct? 1- Aspiration of joint fluid to obtain a cell count 2- Revision of the UKA using primary total knee arthroplasty (TKA) components 3- Revision of the UKA using a revision TKA with augments 4- Obtain erythrocyte sedimentation rate (ESR) and C-reactive protein (CRP) levels PREFERRED RESPONSE: 4- Obtain erythrocyte sedimentation rate (ESR) and C- reactive protein (CRP) levels DISCUSSION This patient likely is experiencing failure of his UKA secondary to poor patient selection. This young, heavy man likely loosened his component secondary to the ongoing varus alignment of the knee and his elevated weight. Despite this likely scenario, the next step is to determine if an infection is the cause of his pain. Prior to obtaining an aspiration, the surgeon can order an ESR and CRP to determine if aspiration is warranted. If laboratory studies are unremarkable, the surgeon likely can forgo the aspiration and proceed to a revision TKA with possible augments on standby.
  • 56. AAOS 2016 Adult Reconstructive Surgery of the Hip and Knee 55 Figure 69a Figure 69b Figure 69c Figure 70a
  • 57. AAOS 2016 Adult Reconstructive Surgery of the Hip and Knee 56 Figure 70b Figure 70c Figure 70d CLINICAL SITUATION FOR QUESTIONS 69 THROUGH 72 Figures 69a through 69c are the radiograph and MR images of a 37-year-old woman who has had a 2-month history of left hip pain. Question 69 of 200 Which single symptom and examination finding combination is most likely in this scenario? 1- Sitting pain with flexion abduction and external rotation of the hip
  • 58. AAOS 2016 Adult Reconstructive Surgery of the Hip and Knee 57 2- Groin pain and pain with internal rotation and adduction while supine with the hip and knee flexed 90 degrees 3- Clicking and abductor lurch 4- Buttock pain and pain with hip extension, adduction, and external rotation while prone PREFERRED RESPONSE: 2- Groin pain and pain with internal rotation and adduction while supine with the hip and knee flexed 90 degrees Question 70 of 200 Measurements have been taken on the radiographs shown in Figures 70a through 70d. Which measurement is abnormal? 1- Head center position 2- Tonnis angle 3- Lateral center edge angle 4- Acetabular inclination PREFERRED RESPONSE: 3- Lateral center edge angle Question 71 of 200 Based on the patient’s continued pain, her imitations, the previously noted radiographic findings, and failure of physical therapy and nonsteroidal anti-inflammatory treatment, what is the best surgical option? 1- Arthroscopic labral repair 2- Arthroscopy with pincer resection 3- Periacetabular osteotomy (PAO) 4- Hip arthroscopy and PAO PREFERRED RESPONSE: 1- Arthroscopic labral repair Question 72 of 200
  • 59. AAOS 2016 Adult Reconstructive Surgery of the Hip and Knee 58 Which presurgical condition is most commonly associated with a poor outcome after a hip joint salvage procedure? 1- Age older than 40 2- Body mass index higher than 30 3- Tonnis grade of 2 or higher 4- Outerbridge grade of III or IV PREFERRED RESPONSE: 3- Tonnis grade of 2 or higher DISCUSSION MR imaging reveals an anterior labral tear, and the radiograph shows minimal arthritis with possible dysplasia. The most common pain location among patients with a labral tear is the groin, and the most common physical finding is a positive impingement test result. Pain with sitting, clicking, and buttock pain are frequently described by patients with a labral tear, but these symptoms are less common than groin pain. A positive posterior impingement test finding is more common in patients with a posterior labral tear. The lateral center edge angle is pictured in Figure 70a. This angle is considered abnormal when it is less than 25 degrees, which may indicate inadequate head coverage. The Tonnis angle and acetabular inclination are different terms for the same angle as seen in Figure 70b. The normal value is between 0 and 10 degrees. The head center position is the distance from the medial aspect of the femoral head to the ilioischial line, as measured in Figure 70c. The head is considered lateralized if the measurement exceeds 10 mm. Figure 70d marks the outline of the anterior and posterior wall, confirming the absence of a cross-over sign. A cross-over sign is present when the 2 lines cross below the lateral aspect of the acetabulum, and it indicates acetabular retroversion. An arthroscopy to treat the labral tear is probably the best option for this patient. A PAO with prior hip arthroscopy might be considered based on the mildly increased anterior center edge angle; the Tonnis angle finding is normal, the femoral head is not lateralized, and the patient is approaching age 40. A PAO without addressing the anterior labral tear could lead to failure and is not the best choice. The patient does not have an acetabular pincer lesion and would not benefit from an acetabular edge resection. Although age beyond 40 years and body mass index higher than 30 can adversely affect clinical outcomes after joint preservation procedures (PAO, hip arthroscopy, and femoral acetabular impingement surgery), the presence of hip arthritis on presurgical radiographs is the most commonly mentioned cause of failed hip joint preservation surgery. Tonnis grade is a radiographic measure of hip arthritis. A higher Outerbridge score is associated with more frequent poor outcomes after hip arthroscopy; however, the Outerbridge cartilage score is
  • 60. AAOS 2016 Adult Reconstructive Surgery of the Hip and Knee 59 determined by direct visualization at the time of surgery. An Outerbridge score cannot be determined presurgically. Question 73 of 200 Clinical studies on the use of topical and intravenous (IV) forms of tranexamic acid (TXA) administration demonstrate which results? 1- IV administration of TXA is substantially more efficacious in minimizing blood loss than topical administration. 2- IV administration of TXA places high-risk patients such as those with coronary stents at an unacceptable risk for a cardiac event during the perioperative period. 3- IV administration of TXA decreases intrasurgical blood loss but has not been shown to decrease postsurgical transfusion rates. 4- Both IV and topical administration of TXA decrease intrasurgical blood loss and postsurgical transfusion rates. PREFERRED RESPONSE: 4- Both IV and topical administration of TXA decrease intrasurgical blood loss and postsurgical transfusion rates. DISCUSSION Numerous studies have demonstrated efficacy of both IV and topical administration of TXA for decreasing blood loss and transfusion rates. Several studies have shown no significant difference between TXA IV and topical administration in decreasing blood loss or lowering transfusion rates. Inconclusive evidence shows that IV administration of TXA places individuals at higher risk for a thromboembolic event. Both IV and topical TXA are equally effective in decreasing blood loss and minimizing transfusion rates.
  • 61. AAOS 2016 Adult Reconstructive Surgery of the Hip and Knee 60 Figure 74a Figure 74b Figure 74c Question 74 of 200 Figures 74a through 74c are the postsurgical radiographs of a 74-year-old man who has stiffness in his right knee 8 weeks after undergoing elective right total knee arthroplasty (TKA). The surgery was performed for primary varus osteoarthritis and was uncomplicated. His range of motion is 5 to 80 degrees. What is the most appropriate next treatment step? 1- Manipulation under anesthesia (MUA) 2- Arthroscopic lysis of adhesions 3- Open arthrolysis of adhesions 4- Revision TKA PREFERRED RESPONSE: 1- Manipulation under anesthesia (MUA) DISCUSSION Stiffness following TKA is a disabling complication. One option involves MUA, a valuable technique with which to increase range of motion after TKA for patients with stiff knees. A systematic review of the literature was performed to identify studies that reported the clinical outcomes and measured range of motion for patients undergoing MUA. Fourteen studies (913 patients) reported range of motion results following MUA at up to 10-year follow- up. The review demonstrated that MUA for a stiff primary TKA is an efficacious procedure to restore range of motion, and it carries a low complication rate. Early gains in motion were
  • 62. AAOS 2016 Adult Reconstructive Surgery of the Hip and Knee 61 reported to be maintained in the long term. A second review systematically evaluated the outcomes of 4 treatments for arthrofibrosis that develops subsequent to TKA (MUA, arthroscopic debridement, open surgical release, and revision TKA). This study showed that there were no significant differences in the Knee Society Score of the 4 treatment modalities. Although open surgical release resulted in the greatest increase in range of motion, there were methodological study limitations because the majority of the papers were case series, which decreased the quality of the evidence. Response 1 is correct because the TKA appears appropriately sized and is well aligned. Responses 2 and 3 are incorrect, considering the early time frame (8 weeks) from initial surgery. Although the patella is unresurfaced in the radiographs, there is no indication for revision TKA (even secondary patellar resurfacing) at this early juncture. Question 75 of 200 During total hip arthroplasty, which characteristic of irradiated (10 Mrad) and subsequently melted highly cross-linked polyethylene should provide a more wear-resistant construct than traditional gamma-irradiated (2.5-4 Mrad)-in-air polyethylene mated with the same head? 1- Resistance to adhesive wear 2- Resistance to abrasive wear 3- Resistance to fatigue wear 4- Resistance to creep PREFERRED RESPONSE: 1- Resistance to adhesive wear DISCUSSION Highly cross-linked polyethylene makes material resistant to adhesive wear. Abrasive wear from third bodies does not decrease wear. The fatigue strength of this material is inferior to traditional polyethylene, and its resistance to creep is the same, if not lower, than that of traditional polyethylene.
  • 63. AAOS 2016 Adult Reconstructive Surgery of the Hip and Knee 62 Figure 76a Figure 76b Figure 77a Figure 77b
  • 64. AAOS 2016 Adult Reconstructive Surgery of the Hip and Knee 63 Figure 78a Figure 78b Figure 79a Figure 79b
  • 65. AAOS 2016 Adult Reconstructive Surgery of the Hip and Knee 64 Figure 80a Figure 80b RESPONSES FOR QUESTIONS 76 THROUGH 80 1- Cemented full revision 2- Cemented femoral revision 3- Cemented acetabular revision 4- Cementless full revision 5- Cementless femoral revision without an extended trochanteric osteotomy 6- Cementless femoral revision with an extended trochanteric osteotomy 7- Cementless acetabular revision 8- Isolated liner/head exchange 9- Isolated liner/head exchange with cementing of the liner Select the most appropriate treatment for each clinical scenario. Question 76 of 200 A 72-year-old man with a history of prior revision for a loose femoral component (Figures 76a and 76b) 1- Cemented full revision 2- Cemented femoral revision
  • 66. AAOS 2016 Adult Reconstructive Surgery of the Hip and Knee 65 3- Cemented acetabular revision 4- Cementless full revision 5- Cementless femoral revision without an extended trochanteric osteotomy 6- Cementless femoral revision with an extended trochanteric osteotomy 7- Cementless acetabular revision 8- Isolated liner/head exchange 9- Isolated liner/head exchange with cementing of the liner PREFERRED RESPONSE: 6- Cementless femoral revision with an extended trochanteric osteotomy Question 77 of 200 A 65-year-old woman with intraoperative findings of well-fixed components and a deficient liner locking mechanism (Figures 77a and 77b) 1- Cemented full revision 2- Cemented femoral revision 3- Cemented acetabular revision 4- Cementless full revision 5- Cementless femoral revision without an extended trochanteric osteotomy 6- Cementless femoral revision with an extended trochanteric osteotomy 7- Cementless acetabular revision 8- Isolated liner/head exchange 9- Isolated liner/head exchange with cementing of the liner PREFERRED RESPONSE: 7- Cementless acetabular revision Question 78 of 200 A 68-year-old woman with a history of thigh pain on startup since her primary total hip arthroplasty (THA) 1 year ago (Figures 78a and 78b) 1- Cemented full revision
  • 67. AAOS 2016 Adult Reconstructive Surgery of the Hip and Knee 66 2- Cemented femoral revision 3- Cemented acetabular revision 4- Cementless full revision 5- Cementless femoral revision without an extended trochanteric osteotomy 6- Cementless femoral revision with an extended trochanteric osteotomy 7- Cementless acetabular revision 8- Isolated liner/head exchange 9- Isolated liner/head exchange with cementing of the liner PREFERRED RESPONSE: 5- Cementless femoral revision without an extended trochanteric osteotomy Question 79 of 200 A 54-year-old woman who had a THA 18 years ago and has mild pain with activities (Figures 79a and 79b) 1- Cemented full revision 2- Cemented femoral revision 3- Cemented acetabular revision 4- Cementless full revision 5- Cementless femoral revision without an extended trochanteric osteotomy 6- Cementless femoral revision with an extended trochanteric osteotomy 7- Cementless acetabular revision 8- Isolated liner/head exchange 9- Isolated liner/head exchange with cementing of the liner PREFERRED RESPONSE: 8- Isolated liner/head exchange Question 80 of 200 An 82-year-old woman who had a THA 14 years ago and has difficulty bearing weight (Figures 80a and 80b)
  • 68. AAOS 2016 Adult Reconstructive Surgery of the Hip and Knee 67 1- Cemented full revision 2- Cemented femoral revision 3- Cemented acetabular revision 4- Cementless full revision 5- Cementless femoral revision without an extended trochanteric osteotomy 6- Cementless femoral revision with an extended trochanteric osteotomy 7- Cementless acetabular revision 8- Isolated liner/head exchange 9- Isolated liner/head exchange with cementing of the liner PREFERRED RESPONSE: 6- Cementless femoral revision with an extended trochanteric osteotomy DISCUSSION Figures 76a and 76b show likely undersizing of the stem with subsidence. Revision, with possibly a modular fluted stem, is indicated. Extended trochanteric osteotomy is indicated to avoid trochanteric fracture and to facilitate fully coated stem removal, especially if trephines are needed. It is unlikely cemented fixation after multiple revisions would provide reliable long-term survivorship. Figures 77a and 77b show an older-style cementless acetabular component with severe liner wear and extensive osteolysis. The well-fixed acetabular component with poor component position and a poor locking mechanism is a good indication for cementless acetabular revision. Figures 78a and 78b reveal likely subsidence and lack of ongrowth of the femoral component. Cementless femoral revision offers the best chance for long-term survivorship. An osteotomy likely is not necessary for removal or implantation. Figures 79a and 79b reveal severe polyethylene wear with well-fixed acetabular and femoral components. This is a good indication for a head and liner exchange. Figures 80a and 80b show a loose cemented stem. Cementless fixation offers good long- term outcomes in this case. An osteotomy is likely necessary in this case because of the varus remodeling of the proximal femur, which would increase the chance of a greater trochanteric fracture with extraction of the existing stem and implantation of the revision stem. Question 81 of 200
  • 69. AAOS 2016 Adult Reconstructive Surgery of the Hip and Knee 68 Which modality has the broadest application for reduction of postsurgical transfusion? 1- Regional anesthesia 2- Tranexamic (TXA) acid administration 3- Reduced transfusion trigger 4- Hypotensive anesthesia PREFERRED RESPONSE: 2- Tranexamic (TXA) acid administration DISCUSSION TXA is easy to administer, inexpensive, and safe for virtually all patients. Multiple studies have demonstrated transfusion rates lower than 3% for total knee arthroplasty and lower than 10% for total hip arthroplasty. Regional and hypotensive anesthesia effectively reduce transfusion; however, they cannot be used in as wide a range of patients as TXA. A reduced transfusion trigger must be considered along with patient symptoms when determining the need for transfusion. Figure 82 Question 82 of 200
  • 70. AAOS 2016 Adult Reconstructive Surgery of the Hip and Knee 69 Which wear mechanism is most likely responsible for the wear damage on the modular tibial insert retrieval shown in Figure 82? 1- Adhesive 2- Abrasive 3- Fatigue 4- Creep PREFERRED RESPONSE: 3- Fatigue DISCUSSION The figure shows the top side of a retrieved tibial liner. Pitting and delamination, which are associated with fatigue wear, are noted. Creep is deformation without wear. Adhesive and abrasive wear is associated with removal of material on the back side of modular tibial components. CLINICAL SITUATION FOR QUESTIONS 83 THROUGH 87 A bilateral cemented total knee arthroplasty (TKA) was performed on an otherwise healthy 63-year-old woman. The surgery and immediate postsurgical course were uneventful. Two days after surgery, while in physical therapy at the hospital, the patient’s oxygen saturation is noted at 92%. Question 83 of 200 What is the most reasonable next step? 1- Multidetector CT scan 2- Ventilation-perfusion scan 3- Reassess on supplemental oxygen 4- Anteroposterior and lateral chest radiographs PREFERRED RESPONSE: 3- Reassess on supplemental oxygen
  • 71. AAOS 2016 Adult Reconstructive Surgery of the Hip and Knee 70 Question 84 of 200 What is the incidence of pulmonary embolism after bilateral TKA under a single anesthetic? 1- Lower than 0.5% 2- Between 0.5% and 1.5% 3- Between 5% and 10% 4- Higher than 10% PREFERRED RESPONSE: 2- Between 0.5% and 1.5% Question 85 of 200 The use of pharmacologic agents and/or mechanical compressive devices for the prevention of venous thromboembolism in patients undergoing elective hip or knee arthroplasty who are not at elevated risk beyond risk associated with the surgery for venous thromboembolism or bleeding was assigned which grade of recommendation by the 2011 AAOS Clinical Practice Guideline, Preventing Venous Thromboembolic Disease in Patients Undergoing Elective Hip and Knee Arthroplasty? 1-- Strong 2- Moderate 3- Limited 4- Inconclusive PREFERRED RESPONSE: 2- Moderate Question 86 of 200 The patient knows that TXA was used as part of the blood conservation strategy for her surgery. She asks you if TXA increases risk for pulmonary embolism. What is the most appropriate response? 1- TXA increases risk for deep venous thrombosis but not pulmonary embolism after TKA. 2- TXA does not increase risk for pulmonary embolism after TKA. 3- TXA increases risk for pulmonary embolism, but the advantages outweigh the risks. 4- The risk for pulmonary embolism after TKA involving TXA has never been studied.
  • 72. AAOS 2016 Adult Reconstructive Surgery of the Hip and Knee 71 PREFERRED RESPONSE: 2- TXA does not increase risk for pulmonary embolism after TKA. Question 87 of 200 Which gene mutation or polymorphism has been shown to most increase the risk for venous thromboembolic disease after elective total joint arthroplasty? 1- Factor V Leiden 2- Prothrombin G20210A 3- MTHFR/C677T/TT 4- Lupus anticoagulant PREFERRED RESPONSE: 3- MTHFR/C677T/TT DISCUSSION Simultaneous bilateral TKA accounts for approximately 6% of the TKAs performed in the United States and is more frequently performed for women. The incidence of pulmonary embolism in this group was between 0.57 and 1.14, according to a 1999 to 2008 registry-based study in the United States. There was not a significant change in incidence during that period. Hypoxemia alone is not an indication for advanced testing for pulmonary embolism. Winters and associates demonstrated that to avoid unnecessary testing, the use of a hypoxia algorithm is a reasonable first step. The use of pharmacologic agents and/or mechanical compressive devices for the prevention of venous thromboembolism in patients undergoing elective hip or knee arthroplasty and who are not at elevated risk beyond that of the surgery itself for venous thromboembolism or bleeding was given a Moderate grade of recommendation in the 2011 AAOS Clinical Practice Guideline, Preventing Venous Thromboembolic Disease in Patients Undergoing Elective Hip and Knee Arthroplasty. Various genetic factors are associated with increased risk for venous thromboembolic disease after TKA. A recent meta-analysis evaluated the genetic and polymorphism profiles associated with venous thromboembolism after arthroplasty. The mutation MTHFR/C677T/TT carried the highest risk (OR 2.36; 95% CI, 1.03-5.42, P = 0.04) for the gene mutations and polymorphisms studied. With the increased use of TXA as a blood-conservation strategy for total joint arthroplasty, there is a theoretical concern about an increased risk for venous thromboembolic disease. A recent study by Duncan and associates included 13,262 elective total joint arthroplasty procedures and demonstrated that TXA does not increase the risk of venous thromboembolism.
  • 73. AAOS 2016 Adult Reconstructive Surgery of the Hip and Knee 72 Figure 88 Question 88 of 200 Figure 88 is the radiograph of a 68-year-old man who fell 3 weeks after undergoing a successful left primary total hip arthroplasty. He is experiencing a substantial increase in pain and an inability to bear weight. What is an appropriate treatment plan? 1- Open reduction and internal fixation (ORIF) of the fracture 2- Remove the current stem, femur ORIF, and insertion of a longer revision stem 3- Femur ORIF with cables and strut graft, leaving the current stem in situ 4- Femur ORIF combined reimplantation of the primary component
  • 74. AAOS 2016 Adult Reconstructive Surgery of the Hip and Knee 73 PREFERRED RESPONSE: 2- Remove the current stem, femur ORIF, and insertion of a longer revision stem DISCUSSION Based on the fact that the fracture is occurring around the stem (type B) and the stem is clearly loose (type B2), the appropriate treatment is removal of the in situ stem (which is loose), ORIF of the femur (cerclage wires, cables, or a plate would be acceptable), and insertion of a longer revision stem (a tapered fluted modular titanium or fully porous coated cylindrical stem) to bypass the fracture. All other responses are incorrect because they provide inappropriate treatment options for a Vancouver B2 fracture. Question 89 of 200 A patient has a painful metal-on-metal (MOM) left total hip arthroplasty (THA). Which test(s) best correlate with prognosis if this patient is having a reaction to metal debris? 1- Erythrocyte sedimentation rate, C-reactive protein, and white blood cell count 2- Serum cobalt and chromium ion levels 3- Metal artifact reduction sequence (MARS) MRI 4- CT scan of pelvis PREFERRED RESPONSE: 3- Metal artifact reduction sequence (MARS) MRI DISCUSSION Painful MOM THA and taper corrosion can cause substantial damage to a patient's hip if left untreated. In this case, the workup for a painful MOM THA starts the same as a workup for a painful metal-on-polyethylene bearing couple. Infection must be ruled out in every case with a set of inflammatory markers. If these markers are remotely elevated, this is an indication for joint aspiration. In patients with metal debris, the pathology report often indicates too many cells to count or cellular debris. Metal ion levels do not seem to correlate with prognosis. There are well-functioning patients with high ion levels and poor-functioning patients with low ion levels. Advanced imaging with MARS MRI to evaluate for peritrochanteric fluid collection, a soft-tissue mass, or synovial/capsular hypertrophy will reveal signs of a metal reaction that indicate the need for a revision discussion. A CT scan can show more advanced bony destruction as an indicator of poor prognosis. These films can be used to determine the need for a structural graft or augments for reconstruction of bone loss attributable to metal debris.
  • 75. AAOS 2016 Adult Reconstructive Surgery of the Hip and Knee 74 Question 90 of 200 A 58-year-old man with insulin-dependent diabetes mellitus underwent primary total knee arthroplasty (TKA) and developed a full-thickness skin slough 3 cm x 4 cm with postsurgical exposure of the patella tendon. There is no change in the appearance of the wound after 2 weeks of wet-to-dry dressing changes. What is the best next treatment step for the soft-tissue defect? 1- Continue dressing changes 2- Split-thickness skin graft 3- Full-thickness skin graft 4- Local rotational flap PREFERRED RESPONSE: 4- Local rotational flap DISCUSSION Following TKA, if wound healing does not occur and deep soft tissues such as the patella tendon are exposed, local rotational flaps are the procedure of choice and should be performed relatively early following the recognition of a soft-tissue wound-healing problem. Fortunately, in the setting of TKA, gastrocnemius flaps are an excellent source for wound coverage of the proximal tibia. RESPONSES FOR QUESTIONS 91 THROUGH 94 1- Isolated head and liner revision 2- 2-stage exchange 3- Acute hemodialysis 4- Femoral component revision 5- Acetabular component revision 6- Revision total hip (all components) Match the most appropriate treatment listed above with each clinical scenario described below. Question 91 of 200 A 48-year-old has had 6 weeks of groin pain and squeaking with motion of the hip joint 2 years after undergoing total hip arthroplasty (THA) with a ceramic-on-ceramic bearing surface
  • 76. AAOS 2016 Adult Reconstructive Surgery of the Hip and Knee 75 using a modular acetabular component. Imaging of the hip shows an asymmetric position of the femoral head within the acetabular component. Erythrocyte sedimentation rate (ESR) and C-reactive protein (CRP) levels are within normal limits. Implants are in appropriate position and not substantially damaged. 1- Isolated head and liner revision 2- 2-stage exchange 3- Acute hemodialysis 4- Femoral component revision 5- Acetabular component revision 6- Revision total hip (all components) PREFERRED RESPONSE: 1- Isolated head and liner revision Question 92 of 200 A 55-year-old man has groin pain and a limp with Trendelenburg gait 5 years after undergoing THA with a metal-on-metal bearing surface and a modular acetabular component. Serum cobalt and chromium ion levels are 55 ppb and 42 ppb, respectively. Acetabular inclination measures 60 degrees. MR images reveal fluid collection and soft-tissue changes. 1- Isolated head and liner revision 2- 2-stage exchange 3- Acute hemodialysis 4- Femoral component revision 5- Acetabular component revision 6- Revision total hip (all components) PREFERRED RESPONSE: 5- Acetabular component revision Question 93 of 200 A 60-year-old has had 6 months of groin pain and a limp with Trendelenburg gait 5 years after undergoing THA with a metal-on-polyethylene bearing surface. Serum cobalt and chromium ion levels are 7.8 ppb and 1.1 ppb, respectively. Cross-sectional imaging shows a 3-cm pseudotumor.
  • 77. AAOS 2016 Adult Reconstructive Surgery of the Hip and Knee 76 1- Isolated head and liner revision 2- 2-stage exchange 3- Acute hemodialysis 4- Femoral component revision 5- Acetabular component revision 6- Revision total hip (all components) PREFERRED RESPONSE: 1- Isolated head and liner revision Question 94 of 200 A 70-year-old man has a posterior dislocation 20 years after undergoing cementless THA with a metal-on-polyethylene bearing. Acetabular inclination is 55 degrees with neutral version. This is his third dislocation, and he has been treated with closed reduction 3 times during the past month. His ESR is 42 mm/h (reference range [rr], 0-20 mm/h) and CRP level is 16.2 mg/L (rr, 0.08-3.1 mg/L). Joint aspiration reveals a cell count of 865 cells (55% neutrophils). 1- Isolated head and liner revision 2- 2-stage exchange 3- Acute hemodialysis 4- Femoral component revision 5- Acetabular component revision 6- Revision total hip (all components) PREFERRED RESPONSE: 5- Acetabular component revision DISCUSSION Treatment options for various pathologic conditions after THA can be challenging, and there are often multiple potential options. Question 91 involves a patient who has fractured a modular ceramic acetabular liner, and an isolated head and liner exchange should be sufficient. Question 92 involves a patient with markedly elevated serum metal ions and abductor dysfunction, suggesting poor bearing function and a probable adverse local tissue reaction (ALTR). Additionally, the acetabular component position is suboptimal, and complete
  • 78. AAOS 2016 Adult Reconstructive Surgery of the Hip and Knee 77 acetabular revision would be appropriate. Question 93 involves a patient with elevated serum metal ions, with cobalt disproportionately higher than chromium in a roughly 7:1 ratio, suggesting corrosive changes at the prosthetic femoral neck (trunnionosis). Cross-sectional imaging continues to be a key part of the evaluation and treatment of patients with metal reactions. This patient’s pain and Trendelenburg gait are suggestive of ALTR. An isolated head and liner exchange should be performed, typically using a ceramic head and titanium sleeve adapter. Question 94 involves late, recurrent instability, probably related to bearing surface wear and acetabular component position. Continued nonsurgical management is unlikely to succeed at this point, so it would be appropriate to proceed with acetabular component revision. Question 95 of 200 When compared to patients with a body mass index (BMI) lower than 35, patients with a BMI above 40 who undergo primary total hip arthroplasty (THA) and total knee arthroplasty (TKA) are likely to have 1- smaller incisions. 2- increased wound complications. 3- fewer 30- and 90-day readmissions. 4- lower rates of patient satisfaction. PREFERRED RESPONSE: 2- increased wound complications. DISCUSSION The obesity epidemic is increasing, and the number of patients with a BMI higher than 35 undergoing THA and TKA is increasing. Controversy exists regarding the optimal BMI cutoff and the ability to perform joint replacements safely in patients who are morbidly obese. Several clinical series as well as national database analyses have shown that morbidly obese patients undergoing THA/TKA are at increased risk for wound complications and 30- and 90-day readmissions. These patients’ incisions are typically larger because of the size of the soft-tissue envelope; although their clinical scores following successful THA/TKA often are lower than scores for controls, the overall change in clinical function and satisfaction is equivalent among nonobese and obese patients.
  • 79. AAOS 2016 Adult Reconstructive Surgery of the Hip and Knee 78 Figure 96a Figure 96b Figure 96c CLINICAL SITUATION FOR QUESTIONS 96 THROUGH 99 Figures 96a through 96c are the anteroposterior and lateral radiographs of a 64-year-old man with long-standing right knee osteoarthritis and pain unresponsive to nonsurgical treatment. This patient is scheduled for navigated cruciate-retaining right total knee arthroplasty. His range of motion is 20 to 120 degrees before surgery. Following bone resections and release of the posterolateral capsule and iliotibial band the knee is stable and extends fully, but during knee flexion there is lift-off of the anterior portion of the trial insert. Alignment is neutral to the mechanical axis. A distal femoral medial resection involved 9 mm of bone with a 9-mm-thick implant. An appropriate tibial resection was perpendicular to the long axis, and the posterior tibial slope was 7 degrees. Question 96 of 200 What is the best next step? 1- Increased distal femoral resection 2- Recession of the posterior cruciate ligament (PCL) 3- Use of a polyethylene insert with better sagittal conformity 4- Use of a thinner polyethylene insert
  • 80. AAOS 2016 Adult Reconstructive Surgery of the Hip and Knee 79 PREFERRED RESPONSE: 2- Recession of the posterior cruciate ligament (PCL) Question 97 of 200 Following treatment, the knee extends fully with good stability in extension, but there is increased anterior translation in flexion. The most appropriate next step is to 1- increase polyethylene thickness. 2- increase distal femur resection and joint line elevation. 3- exchange to a polyethylene insert with more sagittal conformity. 4- perform posterior femoral resection and downsize the femoral implant. PREFERRED RESPONSE: 3- exchange to a polyethylene insert with more sagittal conformity. Question 98 of 200 What is the most likely early postsurgical complication for this patient? 1- Knee dislocation 2- Peroneal nerve palsy 3- Patella fracture 4- Supracondylar fracture PREFERRED RESPONSE: 2- Peroneal nerve palsy Question 99 of 200 After surgery, this patient continues to experience pain and swelling of the knee with recurrent effusions. He returns to the office with continued pain 2 years after surgery. He describes instability, particularly when descending stairs. Upon examination, there is range of motion from 0 to 120 degrees with no extensor lag. The knee is stable to varus and valgus stress in extension, but there is flexion instability in both the anterior-posterior direction and in the varus-valgus direction. Bracing leads to a slight decrease in symptoms but is not well tolerated. Isokinetic testing demonstrates decreased knee extension velocity at mid push.